You are on page 1of 197

CH G,

TON HC?
H Huy Khoi

HM MOEBIUS V NH L
PHN D TRUNG HOA
Phng H Hi

V CC
CHUYN MC
KHC

DN NHP V HM ZETA RIEMANN


V PHP BIN I MELLIN
Ng Bo Chu

GII NOBEL CA EINSTEIN HAY L


SNG IN THOI C GY UNG TH HAY KHNG
m Thanh Sn

NO
thng 6 - 2016
09

Khng c g gn vi
thc tin hn l mt l
thuyt p

Tch ca hai s nguyn, mi s


l tng hai bnh phng, cng l
tng ca hai bnh phng.

ISAAC NEWTON

DIOPHANTUS

CH BIN:
Trn Nam Dng

BIN TP VIN:
V Quc B Cn
Trn Quang Hng
Nguyn Vn Huyn
Nguyn Tin Lm
L Phc L
Ng Quang Dng
Nguyn Tt Thu
ng Nguyn c Tin

NO
thng 6 - 2016
09

LI NG CHO EPSILON S 9
Ban Bin tp Epsilon

Epsilon s 9, ra mt vo thng 6, 2016 k nim chng ng mt nm ri y n lc ca Ban


Bin tp, cc tc gi v cng tc vin cng nh y n tnh t pha c gi ng h chng ti
sut t bui ban u.
S thng 9 n c vi cc bn cng l thi im ma h bt u rn r. Nhng k ngh vy
gi v hoa phng ging y sn trng, lp hc, sc lan khp nng, khp nhng con ng
khin bt k ai v tnh lt ngang qua u man mc bit bao hoi nim. Khng kh thng su
lm sng li trong chng ti, nhng ngi thc hin Epsilon, rt nhiu k c p . V vy
chng ti mun dnh ln ra mt ny gii thiu v ch Trung hc C s.
Bn cnh nhng chuyn mc nh k, cc ti v kin thc ton trung hc c s s ln lt
c trnh by trong s tp ch ma h ny.
Ban bin tp v cc cng tc vin hy vng rng, Epsilon s 9 s gp thm nim vui hc tp vo
ma h trn ngp nim vui ca cc em hc sinh, cng nh lm sng li k c mt thu bng en,
mc tm vi nhng c gi qua tui hoa nin.
Trn trng.
Ban Bin tp Epsilon.

MC LC
Ban Bin tp Epsilon
Li ng cho Epsilon s 9 . . . . . . . . . . . . . . . . . . . . . . . . . . . . . . . . . .

Phng H Hi
Hm Moebius v nh l phn d Trung Hoa . . . . . . . . . . . . . . . . . . . . . . . .

Ng Bo Chu
Dn nhp v hm zeta Riemann v php bin i Mellin . . . . . . . . . . . . . . . . . .

m Thanh Sn
Gii Nobel ca Einstein hay l sng in thoi c gy ung th hay khng . . . . . . . . .

13

ng Minh Tun
H mt m kha cng khai da trn ng cong Elliptic - Tng quan v h mt m kha
cng khai . . . . . . . . . . . . . . . . . . . . . . . . . . . . . . . . . . . . . . . . . .

15

ng Nguyn c Tin
Nghch l trai gi . . . . . . . . . . . . . . . . . . . . . . . . . . . . . . . . . . . . . .

35

Job Bouwman
Php bin i Fourier c ngha vt l g? . . . . . . . . . . . . . . . . . . . . . . . . .

39

Kiu nh Minh
Tp hp tr mt v ng dng . . . . . . . . . . . . . . . . . . . . . . . . . . . . . . . .

45

inh Trung Ha
About means of non-negative numbers and positive definite matrices . . . . . . . . . . .

59

Trnh Huy V
im Kosnita v mt s ng thng i qua n . . . . . . . . . . . . . . . . . . . . . .

65

Trn Quang Hng, Dng nh Ngc


nh l Sawayama v Thbault trong cc bi ton hnh hc thi Olympic . . . . . . . . .
4

83

Tp ch Epsilon, S 09, 06/2016


L Phc L
V bi ton tam gic 80-80-20 . . . . . . . . . . . . . . . . . . . . . . . . . . . . . . . 107
Nguyn Ti Chung
S dng tng tch phn tnh gii hn dy s . . . . . . . . . . . . . . . . . . . . . . . 113
Nguyn Ngc Giang
Sng to vi mt bi ton hnh hc Trung hc c s . . . . . . . . . . . . . . . . . . . . 125
Trung Dng
Nu bn khng nui dng, am m s t b bn" - Tr chuyn v con ng n vi
CMU ca Phm Hy Hiu . . . . . . . . . . . . . . . . . . . . . . . . . . . . . . . . . . 141
V Quc B Cn
V phong tro Olympic Ton Saudi Arabia . . . . . . . . . . . . . . . . . . . . . . . . 151
Trn Nam Dng
Bi ton hay li gii p . . . . . . . . . . . . . . . . . . . . . . . . . . . . . . . . . . 155
Trn Nam Dng
ng nht thc Brahmagupta-Fibonacci v ng dng . . . . . . . . . . . . . . . . . . . 157
Ban bin tp
Mt s bi ton trong thi vo cc trng chuyn . . . . . . . . . . . . . . . . . . . . 165
Ban bin tp
Tuyn chn cc thi Olympic nm 2016 dnh cho hc sinh THCS . . . . . . . . . . . . 175
Ban bin tp
Li gii thi Ton quc t Formula of Unity - The Third Millennium . . . . . . . . . . 181
Trn Nam Dng
Cc vn c in v hin i . . . . . . . . . . . . . . . . . . . . . . . . . . . . . . . 189

Tp ch Epsilon, S 09, 06/2016

HM MOEBIUS V NH L PHN D
TRUNG HOA
Phng H Hi
(Vin Ton hc Vit Nam)
Cu lc b (CLB) Ton hc Vin ton hc c thnh lp cch y 5 nm. u tin CLB
sinh hot sng ch nht mi tun vi cc bi ging ton ca cc GS, TS, cc thy gio chuyn
ton dnh cho hc sinh THPT. Sau to iu kin cho cc hc sinh tnh xa, CLB t
chc nhng t ging di ngy di hnh thc trng ng, trng h, trng xun. Gn y
CLB ton hc Vin ton hc t chc thm cc lp hc dnh cho hc sinh THCS vi mc
tiu nh hng v to ngun. Epsilon xin gii thiu vi bn c bi vit ca GS Phng H
Hi k li bui dy ngu hng ca mnh vo chiu th ba 7/6/2016 va qua.
Bi vit c ly t trang facebook c nhn ca gio s Phng H Hi.

hiu ni dung bi vit, ta cn n nh ngha ca hm Moebius. Hm Moebius (n) c xc


nh trn tp hp cc s nguyn dng v nhn gi tr trong tp hp {1, 0, 1}. Ta c (1) = 1.
Khi n > 1, (n) s bng 0 nu n c t nht mt c chnh phng ln hn 1. Trong trng hp
n khng c c chnh phng (ln hn 1) th vi n = p1 p2 pr l tch ca cc s nguyn t
phn bit ta s c (n) = (1)r . V d ta c bng gi tr ca hm Moebius vi n 6 15.
n
(n)

1
1

2
1

3
1

4
0

5
1

6
1

7
1

8
0

9
0

10
1

11
1

12
0

13
1

14
1

15
1

Lu ri khng report v Math Circle. Hm nay ti c mt bui dy kh ngu hng. Hm


Moebius c nhc ti vi ln. Hm nay ti mun thuyt phc cc bn trong lp rng hm
ny bng 0 ti phn ln cc s t nhin. Ni phi c sch, mch phi c chng. Ta s th lit
k xem hm s trit tiu ti nhng gi tr no.
Ta bit rng hm s trit tiu ti n nu n chia ht cho bnh phng mt s nguyn t. Nh vy
trong 10 s t nhin u tin, hm s trit tiu ti 4, 8, 9 - ba s.
Trong 10 s t nhin tip theo hm s trit tiu ti 12, 16, 18, 20 - bn s. Tnh hnh sau km
i mt cht: 32, 36, 40. Ri li kh ln: 44, 45, 48, 49, 50.
D sao th cc bn trong lp cng thy rng tnh hnh c chiu hng tt ln, ngha l cng ngy
th t l cc s m ti hm trit tiu cng tng. c bit ta thy xut hin ba s lin tip m ti
hm trit tiu: 48, 49, 50.
- Nu chng ta i tip th chng ta c th tm thy 10 s t nhin lin tip m ti hm Moebius
trit tiu. Thy tuyn b.
Khng nh c v lm cho cc bn hc sinh ngc nhin. - Chng ta thy trn c ba s lin
tip. Bn no c th chng minh rng tn ti bn s lin tip m ti hm trit tiu?
Cu hi c v hi kh.
- Hay l th ny: Chng minh rng tn ti v hn cp hai s t nhin lin tip m ti hm
Moebius trit tiu.
7

Tp ch Epsilon, S 09, 06/2016


Ngay lp tc c li gii: xt hai s a2 1 v a2 , hm hin nhin trit tiu ti a2 , ch cn chn a
l th a2 1 chia ht cho 4.
- Th ba s th sao, liu c tn ti v hn b ba s t nhin lin tip m hm Moebius trit tiu
ti ?
Sau mt lc trao i th cc bn xut li gii th ny: xt s a3 1, a3 , a3 + 1. Hm Moebius
tt nhin trit tiu ti a3 . Ta s tm a sao cho a3 1 chia ht cho 4 cn a3 + 1 chia ht cho 9. Tuy
nhin li gii cha cht ch.
Thy gii thch cho cc bn rng a3 1 chia ht cho 4 khi v ch khi a 1 chia ht cho 4 cn
a3 + 1 chia ht cho 9 khi v ch khi a + 1 chia ht cho 3. Vy bi ton l nhng s a no tha
mn cc iu kin ny?
y l mt bi ton d i vi cc bn, cu tr li c ngay: a chia 12 d 5, hay a = 12k + 5.
- Bi ton tm a a ng d vi 3 mod 4 v ng d vi 2 mod 3 chnh l bi ton gii h
phng trnh ng d bc nht, thy gii thch cho cc bn. Mi ngi rt phn khch.
- By gi ta hy xem nguyn tc gii h nh th no: Trc ht ta tm mt nghim ring, chnh
l s 5, t ta ch cn cng thm nghim tng qut, l s 12 - bi chung nh nht ca 3 v 4.
Qu d hiu i vi cc bn hc sinh.
- By gi ta hy xt mt h 3 phng trnh xem sao. Chng hn: a ng d vi 1 mod 3, ng d
vi 2 mod 4 v ng d vi 4 mod 5.
a = 34 + 60k, ch trong vi pht cc bn c cu tr li.
- Chng ta c th gii h trn dn dn, xt hai phng trnh u tin ta s c nghim: a ng
d vi 10 mod 12, sau ta li xt phng trnh ny cng vi phng trnh th ba, a ng d vi
4 mod 5, suy ra nghim bng phng php tng t.
- Bng cch ny ta chng minh c nh l phn d Trung Hoa: Mi h phng trnh ng d
bc nht vi cc modun i mt nguyn t cng nhau u c nghim. Ch cn xt dn dn, mi
ln hai phng trnh.
- Bn no bit ti sao h hai phng trnh ng d bc nht vi modun nguyn t cng nhau lun
c nghim?
- S dng h thc Bezout, mt bn tr li ngay lp tc.
Th l nh l phn d Trung Hoa c chng minh, t nht l v nguyn tc. Tr li bi ton
ban u, phng php s dng a3 ri thm bt 1 khng th cho ta li gii cho cu hi: Tm 10 s
t nhin lin tip m ti hm Moebius trit tiu. l v phng php xy dng ca ta khng
ph hp. Chng ta c th s dng nh l phn d Trung Hoa c mt cch xy dng n gin
hn nhiu. Ch cn c 5 pht mt bn c li gii hon chnh cho bi ton ny.

DN NHP V HM ZETA RIEMANN V


PHP BIN I MELLIN
Ng Bo Chu
(i hc Chicago)
c lch s ton hc ta thy rng trc y th tn l mt phng tin trao i thng tin ton
hc rt quan trng. Rt nhiu nhng tng, gi thuyt, d on v c nhng chng minh
hay phn v d c trnh by ln u tin trong nhng bc th ch khng phi l
trong cc tp ch v cun sch. Newton, Leibniz, Fermat, Euler, Gauss, Jacobi, Bernoulli,
Mersenne, ... u c nhng cng b dng th tn.
Ngy nay, vi nhng phng tin giao tip hin i nh blog, trang wordpress, archiv . . . ta
c nhiu cch cng b cc tng hay tin n phm ca mnh. Tuy nhin, trao i hc
thut bng th tn, email vn l mt hnh thc thng dng v hiu qu.
Thng tin qua email c th s khng hnh thc v cht ch nh nhng bi bo, nhng cun
sch, c nhiu tng cn th rp, nm na, nhng chnh iu lm c gi d c hn,
gn gi hn, d hiu hn.
Chng ti xin gii thiu vi bn c l th ca GS Ng Bo Chu gi cho GS Phng H
Hi nhn vic GS Hi mun t chc mt kha hc v hm zeta v php bin i Mellin
chun b cho chui bi ging v l thuyt biu din ca GS Ng Bo Chu v GS Phm Hu
Tip vo thng 8/2016.
Dch bi Nguyn V Duy Linh v Trn Nam Dng hiu nh.

From:
i Hc CHICAGO
Khoa Ton
5734 University Avenue, Chicago IL 60637
Ng Bo Chu
To:
GS. Phng H Hi
Vin Ton hc Vit Nam
18 Hong Quc Vit, H Ni
Ngy 5 thng 5 nm 2016.
Hi thn mn,
T chc mt seminar hc tp v hm zeta c v l mt tng tt.
Ti thit ngh nn m u bng mt vi vn rt cn bn ca l thuyt s gii tch. Chng ta
hy xem xt mt hm s hc tc l n thun mt dy s a1 , a2 , . . . c nh ngha theo kiu
s hc no . y l mt khi nim tru tng v ngi ta khng th lm c g mc
tru tng nh vy. Trn mt phng din khc, ta c rt nhiu v d c th:
1. an = 1,
9

Tp ch Epsilon, S 09, 06/2016


2. an = (n) l hm Moebius,
3. an = d(n) l s nhng c s ca n,
4. an = r(n) l s cch biu din n nh l tng ca hai bnh phng,
5. an = 1 nu n nguyn t v bng 0 nu ngc li,
6. an = log(p) nu n l ly tha ca p v 0 nu ngc li, ...
Mc du nhng dy s ny rt bt thng, ngoi tr dy u tin, tng Cesaro a1 + a2 + + an
c tnh tim cn no . Chng hn chng ta xc nh tnh tim cn ca dy th 5, chng ta th
tnh s lng s nguyn t nh hn mt s no .
c lng tng Cesaro ca d(n) v r(n) l nhng vn s cp c in trong l thuyt s. i
vi r(n), l nhng bi ton v hnh trn Gauss: C bao nhiu im nguyn nm trong hnh
trn khi bn knh tin v . Bng nhng l lun s cp nh Gauss lm, chng ta c
X
r(n) = X + O(X 1/2 )
(0.1)
n<X

Seminar c th bt u vi bi ton ny v ngi ta khng cn c kin thc hiu cch gii. N


c gii thch rt k lng trong chng 6 ca quyn Nhp mn l thuyt s gii tch ca
Chandrasekharan. Ti ch ny, ta cn cp rng sai s O(X 1/2 ) c th ci thin ng k nh
mt cng c gii tch tt hn. Chng hn, nh cng thc ly tng Poisson, ngi ta c th t
c O(X 1/3 ).
Cch truyn thng hiu tnh tim cn ca tng Cesaro l to ra chui Dirichlet
X
D(s, a) =
an ns ,
n

vi bin phc s. Bng cch p t iu kin tng trng cho an , chui Dirichlet hi t trn mt
na mt phng <(s) > no .
hn chui Dirichlet tng ng vi dy an = 1 chnh l
P Chng
s
Riemann -function (s, a) = n n hi t trn min <(s) > 1.
C mt vi thao tc m chng ta nn lm quen khi lm vic vi chui Dirichlet. Chng hn,
trong bn v d u tin, an c tnh cht nhn tnh: amn = am an vi (m; n) = 1. Chui Dirichlet
tng ng c th phn tch thnh nhn t thnh tch Euler tc l tch ca cc tha s nh s
bng s nguyn t. Chng hn nh:
(s) =

(1 ps ) .

(0.2)

Mt nhn xt khc nhng theo li tng t l chui Dirichlet tng ng vi tch chp ca cc
hm s hc. Chng hn nh
X
d(n)an ns = (s)2 ,
(0.3)
n

v
X

(n)an ns = (s)1 .

10

(0.4)

Tp ch Epsilon, S 09, 06/2016


Nhng hng ng thc ny l hin nhin khi xem xt nh l chui Dirichlet hnh thc. Nh l
cc hm phc, chng c th c chng minh d dng trn min m chui Dirichlet hi t tuyt
i. Tt c nhng iu ny c gii thch hon ho trong chng 2 ca quyn Nhp mn l
thuyt s gii tch ca Apostol.
Mt trong nhng hng chnh ca l thuyt s gii tch c in l dn ra mt c lng tim cn
ca tng Cesaro t nhng cc ca chui Dirichlet. chnh l ni dung ca nh l Tauberian.
Mt vn kh chu l c nhiu phin bn ca nh l Tauberian. Phin bn n gin nht c th
l nh l Wiener-Ikeharas c gii thch trong chng 11 ca Chandrasekharan chng hn.
Trong bt c
hp no, nh l Tauberian cho ta bit mt iu i loi nh sau: Gi s rng
Ptrngs
D(s, a) = n an n c th thc trin phn hnh vt qua ng thng <(s) = 1 . Gi s
rng n ch c mt cc cp 2 ti 2 v mt cc cp 1 ti 1. Khi biu thc tim cn s c dng:
X
an = c2,2 X 2 log(X) + c2,1 X 2 + c1,1 X +
(0.5)
n<X

trong c2,2 , c2,1 l h s Laurent ca khai trin D(a, s) thnh chui Laurent ti s = 2 . . . y
s m 2 trong X 2 tng ng vi v tr ca cc ti 2, s hng log(X) xut hin l v cc ti 2 c
cp 2 . . .
Mt nhiu thi gian cho cc nh l Tauberian cng khng hay lm v trn mt phng din n
ch l vn k thut, v trn mt phng din khc phn gii tch t i cn dng chng minh
cc nh l Tauberian c v l khng thch ng xt trn quan im l thuyt s.
Nu nh ti khng lm th c mt cch tip cn tt hn s dng ci gi l tng trn. Ngi ta
c th c bi post ca Emmanuel Kowalski trn blog ca ng ta c gii thiu v tng trn:
https://blogs.ethz.ch/kowalski/smoothing-sums-wiki-page/
Cch tt nht khi u vi tng trn l quay v vi bi ton hnh trn Gauss. Chng ta pht
biu li bi ton hnh trn Gauss tng qut nh sau: Cho hm : R2 C, chng hn nh hm
c trng ca a n v, chng ta mun bit tim cn ca
X
(tn)
(0.6)
nZ 2

khi t 0. Nu l hm c trng ca a n v, khi

(tn) s l s nhng im nguyn

nZ 2

bn trong a bn knh t1 .
im chnh yu ca chin lc tng trn l nhn ra rng thay v lm vic vi mt hm th
th, nh l hm c trng ca a n v c nhng bc nhy th trn chu vi ca ng trn,
tt hn chngP
ta lm vic vi nhng hm th trn vi gi thit compact. i vi nhng hm trn,
tim cn ca nZ 2 (tn) l h qu trc tip ca cng thc ly tng Poisson:
X
X
1 n)
(tn) = t1
(t
(0.7)
nZ 2

nZ 2

trong l bin i Fourier ca Bin i Fourier khng cn h tr compact na, nhng


vn mt hm gim nhanh trong khng gian Schwartz. v phi, khi t1 , mi s hng,
ngoi tr n = 0, gim rt nhanh. Do vy ta c
X
+ O(tr ), vi mi r.
(tn) = t1 (0)
(0.8)
nZ 2

11

Tp ch Epsilon, S 09, 06/2016


S hng tng ng vi din tch ca a, ni xut hin ca s trong bi ton hnh trn Gauss.
Mc d vy, nu chng ta p dng trc tip cng thc ly tng Poisson cho hm c trng ca
a n v, chng ta s gp rc ri to v nhng im k d ln ca s dn ti s tng nhanh ca
Tt hn chng ta xp x hm c trng ca a bng mt hm trn no v
bin i Fourier .
c gng khng ch sai s. Lm nh vy theo mt ng li thng minh, chng ta nhn c sai
s O(t1/3 ). Ti nh thy li gii thch rt p ny u nhng ti khng tm thy n na.
Nhng n s l mt bi tp tht tt cho nhng ai tham d seminar ca anh.
Mt bi hc rt ra t php gii bi ton hnh trn Gauss l chng ta c th qun i hm th th
v thay vo ch lm vic mt hm trn. Phn l thuyt s hon ton ni v hm th trn.
i vi mi bi ton c th, ngi ta phi ct cng xp x mt hm th th bng nhng hm trn.
iu ny cng cho ta mt hnh dung tt hn v cc hm s hc v cc chui Dirichlet theo ngha
cc phn b. Mi dy an tng va phi c th xem nh phn b
X
7 a() =
an (n)
(0.9)
n

Tch
ca hm s hc c th chuyn thnh tch chp ca phn b. ng thi chui Dirichlet
P chp
s
chP
l bin i Mellin ca phn b a(). Chng hn nu bin i Mellin ca
n an n
phn b 7 n (n) l hm Riemann zeta (s). Ta cn phi quen vi cc phn b mi c th
hiu r bin i Mellin ca mt phn b. Cch lm ny c mt phn thng kh ln: Ngi ta c
th tc b nhng c lng trung gian cng knh ca gii tch Fourier v x l mi th nh
tnh ton i s.
Ngi ta c th nh ngha bin i Mellin ca mt phn b loi a nh sau. i vi mt hm
Schwartz , tch chp a li l mt hm Schwartz. C hai bin i Mellin ca v a vn
cn l hm trn vi tim cn va phi. Khi chng ta c th nh ngha bin i Mellin M ()
v M (a ) ri cho M (a) = M (a )/M ().
tng ny c v c nhiu ngi bit, mc du ti khng nh r n c gii thch trong
ti liu no. t nht n c gii thch trong cc bc th ngn ti gi cho anh trc y v
hm . Nu khng th anh c th c v php bin i Mellin trong bi ging ca Igusa
trong TIFR: http://www.math.tifr.res.in/publ/ln/tifr59.pdf.Zagier c
vit bi gii thiu s cp hn: http://people.mpim-bonn.mpg.de/zagier/files/
tex/MellinTransform/fulltext.pdf
By gi ngi ta c th tip cn hm Riemann zeta theo li phn b. Cng
P thc ly tng Poisson
c th c pht biu nh sau: phn b lc Dirac 7 Z () =
(n), l php bin i
nZ

Fourier ca chnh n. By gi tnh bin i Mellin v bin i Fourier ca Z ta c phng trnh


hm ca hm Riemann zeta. Trong qu trnh tnh ton, ngi ta phi chng minh rng c thc
trin phn hnh da trn s tc ng ln nhau ca php bin i Mellin, tim cn ca tng Cesaro
v cng thc ly tng Poisson.
Mt bi tp rt tt l th tng qut ha l thuyt ca cc hm zeta Riemann nh gii thch
trn i vi m rng bc hai F ca Q. Chng hn trong trng hp tch c F = Q Q,
chng ta cn phi quay tr li dy an = d(n). Nu nh F = Q[i] l trng hu t Gauss, chng
ta cn phi quay tr li bi ton hnh trn Gauss ...
Ti im ny ngi ta c th hi, phi chng php ly tng Poisson l tt c nhng g chng ta
cn c lng cc tng Cesaro? Xt cho cng, chng ta cn chui Dirichlet lm g? Cu
12

Tp ch Epsilon, S 09, 06/2016


tr li l php ly tng Poisson lm vic vi tng c ch s nguyn v khng th lm g vi tng
c ch s l s nguyn t. Chng hn php ly tng Poisson mt mnh n khng th lm vic vi
cc dy (0.5) v (0.6). Ni cch khc php ly tng Poisson mt mnh n khng th chng minh
c nh l s nguyn t.
i vi nh l s nguyn t, ni chung chng ta cn phi x l nh sau: Chng ta bt u vi
dy (0.1), chng ta to chui Dirichlet chnh l hm zeta Riemann. Chng ta dng cng thc ly
tng Poisson chng minh rng c thc trin phn hnh v phng trnh hm. Chng ta suy ra
rng d log cng c thc trin phn hnh v phng trnh hm. Gi y d log l chui Dirichlet
ca dy (0.6). Chng ta hon tt cng vic nu chng ta bit cc cc ca d log . Nhng cc cc
ca d log cn bn l cc khng im ca , chnh l l do chng ta mun nh v cc khng
im ca .
Chng hn, nh l s nguyn t da trn s kin l khng c khng im trn mt min m
no cha <(s) > 1, iu ny c chng minh bi Hadamard c trm nm v trc.
Nu anh c thi gian v can m, anh hy nghin cu li tt c nhng th cng nh nh l
Dirichlet v s nguyn t trong cp s cng. l ch m chng ta cn mt lut thun nghch
no . Nhng anh khng cn bt u vi lut thun nghch v tht s ra n khng phi l hng
chnh trong s pht trin ban u ca hm zeta, t ra theo kin ca ti. Tt nhin v sau lut
thun nghch s trn ln rt nhiu vo l thuyt ca hm zeta cng nh trong chng trnh ca
Langlands, nhng mi lc hc mt th th vn d dng hn.
Chc anh mi iu tt lnh
Chu

13

Tp ch Epsilon, S 09, 06/2016

14

GII NOBEL CA EINSTEIN HAY L SNG


IN THOI C GY UNG TH HAY KHNG
(m Thanh Sn - H Chicago)

Bi vit c tham kho t blog ca gio s m Thanh Sn.


Albert Einstein c l l nh vt l ni ting nht t trc n nay. Cng chng thng bit n
ng ta nh ngi khm ph ra thuyt tng i, lm thay i quan nim ca chng ta v khng
gian v thi gian. Thuyt tng i bao gm thuyt tng i hp, c Einstein tm ra nm
1905 v thuyt tng i rng c ng ta tm ra 10 nm sau. Tuy nhin c th khng phi ai
cng bit l gii thng Nobel v vt l nm 1921 c trao cho Einstein v mt khm ph khc
ca ng: Hiu ng quang in. y l cng trnh Einstein vit cng vo nm 1905, cng nm
vi cng trnh v thuyt tng i hp v mt cng trnh na v chuyn ng Brown. Hiu ng
quang in l ng gp ln nht ca Einstein vo thuyt lng t, l thuyt m sau ny c
Bohr, Heisenberg, Schrodinger v nhiu ngi khc pht trin ln nhng li b Einstein nghi ng
n cui i.

Hiu ng quang in l hin tng khi ta chiu nh sng vo mt tm kim loi th thnh thong
in t b bt ra khi kim loi. Ta c th on l nh sng cng mnh th cng nhiu in t
b bt ra. Phn on ny ho ra l khng hon ton ng: C nhng ngun nh sng rt mnh
khng gy ra hiu ng quang in, nhng c nhng ngun yu hn li gy ra hiu ng ny. Thc
nghim cho thy rng hiu ng quang in ph thuc vo tn s ca nh sng. V d, vi cng
mt mu kim loi, nh sng hoc tia hng ngoi khng gy ra hiu ng nhng nh sng tm
hoc cc tm li c tc dng.
15

Tp ch Epsilon, S 09, 06/2016


Einstein gii thch iu ny bng cch p dng v m rng gi thuyt lng t ca Planck.
Einstein gi thuyt rng nh sng bao gm cc ht photon, mi ht mang mt nng lng t l
thun vi tn s ca nh sng.
E = hv.
y E l nng lng ca ht photon, v l tn s ca nh sng, v h l hng s Planck. Cng
thc trn c tn l cng thc Planck, cng thc m theo ti ng l ra phi ni ting hn cng
thc E = mc2 .
Hiu ng quang in l qu trnh mt ht photon truyn nng lng cho mt ht in t. bt
mt in t ra khi mnh kim loi ta cn mt nng lng ti thiu nht nh, ta gi l . Nh
vy ch khi v > h nh sng mi c th bt c in t ra khi khi kim loi. Nu v < h th
ngun sng c mnh th no cng khng c photon nng lng gy ra hiu ng quang
in. Bn c th hi liu c khi no hai ht photon, hoc nhiu hn, cng hp sc bt ra mt
in t hay khng. iu ny v nguyn tc c th xy ra, nhng xc sut rt thp, c th b qua.
Hiu ng quang in c lin quan trc tip n mt cu hi hay c t ra hin nay: in thoi
di ng c gy tc hi cho sc kho hay khng? Mt trong nhng iu lm nhiu ngi lo lng
l kh nng gy ung th ca sng in thoi (v d xem bi ny). Nhiu ngi cn ni l sng
in t trong l vi sng cng c th gy ra ung th.
Nu ta nh li cng thc E = hv ca Einstein th ta s thy nhng lo lng ny khng c c s.
l do tn s sng ca cc thit b in t qu thp c th gy ra nhng bin i ca phn
t ADN. Tn s sng trong l vi sng l 2500 MHz, tn s ca in thoi di ng l 800 MHz
hay 1900 MHz. Hng s Planck l 4 109 eV/MHz, nh vy 2500 MHz tng ng vi nng
lng 10 phn triu eV, trong khi cc qu trnh ho hc hay sinh ho cn nng lng c t nht
0.1 eV, nu khng phi l 1 eV. S chnh lch n 10 100 nghn ln gia hai c nng lng
lm cho l vi sng hay in thoi di ng khng th lm bin i gien ca ming tht trong l
hay c th chng ta. (Tia cc tm th li khc, v tn s ca tia cc tm cao hn tn s ca in
thoi di ng n c triu ln, nn n c nng lng gy tc hi cho t bo).
Tt nhin l in thoi di ng hay cc thit b in t c th c nhng tc hi khc, v d cho
tm l hay l gic ng ca ngi dng, nhng chng ngoi khun kh ca bi vit ny.

16

H MT M KHA CNG KHAI DA TRN


NG CONG ELLIPTIC - TNG QUAN V
H MT M KHA CNG KHAI
ng Minh Tun
(Vietkey)
Trong s ny, Epsilon trn trng gi n c gi phn u tin ca chuyn
v h mt m kha cng khai da trn ng cong Elliptic ca tc gi ng Minh
Tun. Phn sau ca chuyn s c chuyn ti tp ch s 10.
Sau y l ton vn ca li m u v chng 1 ca lot chuyn .
LI M U
Thng 3 nm 2016, B Ngoi Giao Hoa K, ng u l b trng John Kerry,
dn mt on i biu ti cc nc ASEAN trong c Vit Nam tho lun v
pht trin Fintech v c bit l v cng ngh Blockchain1 .Thng 9 nm 2015, y
ban giao dch hng ho tng lai M cng b, Bitcoin chnh thc c a vo
danh sch hng ha c php giao dch ti M2 .Cng ngh Blockchain v Bitcoin
l cng ngh tin s ra i nm 2009 v ngy cng c nhiu quc gia v cc t chc,
doanh nghip cho php lu hnh v thanh ton bng loi tin s ny trong khng
gian mng Internet ton cu. Thng 4-2016, gi tr thng mi ca Bitcoin ln
n 6.5 t USD 3 . Nn tng c s ca Bitcoin chnh l l thuyt v mt m m c th
y l hm bm v l thuyt v ch k s da trn H mt ng cong Elliptic
(ECC).
Bn cnh vic s dng trong tin s Bitcoin4 , ECC cn c ng dng rt
nhiu trong thc tin ngnh Cng ngh thng tin [1]. Cc trang Web bo mt https
(http-secure) thng c dng trong thanh ton in t hay ng dng ring t nh
gmail u s dng cc giao thc TLS (Transport Layer Security) m trc l
1

B. Cohen. (April 12, 2016) U.S. State Department Recommends Development of Blockchain and Distributed Ledgers to International Partners. [Online]. Available: http://www.nasdaq.com/article/us-state-departmentrecommendsdevelopment-of-blockchain-and-distributed-ledgers-to-international-partnerscm605334.
2
(21/09/2015) Bitcoin chnh thc c M cng nhn l hng ho. [Online]. Available: http://vnreview.vn/tintuc-kinh-doanh/-/view-content/content/1654484/bitcoin-chinh-thuc-duoc-My-cong-nhan-la-hang-hoa.
3
(15/4/2016) https://markets.blockchain.info/
4
a ch v Bitcoin c tnh da trn kha cng khai ca ECC vi hm bm bo mt c di 256-bit
SHA256 v thut ton Base58Encode dng chuyn s thnh dng 56 k t, RIPEMD (RACE Integrity Primitives
Evaluation Message Digest) l mt h hm bm bo mt khc:
Version = 1(byte)
KeyHash = Version + RIPEMD(SHA256(PublicKeyEC))
CheckSum = SHA256(SHA256(KeyHash))
BitcoinAddress = Base58Encode(KeyHash + CheckSum)

17

Tp ch Epsilon, S 09, 06/2016


SSL (Secure Socket Layer). Trong cc giao thc ny ECC c s dng trao i
kha phin. Cc giao dch remote access c s dng rt nhiu trong th gii Unix,
Linux l SSH (Secure SHell) cng s dng ECC trao i kha. u im ca h
mt s dng ng cong Elliptic (ECC) l c di kha nh (160 bit tng ng
vi kha di 1024 Bit trong h mt RSA), do s dng di kha nh nn ti
nguyn phc v cho ECC thng nh hn rt nhiu, bn cnh hiu nng tnh ton
cng c nng cao r rt. Hin nay ECC ang l xu th thay th RSA.
C s ton hc ca h mt ECC l nhm giao hon Abel cc im nm trn
ng cong Elliptic. Ngoi vic ng cong Elliptic l c s cho h mt ECC, h
mt ID-Based, ng cong Elliptic (EC) cn l cng c hu hiu phn tch s
nguyn ra tha c nguyn t [2, 3, 4], hoc dng kim tra tnh nguyn t ca
s nguyn [3]. EC cng l c s chng minh nh l Fermat ni ting tn ti
nhiu trm nm qua.
ng cong Elliptic l mt trng hp c bit ca phng trnh Diophant. L
thuyt v ng cong Elliptic (EC) rt phong ph v s. Trong [5] tc gi Serge
Lang pht biu v phng din hc thut: C th vit v tn v ng cong
Elliptic. Cc l thuyt v khi nim lin quan ti EC c th lit k mt s nh di
y:

L thuyt nhm, vnh, trng trong i s tru tng [3, 6];


a tp Affine, a tp Jacobian v a tp x nh trong hnh hc i s [7, 8];
im Torsion, Divisor, cp song tuyn tnh Weil, Tate-Lichtenbaum [3];
L thuyt trng Galois, t ng cu-nh x Frobenius [3, 9];
L thuyt Baker-Feldman, Baker-Tijdeman v l thuyt Kummer [5];
S p-adic, Isogenies, hm Sigma v hm Zeta [5, 7, 10, 3, 4];
Nhm i ng iu, i ng iu Galois v i ng iu phi giao hon
(Topo i s) [8, 4];
Nhm MordellWeil, Selmer v nhm ShafarevichTate [8, 11];
Phng php hnh hc v Ta tuyn tnh (Quasilinear) [12].
Vi ngha to ln c v thc tin v hc thut, EC l nn tng ton hc quan
trng trong i s hin i cng nh l thuyt mt m hin i. EC cng l nn tng
quan trng trong chnh ph in t v thng mi in t. Chnh v nhng iu ny
m Chuyn H mt m kha cng khai da trn ng cong Elliptic c la
chn trnh by bo co.
Vi khi lng kin thc v khi nim s nh lit k trn vic nghin
cu v o su v ng cong Elliptic gp khng t kh khn cho nhng ngi lm
Cng ngh thng tin (CNTT) m ton hc khng phi l chuyn mn chnh. Mc
tiu ca chuyn ny l tng hp nhng khi nim v kin thc c bn nht ca
EC lin quan n c s ton hc ca H mt da trn ng cong Elliptic. ng
thi ngi vit cng chng minh li mt s nh l v b theo cch d hiu hn,
trnh dng n cc khi nim qu phc tp v xa l vi chuyn ngnh CNTT. Cc
php ton ca ng cong Elliptic c trnh by trong bo co phn ln c ci
t bng phn cng s dng cng ngh FPGA (Field-Programmable Gate Array)
ca Xilinx trong khun kh ti cp Nh nc KC01-18 ( c nghim thu
trong nm 2014) do ngi vit bo co lm ch nghim ti, kt qu c cng b
ti [13].
18

Tp ch Epsilon, S 09, 06/2016


Phm vi ca chuyn cng c gii hn vi nhng khi nim v l thuyt
cho cc ng dng c bn ca EC, cc pht trin ca EC thnh h mt ID-Based,
hoc cc ng dng v ch k s tp th, ch k s nhm, ch k ngng, ch k y
nhim, ch k s m s khng c cp n trong khun kh ca bo co ny.
Bo co chuyn c kt cu thnh 02 chng, chng 1 trnh by cc khi
nim, nh ngha c bn v ng cong Elliptic (Phng trnh ca EC, nhm cng
Abel cc im trn ng cong, chng minh nh l v nhm...). Chng 2 trnh
by v H mt da trn ng cong Elliptic v mt s ng dng trong m ha, xc
thc ch k s, trao i kha d trn bi ton kh Logarithm ri rc.

K hiu
tp hp s t nhin
tp hp s t nhin khc 0
trng s nguyn
trng s hu t
c s ca trng K
du ng d
dng v cng (tng ng vi +)
c s chung ln nht
bc ca a thc
nh thc
H m cng khai da trn bi ton phn tch ra tha s nguyn t
do Rivest-Shamir-Adleman pht trin
EC
ng cong Elliptic (Elliptic Curve)
ECC
H mt da trn ng cong Elliptic (Elliptic Curve Cryptography)
ECDLP Elliptic Curve Logarithm Problem
ECDH
Thut ton Elliptic Curve DiffieHellman
ECDSA The Elliptic Curve Digital Signature Algorithm
ECIES
The Elliptic Curve Integrated Encryption System
ECMQV Elliptic Curve MenezesQuVanstone protocol

N
N
Z
Q
char(K)

gcd
deg
det
RSA

1. Tng quan v ng cong Elliptic


Nm 250 sau Cng nguyn, Diophant khi gii bi ton tm s tng ca thp cc qu cu m khi
tri ra mt t c th xp thnh mt hnh vung dn n gii phng trnh (y l s qu cu
trn 1 cnh hnh vung; x l s tng ca thp):
y 2 = 12 + 22 + 32 + + x2 =

x(x + 1)(2x + 1)
6

Phng trnh y 2 = x(x + 1)(2x + 1)/6 l mt dng ca ng cong Elliptic.


Nm 1637, nh ton hc v vt l hc ngi Php Pierre de Fermat cng b nh l Fermat cui
cng khi vit trn l bn copy cng trnh ca Diophant: Phng trnh sau y l v nghim:
xn + y n = z n ,
19

n>2

Tp ch Epsilon, S 09, 06/2016


Hn ba th k, c rt nhiu nh ton hc c gng chng minh nh l ny xong u tht bi,
mi cho n nm 1994, Andrew Wiles, gio s trng Princeton gy mt ting vang ln
trong cng ng ton hc th gii vo thi im khi s dng ng cong Elliptic c dng
y 2 = x(x an )(x + bn ) cng vi l thuyt v Modul chng minh nh l Fermat cui cng.
Nm 1987, Trong [14], Lenstra xut thut ton phn tch s nguyn ra tha s nguyn t s
dng ng cong Elliptic, l thut ton tng i nhanh, chy vi thi gian di hm m v
l thut ton nhanh th 3 trong vic phn tch ra tha s nguyn t, sau phng php sng a
thc ton phng v phng php sng trng s tng qut.
Trong lnh vc mt m, vo nm 1985, Victor S. Miller cng b bi bo u tin v ng dng
ng cong EC trong mt m Use of Elliptic Curves in Cryptography [15] v sau l Neal
Koblitz vi Elliptic curve cryptosystem [16] vo nm 1987. T cho n nay c rt nhiu
cng b nghin cu v EC v l thuyt v trong thc tin cng ngy ng dng ECC cng c s
dng rng ri, v c a thnh cc tiu chun.
Mt s tiu chun lin quan n ng cong Elliptic:
IEEE 1363:

Tiu chun ny bao gm gn nh tt c cc thut ton


v cc h kha cng khai trong c ECDH, ECDSA,
ECMQV v ECIES. Trong phn ph lc c c cc
thut ton c bn v l thuyt s lin quan n h mt
kha cng khai.
ANSI X9.62 v X9.63: Cc chun ny tp trung vo ng cong Elliptic v
c th v ECDSA trong X9.62 v ECDH, ECMQV
v ECIES trong X9.63. Cc chun ny cng xc nh
khun dng cc d liu v danh mc cc ng cong
khuyn co s dng.
FIPS 186.2:
Tiu chun ca NIST cho ch k s, m t chi tit v
thut ton DSA algorithm.
L tiu chun c bin son bi nhm cc doanh
SECG:
nghip dn dt bi cng ty Certicom, gn nh l nh
x ca cc chun ANSI nhng c tip cn trn mi
trng Web t Website http://www.secg.org/
ISO 15946-2:
Tiu chun m t v ECDSA v ECIES (cn c gi
l ECIES-KEM).
RFC 3278:
Use of Elliptic Curve Cryptography (ECC) Algorithms in Cryptographic Message Syntax (CMS) l
khuyn ngh s dng thut ton ECC trong m ha
thng ip vn bn.

2. Phng trnh Weierstra ca ng cong Elliptic


Trong ti liu ny, a phn s cc ng cong Elliptic s c nghin cu di dng sau:
y 2 = x3 + Ax + B,

(2.1)

Trong A v B l cc hng s. Cc gi tr ca x, y, A, B thng l cc gi tr trn mt trng


no , v d nh R (s thc), Q (s hu t), C (s phc), hoc trng hu hn Fq , vi q = pn
trong p l s nguyn t vi n = 1. Nu K l mt trng c a, b K, khi ta ni ng cong
20

Tp ch Epsilon, S 09, 06/2016


Elliptic c nh ngha trn trng K. im (x, y) trn ng cong Elliptic vi (x, y) K
c gi l im KHu t. Dng tng qut phng trnh Weierstrass ca ng cong Elliptic
s c biu din di dng:
y 2 + a1 xy + a3 y = x3 + a2 x2 + a4 x + a6 ,

(2.2)

Trong a1 , , a6 l cc hng s. Dng (2.2) thng c s dng vi cc trng K c c s


chap(K) bng 2 hoc 3. Khi K c chap(K) khc 2 c th bin i (2.2) thnh dng sau:
2




 2


a21 2
a1 a3
a3
a1 x a3
3
+
= x + a2 +
x + a4 +
x+
+ a6 ,
y+
2
2
4
2
4
C th vit li nh sau:
y12 = x3 + a02 x2 + a04 x + a06 ,
Vi y1 = y + a1 x/2 + a3 /2 v vi cc hng s a02 , a04 , a06 . Khi K c chap(K) khc 3 c th dng
php th x1 = x + a02 /3 v ta c:
y12 = x31 + Ax + B,
Trong A, B l cc hng s no . ng cong (2.1) c nh thc = 16(4A3 + 27B).
ng cong ny s suy bin v khng c 3 nghim phn bit khi = 0, trong ti liu ny
chng ta ch xt cc ng cong c 6= 0.

3. Cng cc im trn ng cong Elliptic


Xt hai im P1 = (x1 , y1 ) v P2 = (x2 , y2 ) trn ng cong Elliptic E : y 2 + a1 xy + a3 y =
x3 + a2 x2 + a4 x + a6 . Php cng gia hai im trn ng cong E c nh ngha nh sau:
P3 (x3 , y3 ) = P1 (x1 , y1 ) + P2 (x2 , y2 )

(3.1)

Trong P3 (x3 , y3 ) = P30 (x3 , y30 ), im P30 (x3 , y30 ) l giao im ca ng cong E v ng
thng i qua P1 v P2 . V 2 im P3 (x3 , y3 ) v P30 (x3 , y30 ) u nm trn ng cong E nn
(x3 , y3 ) v (x3 , y30 ) phi tha mn phng trnh (2.2). Cng thc tnh cc gi tr (x3 , y3 ) s
c chng minh di y.
Trong cc cc ti liu c bn v nng cao c tham chiu nhiu v ng cong Elliptic nh
[3, 7, 8] ngi vit vn cha tha mn vi cc dn dt v chng minh cng thc tng qut cho
cc gi tr (x3 , y3 ), do cc cng thc ny s c chng minh chi tit trong ti liu ny. ng
thng i qua 2 im P1 v P2 c phng trnh l:
y = x +

(3.2)

Trong l h s gc ca ng thng i qua P1 , P2 . Ta c:


y1 = x1 +
y2 = x2 +
y30 = x3 +
21

(3.3)
(3.4)
(3.5)

Tp ch Epsilon, S 09, 06/2016

Hnh 5.1: Php cng trn ng cong Elliptic

3.1. Trng hp 2 im khng trng nhau P1 6= P2


T (3.3) v (3.4) suy ra: y1 y2 = (x1 x2 ), khi P1 6= P2 , ngha l x1 6= x2 ta c cng thc:
=

y1 y2
x 1 x2

= y1 x1 = y1

(3.6)
y1 y2
x1 y2 x2 y1
x1 =
x1 x 2
x 1 x2

(3.7)

Tip theo thay y (3.2) vo phng trnh (2.2) ta c:


(x + )2 + (a1 x + a3 )(x + ) = x3 + a2 x2 + a4 x + a6

(3.8)

T dn n phng trnh r(x) = 0 vi:


r(x) = x3 + (a2 2 a1 )x2 + (a4 2 a3 a1 )x + a6 2 a3
22

(3.9)

Tp ch Epsilon, S 09, 06/2016


Bit rng r(x) c 3 nghim phn bit nn c th vit:
r(x) = (x x1 )(x x2 )(x x3 )
= (x2 (x1 + x2 )x + x1 x2 )(x x3 )
= x3 (x1 + x2 )x2 + x1 x2 x x3 x2 + (x1 + x2 )x3 x x1 x2 x3
= x3 (x1 + x2 + x3 )x2 + (x1 x2 + x1 x3 + x2 x3 )x x1 x2 x3

(3.10)

ng nht cc h s x2 ca r(x) 2 phng trnh (3.9) v (3.10) ta c: x1 + x2 + x3 =


(a2 2 a1 ) t y c th tnh c x3 theo cng thc sau:
x3 = 2 + a1 a2 x1 x2

(3.11)

n y cn phi tnh tip gi tr y3 , lc ny x3 tnh xong nn c th coi l hng s, c th


vit li (2.2) thnh dng sau:
y 2 + (a1 x3 + a3 )y (x33 + a2 x23 + a4 x3 + a6 ) = 0

(3.12)

Phng trnh bc 2 ny c 2 nghim l:

(a
x
+
a
)

1
3
3
(3.13)
y3 , y30 =
21
Cng 2 nghim ny ta s c: y30 + y3 = a1 x3 a3 , mt khc do y30 nm trn ng thng P1 , P2
nn y30 = x3 + . T y c th tnh c y3 theo cng thc 5 :
y3 = x3 a1 x3 a3

(3.14)

Thay t (3.7) ta c th tnh y3 di dng sau:


y3 = (x1 x3 ) y1 a1 x3 a3

(3.15)

3.2. Trng hp 2 im trng nhau P1 = P2


Khi ny x1 = x2 v y1 = y2 do cng thc tnh (3.7) khng s dng c v xut hin php
chia s 0. Trong trng hp ny chnh l h s gc ca ng thng tip tuyn ng cong E
dy
ti P1 hay P2 . H s gc ca tip tuyn ca E chnh l o hm dx
, s dng cc quy tc ly o
hm ca tch, o hm ca hm s hp v ly o hm 2 v ca phng trnh (2.2) theo dx ta c:
d(y 2 + a1 xy + a3 y)
dx
d(y 2 ) d(a1 xy) d(a3 y)
+
+
dx
dx
dx
2
d(y ) dy
d(xy)
dy

+ a1 (
) + a3
dy
dx
dx
dx
dy
dy
dy
2y
+ a1 (y + x ) + a3
dx
dx
dx
dy
(2y + a1 x + a3 )
dx
dy
dx
5

d(x3 + a2 x2 + a4 x + a6 )
dx

= 3x2 + 2a2 x + a4
= 3x2 + 2a2 x + a4
= 3x2 + 2a2 x + a4
= 3x2 + 2a2 x + a4 a1 y
=

3x2 + 2a2 x + a4 a1 y
2y + a1 x + a3

Ghi ch: Cc tc gi H. Cohen v G. Frey trong cun Handbook of Elliptic and Hyperelliptic Curve Cryptography [7] trnh by din gii v cch tnh y3 da trn s i xng qua trc x l khng chnh xc v thiu r rng
i vi ng cong Elliptic dng tng qut, cch gii thch ca ngi vit bng cch gii phng trnh bc 2 y
c th coi l ng n v r rng hn.

23

Tp ch Epsilon, S 09, 06/2016


Nh vy vi im P1 (x1 , y1 ) ta c:
=

3x21 + 2a2 x1 + a4 a1 y1
2y1 + a1 x1 + a3

(3.16)

Trong tt c cc trng hp im P3 l tng ca 2 im P1 , P2 s l im c ta l:


P3 (x3 , y3 ) = (2 + a1 a2 x1 x2 , (x1 x3 ) y1 a1 x3 a3 )

(3.17)

Vi ng cong E dng (2.1), khi a1 = a3 = a2 = 0 v P3 s c tnh theo cng thc:


P3 (x3 , y3 ) = (2 x1 x2 , (x1 x3 ) y1 )

(3.18)

Trong trng hp P1 = P2 , (3.16) s c bin i thnh:


3x21 + a4
=
2y1

(3.19)

4. Nhn v hng cc im trn ng cong Elliptic


Vi n N \ {0} nh ngha php nhn v hng ca im P nm trn ng cong E l php
cng n ln chnh bn thn im P :
P 7 nP = P
| +P +
{z + P} = Q
n ln

ti u php nhn v hng, c th s dng phng php Nhn i-v-cng, u tin biu
din s n di dng: n = n0 + 2n1 + 22 n2 + + 2m nm vi [n0 . . . nm ] {0, 1}, sau p
dng thut ton:
Algorithm 1 Phng php Nhn i-v-cng
1: Q 0
2: for i = 0 to m do
3:
if ni = 1 then
4:
Q Cngim(Q,P)
5:
end if
6:
P Nhni(P)
7: end for
8: return Q
Ngoi phng php Nhn i-v-cng, c th s dng phng php Trt-ca-s. Cc phng
php ny cho php nhn v hng mt cch ti u.
Lu ca ngi vit:
Khng tn ti php nhn 2 im trn ng cong E, c ngha l khng tn ti P Q vi
P, Q E.
Khng tn ti thut ton chia v hng Q : n. Bit rng Q = nP , bi ton tm s n l bi
ton Logarithm ri rc s c cp ti chng sau. y l bi ton kh, thng thng
phi th ln lt n = 1, 2, . . . , n 1 php cng im P , cho n khi tng bng Q, tuy
nhin c mt s thut ton ti u hn tm n nhng vn khng th gii c bi ton ny
trong thi gian a thc v th da vo kh ny c th xy dng ra h mt ng cong
Elliptic vi cc giao thc cho m ha, xc thc v trao i kha.
24

Tp ch Epsilon, S 09, 06/2016

Hnh 5.2: V d v tnh cht kt hp trn ng cong Elliptic

5. Nhm (+) ca cc im trn ng cong Elliptic


Xt ng cong Elliptic E c nh ngha bi phng trnh y 2 = x3 + Ax + B. Xt 3 im
nm trn ng cong E l P1 , P2 , P3 ln lt c cc ta l (x1 , y1 ), (x2 , y2 ), (x3 , y3 ).
25

Tp ch Epsilon, S 09, 06/2016


cc im trn ng cong Elliptic to thnh nhm (+), im v cng () s c thm
vo ng cong, k hiu l O, im ny s nm trn cng v di cng ca trc y. Mt trong
nhng thuc tnh quan trng nht ca ng cong Elliptic l tn ti nhm cc im vi php
cng nm trn ng cong.
nh l 5.1. Php cng vi cc im P, P1 , P2 , P3 trn ng cong E tha mn cc tnh cht
ca nhm:
1. (Giao hon): P1 + P2 = P2 + P1 ;
2. (im n v): P + = P ;
3. (im nghch o): Tn ti P 0 ca P sao cho P + P 0 = ;
4. (Kt hp): (P1 + P2 ) + P3 = P1 + (P2 + P3 ).
Chng minh. (1. Tnh cht giao hon) ca php cng 2 im P1 , P2 l hin nhin t cng thc
tnh ta ca im tng, cc gi tr c giao hon th gi tr tnh bi cng thc ny cng khng
thay i, hoc v mt hnh hc ng thng i qua P1 , P2 d c xut pht t P1 hay P2 th u
nh nhau v cng ct ng cong E ti mt im chung duy nht.
(2. im n v) khng cn phi chng minh v n xut pht t nh ngha. C th l gii r hn
v im () v cch nh ngha php cng trn E (theo quan im ca ngi vit) nh sau: Khi
ng cong E khng suy bin, n s ct mt ng thng c nh ngha bi phng trnh
(3.2) 3 im, thc vy theo cc php bin i mc 3.1 (trang 20) phng trnh (3.9) r(x) s
c 3 nghim phn bit. Mt khc E i xng qua trc x do phng trnh (2.1) c thnh phn y 2
nn lun tn ti hai gi tr y, y tha mn (2.1), cng do tnh i xng ny nn ng cong E s
ct cc ng thng song song vi vi trc y 2 im, v nu ct thm 1 im na th s phi
ct thnh 4 im do tnh i xng, iu ny l mu thun v phng trnh bc 3 ch c ti a 3
nghim. trng hp ny nu cng 2 im nm trn ng song song trc y s khng tm c
im th 3 do vy P1 + P2 s khng tn ti. Chnh v nhm cc im trn E c tnh ng bt
buc chng ta phi nh ngha thm im coi nh l im th 3 nm trn ng cong E, v
n s nm v cc 2 u trc y.
Tip theo, c th l gii (ca ngi vit6 ) v php nh ngha php cng 2 im trn E nh sau.
tha mn tnh cht tn ti im n v theo nh ngha v nhm G vi mi gi tr a G tn
ti e G sao cho
ae=ea=a
(5.1)
Xt im P trn ng cong E, khi cn tnh P + , d thy im ny chc chn phi nm
trn cng ng thng song song trc y v nu khng s ct E 2 im na cng vi to
thnh 4 im v iu ny l phi l. Nu nm trn ng song song y th n s ct E im
i xng qua trc x, nu coi im ct ny l tng P + th nh vy s tn ti P + = P 0 v
iu kin (5.1) s khng c tha mn. V vy s phi nh ngha im tng khng phi l giao
6

Khi bt u nghin cu v ng cong Elliptic, lun c 2 cu hi m ngi vit khng th tm thy trong nhiu
ti liu k c nhng cun kinh in v Elliptic:
1. Ti sao phi chn lm im trung ha.
2. Ti sao P1 + P2 = P3 m P3 khng nm trn ng thng i qua P1 , P2 m phi l im i xng ca giao
im qua trc x.
Trong chuyn ny c 2 cu hi u c ngi vit l gii mt cch r rng v y .

26

Tp ch Epsilon, S 09, 06/2016


trc tip vi E m phi ly l im i xng i xng ca im i xng s quay li chnh P
v ta c P + = P .
(3.im nghch o) Cng t nhn xt rng lun tn ti 2 im P, P 0 nm trn cng ng thng
song song vi trc y v s ct ng cong E im , coi 2 im ny l nghch o ca nhau
v s lun c: P + P 0 = .
(4. Tnh cht kt hp) Chng minh (P1 + P2 ) + P3 = P1 + (P2 + P3 ) khc hn vi 3 iu kin
khc v nhm, v n c bit phc tp. C 2 cch chng minh iu ny l dng phng php
hnh hc hoc i s. C th tham kho chng minh bng hnh hc qua cc ti liu [3], [17] v
[4], tuy nhin chng minh bng phng php hnh hc tng i kh hiu vi mt s nh l
trong khng gian x nh. Di y l mt s im c s dng chng minh bng phng
php i s. Trc tin tnh cc im tng sau:

P12
P32
P123
P321

= P 1 + P2
= P 3 + P2
= (P1 + P2 ) + P3 = P12 + P3
= (P3 + P2 ) + P1 = P32 + P1

T cc cng thc tnh gi tr ta (x, y) ca im tng, c th d dng bin i tnh im:


(y2 y1 )
(x2 x1 )
(y2 y3 )
=
(x2 x3 )

12 =

(5.2)

32

(5.3)

(y2 y1 )2
x1 x2
(x2 x1 )2
= 12 (x1 x12 ) y1 = 312 + (2x1 + x2 )12 y1
(y2 y1 )3
(2x1 + x2 )(y2 y1 )
=
+
y1
3
(x2 x1 )
x 2 x1
(y2 y3 )2
= 232 x3 x2 =
x3 x2
(x2 x3 )2
= 32 (x3 x32 ) y3 = 332 + (2x3 + x2 )32 y3
(y2 y3 )3
(2x3 + x2 )(y2 y3 )
=
+
y3
3
(x2 x3 )
x 2 x3
(y12 y3 )
=
(x12 x3 )
(y32 y1 )
=
(x32 x1 )

x12 = 212 x1 x2 =

(5.4)

y12

(5.5)

x32
y32

123
321

(5.6)
(5.7)

(5.8)
(5.9)

Cui cng cch tnh ta ca cc im P123 , P321 theo ta 3 im P1 (x1 , y1 ), P2 (x2 , y2 ), P3 (x3 , y3 )
27

Tp ch Epsilon, S 09, 06/2016


c biu din di cc dng cng thc t (5.10) n (5.13).

x123 =

(y2 y1 )3
(2x1 + x2 )(y2 y1 )

y1 y3
+
3
(x2 x1 )
x2 x1

2
(y2 y1 )2
x1 x2 x3
(x2 x1 )2

2

(y2 y1 )2
+ x1 + x2 x3
(x2 x1 )2
(5.10)

3

(y2 y1 )3
(2x1 + x2 )(y2 y1 )
y1 y3
+
3
(x2 x1 )
x2 x1
(5.11)
y123 =

3
(y2 y1 )2
x1 x2 x3
(x2 x1 )2



2(y2 y1 )2
(y2 y1 )3
(2x1 + x2 )(y2 y1 )
y1 y3
2x1 2x2 + x3

+
(x2 x1 )2
(x2 x1 )3
x2 x1
+
(y2 y1 )2
x1 x2 x3
(x2 x1 )2
(2x1 + x2 )(y2 y1 )
(y2 y1 )3

+ y1
+
3
(x2 x1 )
x 2 x1



x321 =

(2x3 + x2 )(y2 y3 )
(y2 y3 )3
+

y3 y1
3
(x2 x3 )
x2 x3

2
(y2 y3 )2
x3 x2 x1
(x2 x3 )2

2

(y2 y3 )2
+ x3 + x2 x1
(x2 x3 )2
(5.12)

3

(y2 y3 )3
(2x3 + x2 )(y2 y3 )
+
y3 y1
3
(x2 x3 )
x2 x3
(5.13)
y321 =
3

(y2 y3 )2
x3 x2 x1
(x2 x3 )2



2(y2 y3 )2
(y2 y3 )3
(2x3 + x2 )(y2 y3 )
2x3 2x2 + x1

+
y3 y1
(x2 x3 )2
(x2 x3 )3
x2 x3
+
(y2 y3 )2
x3 x2 x1
(x2 x3 )2
(y2 y3 )3
(2x3 + x2 )(y2 y3 )
+

+ y3
3
(x2 x3 )
x 2 x3


Cn phi chng minh rng P123 = P321 iu ny c ngha cn phi chng minh:
dx = x123 x321 = 0
dy = y123 y321 = 0

(5.14)
(5.15)

Trin khai v tri ca (5.14), s c mt phn s m t s v mu s bao gm tng cng 1446


thnh phn, tng t v tri ca (5.15) c tt c 10081 thnh phn c dng nk xi11 xi22 xi33 y1j1 y2j2 y3j3 ,
trong s m i1 , i2 , i3 , j1 , j2 , j3 nm trong khong [0..12] v nk l h s ca thnh phn biu
28

Tp ch Epsilon, S 09, 06/2016


thc trn. C th kim tra tnh ng n ca (5.14) v (5.15) bng phn mm Maple vi cc rng
buc sau:
y12 = x31 + Ax1 + B
y22 = x32 + Ax2 + B
y32 = x33 + Ax3 + B

(5.16)
(5.17)
(5.18)

6. ng cong Elliptic trn trng hu hn Fq


6.1. Trng hu hn Fq
Cc ng dng v mt m ca ng cong Elliptic a s ch s dng cc ng cong trn trng
hu hn.
Xt Fq l mt trng hu hn (hu hn s phn t s nguyn dng):
Fq = {0, 1, 2 . . . , q 1}
q l mt s nguyn t hoc c dng q = pm vi p l mt s nguyn t v m l mt s nguyn
dng. Khi ny p c gi l c s char(q) = p v m l bc m rng ca Fq .
Trong thc t v c bit trong cc thit b phn cng [18], ngi ta thng s dng trng hu
hn F2m . Khi php cng trong trng ny n gin ch l php ton XOR (Exclusive OR).
Nhiu ti liu cho thy lm vic vi F2m hiu qu hn 40% so vi lm vic vi trng Fq . Nhm
thc hin ti cp Nh nc KC01.18 do ngi vit lm h nhim ti ci t ton b cc
php ton v ng cong Elliptic trn trng F2m cho Chip Spartan 6 ca Xilinx cho bi ton
xc thc v trao i kha phin trong thit b VPN IPSec.
Trng F2m thng c biu din di dng t hp tuyn tnh ca cc vector gm m phn t
{0 , 1 , . . . , m1 }, mi phn t F2m u c th c bin din di dng:
= a0 0 + a0 0 + . . . + am1 m1 ,

ai {0, 1}

C nhiu phng php xy dng c s ca F2m : a thc c s v c s chun tc. Cc thut


ton thc hin cc php ton trn EC c th tm thy trong [19].

6.1.1. a thc c s
Xt a thc f (x) = xm + fm1 xm1 + . . . + f2 x2 + f1 x + f0 (vi fi F2 , i = 0, . . . , m 1)
l mt a thc bt kh quy bc m trn trng F2 , ngha l khng th phn tch f (x) thnh cc a
thc tha s khc c bc nh hn m. f (x) gi l a thc rt gn. Trng hu hn F2m s l tp
tt c cc a thc trn F2 c bc nh hn hoc bng m.
F2m = {am1 xm1 + . . . + a2 x2 + a1 x + a0 : ai {0, 1}}
Cc phn t (am1 xm1 + . . . + a2 x2 + a1 x + a0 ) thng c biu din di dng chui bit
(am1 . . . a1 a0 ) c di l m.
Cc php ton trong trng F2m :
29

Tp ch Epsilon, S 09, 06/2016


Php cng:
(cm1 . . . c1 c0 ) = (am1 . . . a1 a0 ) + (bm1 . . . b1 b0 ),

c i = ai b i .

Php nhn:
(rm1 . . . r1 r0 ) = (am1 . . . a1 a0 ).(bm1 . . . b1 b0 )
Trong (rm1 xm1 + . . . + r1 x + r0 ) = (am1 xm1 + . . . + a1 x + a0 ) (bm1 xm1 +
. . . + b1 x + b0 ) mod f (x).
a thc rt gn f (x) thng c dng sau:
Trinomial basis (TPB):
f (x) = xm + xk + 1, 1 6 k 6 m 1.
Pentanomial basis (PPB):
f (x) = xm + xk3 + xk2 + xk1 + 1,

1 6 k1 < k2 < k3 6 m 1.

6.1.2. C s chun tc
2

m1

c s c dng {, 2 , . . . , 2 } vi F2m , khi mi phn t a F2m u c


F2m s dng
Pm1
i
dng a = i=0 ai 2 , ai {0, 1} v cng c biu din di dng chui bit (a0 a1 . . . am1 )
c di l m. Vi c s ny php bnh phng s thc hin rt n gin ch bng cch quay bit.
Cc php ton trong trng F2m :
Php cng:
(c0 c1 . . . cm1 ) = (a0 a1 . . . am1 ) + (b0 b1 . . . bm1 ),

ci = (ai + bi ) mod 2.

Php bnh phng:


2

a =

m1
X
i=0

!2
ai

2i

m1
X

ai

2i+1

m1
X

ai1 2 = (am1 a0 a1 . . . am2 )

i=0

i=0

Php nhn:
Xt p = T m + 1 v u Fp l phn t bc T , nh ngha chui F (1), F (2), . . . , F (p 1)
ta s c:
F (2i uj mod p) = i, 0 6 i 6 m 1, 0 6 j 6 T 1.
(c0 c1 . . . cm1 ) = (a0 a1 . . . am1 ).(b0 b1 . . . bm1 )
Pp=2

Pk=1 aF (k+1)+l bF (pk)+l , nu T chn.


m/2
cl =
k=1 (ak+l1 bm/2+k+l1 + am/2+k+l1 bk+l1 )

Pp=2
+ k=1 aF (k+1)+l aF (k+1)+l , nu T l.

6.2. Tng s im ca ng cong Elliptic trn trng hu hn Fq


E l ng cong Elliptic trn trng Fq , bi v cp (x, y) vi x, y Fq l hu hn do nhm
E(Fq) cng s l nhm hu hn. Cc gi tr x, y l cc s nguyn, d dng nhn thy khng

phi vi mi gi tr x u tm c gi tr nguyn y bi v khng phi bao gi y cng l mt


s nguyn dng. Cu hi t ra l s im ca ca ng cong Elliptic trn trng Fq l bao
nhiu? Xc nh s im trn ng cong E nhm xc nh khng gian kha ca h mt.
Sau y l phn trnh by v vic tnh tng s im ca ng cong Elliptic trn trng hu hn
Fq .
30

Tp ch Epsilon, S 09, 06/2016



B 1. M v N l hai ma trn 2 2. M =




m11 m12
n11 n12
,N =
, vi cc s nguyn
m21 m22
n21 n22

a, b ta c:
det(aM + bN ) = a2 det(M ) + b2 det(N ) + ab(det(M + N ) det(M ) det(N ))

(6.1)

Chng minh. Theo nh ngha v nh thc ta c:


det(M ) = m11 m22 m21 m12 , det(N ) = n11 n22 n21 n12


am11 + bn11 am12 + bn12
det(aM + bN ) = det
am21 + bn21 am22 + bn22
= (am11 + bn11)(am22 + bn22 ) (am21 + bn21 )(am12 + bn12 )
= a2 (m11 m22 m21 m12 ) + b2 (n11 n22 n21 n12 )+
+ ab(m11 n22 + n11 m22 m21 n12 n21 m12 )
= a2 det(M ) + b2 det(N ) + ab(m11 n22 + n11 m22 m21 n12 n21 m12 )
(6.2)
Khi a = b = 1 p dng cng thc trn ta c:
det(M + N ) = det(M ) + det(N ) + (m11 n22 + n11 m22 m21 n12 n21 m12 )

(6.3)

Nhn c 2 v (6.3) vi ab sau tr vo (6.2) s c kt qu (6.1) l iu cn phi chng


minh.
nh ngha 6.1. im n-xon (Torsion): Cho ng cong Elliptic E c nh ngha trn trng
K, cho n l s nguyn dng, tp cc im Torsion E[n] l tp cc im trn ng cong c
tnh cht nh sau:


(6.4)
E[n] = P E(K) | nP =
K l ng i s ca K.
nh ngha 6.2. Divisor: Cho ng cong Elliptic E c nh ngha trn trng K, vi mi
im P E(K) nh ngha mt k hiu hnh thc (formal symbol) [P ], khi divisor D s l:
X
D=
aj [Pj ], aj Z
j

f l mt hm trn E m khc 0, khi divisor ca f s l:


X
div(f ) =
ordP (f )[P ] Div(E)
P E(K)

f = urP g, vi r Z v g(P ) 6= 0, , u(P ) = 0, nh ngha bc ca f ti P l:


ordP (f ) = r.
nh ngha 6.3. Cp Weil: l mt nh x t 2 im trong nhm cc im Torsion thnh gi tr
bc th n ca n v:
n = {x K | xn = 1}

en : E[n] E[n] n ,
31

Tp ch Epsilon, S 09, 06/2016


B 2. Vi mi t ng cu bt kh tch trn E, v vi mi S, S1 , S2 , T, T1 , T2 E[n] ta
c:
en ((S), (T )) = en (S, T )deg()
en (T, T ) = 1
en (S1 + S2 , T ) = en (S1 , T )en (S2 , T )
en (S, T1 + T2 ) = en (S, T1 )en (S, T2 )
Chng minh c th xem trong [3].
Gi thit {T1 , T2 } l c s ca E[n], mi phn t trong E[n] u c th biu din di dng t
hp tuyn tnh m1 T1 + m2 T2 . l mt t ng cu trong E[n], n l mt s nguyn khng chia
ht bi char(K). Tn ti cc s a, b, c, d Z sao cho:
(T1 ) = aT1 + cT2 ,

(T2 ) = bT1 + dT2

Do mi t ng cu u c th c biu din bi ma trn 2 2:




a b
n =
c d

B 3. l mt t ng cu trong E[n], n l mt s nguyn khng chia ht bi char(K) khi


det(n ) deg() mod n.
Chng minh. t = en (T1 , T2 ), theo b 2 ta c:
deg() = en ((T1 ), (T2 )) = en (aT1 + cT2 , bT1 + dT2 )
= en (T1 , T1 )ab en (T1 , T2 )ad en (T2 , T1 )cb en (T2 , T2 )cd
= adbc = det(n )

Nu , l 2 t ng cu trn E, v a, b l cc s nguyn th t ng cu a + b c nh
ngha nh sau:
(a + b)(P ) = a(P ) + b(P )

B 4. deg(a + b) = a2 deg + b2 deg + ab(deg( + ) deg deg )


Chng minh. Biu din cc t ng cu , bng cc ma trn n , n (vi mt s c s trong
E[n]), theo a + b s c biu din bng an + bn . p dng cng thc (6.1) ta c:
det(an + bn ) = a2 det(n ) + b2 det(n ) + ab(det(n + n ) det(n ) det(n ))
Theo b 3 chng ta s c:
deg(a + b) = a2 deg() + b2 deg() + ab(deg( + ) deg() deg())

32

Tp ch Epsilon, S 09, 06/2016


nh l 6.4. (Hasse) Nu E l ng cong Elliptic trn trng Fq , v #E(Fq) l tng s im
trn ng cong th:

q + 1 2 q 6 #E(Fq) 6 q + 1 + 2 q.

(6.5)

Chng minh. Trc tin xt nh x Probenius c nh ngha nh sau:


q : Fq Fq ,
x 7 xq
C th vit mt cch khc:
q (x, y) = (xq , y q ),

q () =

Khi thay cc gi tr xq , y q vo phng trnh (2.1) d thy (x, y) cng nm trn ng cong E.
nh x q l mt t ng cu v c th biu din bng hm a thc hu t c bc l q. o hm
ca xq l qxq1 s bng 0 bi v q = 0 trong trng Fq . Do o hm bng 0 nn q l kh tch
(separable).
Bi v q l t ng cu trong E do 2q = q q cng l t ng cu v nq cng l t ng
cu trong E. Php nhn vi 1 cng l t ng cu do tng nq 1 l ng cu trong E.
q l kh tch (separable) nhng q 1 s l bt kh tch do bc ca n s bng s phn t
ca hch q 1 c ngha l s im trn ng cong E s l:
#E(Fq ) = deg(q 1)
Vi cc s nguyn r, s, p dng b 4 ta c:
deg(rq s) = r2 deg(q ) + s2 deg(1) + rs(deg(q 1) deg(q ) deg(1))
Bi v deg(1) = 1 v deg(q ) = q nn:
deg(rq s) = r2 q + s2 + rs(deg(q 1) q 1))
t a = (deg(q 1) q 1) = q + 1 #E(Fq ), bi v deg(rq s) > 0 suy ra
r2 q + s2 + rsa > 0 hay vi mi r, s ta c:
q

 r 2

r

+1>0
s
s

Do = a2 4q 6 0 hay l |a| 6 2 q cng c ngha l |q + 1 #E(Fq )| 6 2 q v l


iu phi chng minh.
Tham kho thm v cch tnh s im trn ng cong E c th xem trong [20].

Ti liu tham kho


[1] J. W. Bos, J. A. Halderman, N. Heninger, J. Moore, M. Naehrig, and E. Wustrow, Elliptic
Curve Cryptography in Practice, Financial Cryptography and Data Security, vol. 8437,
pp. 157175, 2014.
33

Tp ch Epsilon, S 09, 06/2016


[2] J. H. Silverman and J. T. Tate, Rational Points on Elliptic Curves - Second Edition.
Springer, 2015.
[3] L. C. Washington, Elliptic Curves Number Theory and Cryptography, Second Edition.
CRC Press, 2008.
[4] J. W. S. Cassels, Lectures on Elliptic Curves.
[5] S. Lang, Elliptic Curves Diophantine Analysis.

University of Cambridge, 1991.


Springer, 1978.

[6] C. Kenig, A. Ranicki, and M. Rockner, Elliptic Curves A Computational Approach. Walter
de Gruyter GmbH & Co., 2003.
[7] H. Cohen and G. Frey, Handbook of Elliptic and Hyperelliptic Curve Cryptography.
Chapman Hall/CRC, 2006.
[8] J. H. Silverman, The Arithmetic of Elliptic Curves.

Springer, 2009.

[9] L. Berger, G. Bockle, L. D. M. Dimitrov, T. Dokchitser, and J. Voight, Elliptic curves,


Hilbert modular forms and Galois deformations. Birkhauser, 2013.
[10] I. F. Blake, G. Seroussi, and N. P. Smart, Advances in Elliptic Curve Cryptography.
Cambridge University Press, 2005.
[11] I. Connell, Elliptic Curve Handbook.

McGill University, 1999.

[12] T. H. Otway, Elliptic Hyperbolic Partial Differential Equations.

Springer, 2015.

[13] Dang Minh Tuan, Che tao thiet bi VPN IPSec bang phan cung dau tien o Vietnam, Tap
chi CNTT & TT, no. 2, pp. 4145, 2014.
[14] H. Lenstra., Factoring Integers with Elliptic Curves, The Annals of Matematics, vol. 126,
no. 3, pp. 649673, 1987.
[15] V. S. Miller, Use of elliptic curves in cryptography, CRYPTO 85, pp. 417428, 1985.
[16] N. Koblitz, Elliptic curve cryptosystem, Math.Comp, vol. 48, no. 16, pp. 203209, 1987.
[17] A. Enge, Elliptic Curves and Their Applications to Cryptography.
Publishers, 2001.

Kluwer Academic

[18] D. Hankerson, J. L. Hernandez, and A. Menezes, Software Implementation of Elliptic


Curve Cryptography over Binary Fields, CHES2000, vol. 1965, pp. 243267, 2000.
[19] D. Hankerson, A. Menezes, and S. Vanstone, Guide to Elliptic Curve Cryptography.
Springer-Verlag, 2004.
[20] R. Schoof, Elliptic Curves Over Finite Fields and the Computation of Square Roots,
Matematics of Computation, pp. 483495, 1985.
[21] I. Blake, G. Seroussi, and N. Smart, Elliptic Curves in Cryptography.
sity Press, 1999.

Cambridge Univer-

[22] H. Cohen, A Course in Computational Algebraic Number Theory. Springer-Verlag, 1993.


34

Tp ch Epsilon, S 09, 06/2016


[23] J. M. Pollard, Monte Carlo Methods for Index Computations (mod p), Mathematics of
Computation, vol. 32, no. 143, pp. 918924, 1978.
[24] S. C. Pohlig and M. E. Hellman, An Improved Algorithm for Computing Logarithms over
GF(p) and its Cryptographic Significance, IEEE Transactions on Information Theory,
vol. 24, pp. 106110, 1978.
[25] A. J. Menezes, T. Okamoto, and S. A. Vanstone, Reducing elliptic curve logarithms to
logarithms in a finite field, IEEE Trans. Inform. Theory, vol. 39, no. 5, pp. 16391646,
1993.
[26] C. Research, Standards For Efficient Cryptography, SEC 1: Elliptic Curve Cryptography.
Certicom Corp, 2000.
[27] L. Gao, S. Shrivastava, and G. E. Sobelman, Elliptic Curve Scalar Multiplier Design Using
FPGAs, CHES99, vol. 1717, pp. 257268, 1999.
[28] L. Laurie, M. Alfred, Q. Minghua, S. Jerry, and V. Scott, An Efficient Protocol for
Authenticated Key Agreement, Designs Codes and Cryptography, vol. 28, no. 2, 1998.
[29] D. Johnson, A. Menezes, and S. Vanstone, The Elliptic Curve Digital Signature Algorithm
(ECDSA), 2001.
[30] T. E. Gamal, A Public Key Cryptosystem and a Signature Scheme Based on Discrete
Logarithms, CRYPTO 84, vol. 196, pp. 1018, 1985.
[31] NIST, Digital Signature Standard (DSS) FIPS 186-4.
Technology, 2013.

National Institute of Standards and

[32] J. Massey and J. Omura, Method and apparatus for maintaining the privacy of digital
messages conveyed by public transmission, Jan. 28 1986, US Patent 4,567,600. [Online].
Available: https://www.google.com/patents/US4567600

35

Tp ch Epsilon, S 09, 06/2016

36

NGHCH L TRAI GI
ng Nguyn c Tin
(i hc Trento, Italia)
LI GII THIU
Tip ni cc chuyn mc Ton hc Gii tr nhng s Epsilon trc, s ny
chng ti gii thiu vi c gi mt nghch l ni ting trong xc sut: Nghch l
trai gi.
Bi ton ny, li mt ln na nh rt nhiu bi ton khc trong chuyn mc
Ton hc Gii tr, xut pht t cy i th Martin Gardner1 vi tn gi l bi ton hai
ngi con (Two Children Problem) ng t Scientific American vo nm 1954:
"Mt ngi c hai con, bit rng c t nht mt con trai, hi kh nng c hai u l
con trai l bao nhiu?"

1. Nghch l trai gi v bi ton ngy th ba


Chng ta hy bt u vi vic phn tch th li gii cho bi ton v hai ngi con ca Martin.
n gin, hy cng thng nht vi gi thit l gii tnh ca mi ngi con l c lp (ngha l
gii tnh ngi em khng ph thuc vo gii tnh ca anh hoc ch ngi ) v kh nng mi
ngi con l nam hay n l bng nhau.
Trc quan v d thy, v t nht mt ngi l trai, nn ngi cn li s hoc trai hoc gi vi kh
nng bng nhau (theo gi thit). Kh nng c 2 u l con trai do vy l 1/2.
Vy u c g l mu thun hay nghch l! Nhng dng li mt cht, chng ta hy th phn tch
cc kh nng c th c ca 2 ngi con khi cha bit gii tnh ca h. C th c 4 trng hp
y theo th t ngi con th 1 v ngi con th 2 l: nam - nam, nam - n, n - nam v n - n,
kh nng xy ra nh nhau. V v ta bit t nht mt trong 2 ngi l nam, nn trng hp cui n
- n b loi ra, ch cn c 3 trng hp u v trong s ny ch c trng hp nam - nam l ng
vi yu cu ca cu hi. V vy, kh nng c 2 u l con trai l 1/3. Nghch l v l lun trn
ch ra kh nng l 1/2!
Nghch l ny thu ht c rt nhiu ngi, c bit l c thi im ngi ta chia ra lm hai
trng phi" i lp hn nhau l trng phi nh phn" v trng phi "tam phn", ng vi 2 l
lun nh gii thch trn. V sau hn 60 nm t khi ra i cho n nay, s mu thun gia hai
trng phi, hay c th nhn nhn nh mu thun gia trc quan v l lun i vi bi ton ny
vn cn xy ra vi rt nhiu ngi v cu tr li u l p n ng tht s khng phi l n
gin, v thot nhn, ai cng ... c l.
Trc khi i vo phn tch u l trng phi "chin thng" nghch l trai gi, chng ti gii
thiu tip vi c gi mt phin bn l ca bi ton ny, xut bi Gary Foshee vo nm 2010.
Bi ton nh sau:
1

Bn c c th xem li Epsilon s 3 v cuc i ca nh ton hc huyn thoi ny.

37

Tp ch Epsilon, S 09, 06/2016


"Mt gia nh c hai con, nu nh bit rng t nht mt c mt a l con trai v sinh vo ngy
th ba th kh nng c hai u l con trai l bao nhiu?"
Hn nhiu bn c s hi: "Thm vo ngy th ba th lin quan g?" V cu tr li hn s lm
ngc nhin cho bn v xc sut c hai u l con trai khng phi 1/2 hay 1/3 m l 13/27.

2. i tm li gii
Trc tin, chng ti h thng li hai bi ton nh sau:
Mt gia nh c hai ngi cn, hi kh nng c 2 u l con trai l bao nhiu, bit rng:
1. Xc sut ca nam v n l bng nhau v bng 1/2.
2. Gii tnh ca hai ngi khng ph thuc vo nhau.
3. Cho bit trc mt iu kin K.
V tng hp li cc trng hp khc nhau vo bng sau:
Trng hp
TH1
TH2
TH3
TH4

iu kin K
Khng c iu kin
Ngi con u l con trai
t nht mt trong hai l con trai
t nht mt trong hai l con trai v sinh vo ngy th ba

By gi, chng ta hy cng xt trng hp n gin trc TH1, khng gian mu y l


= {T T, T G, GT, GG} vi T l trai v G l gi. V do vy, d thy xc sut T T l 1/4. Vi
TH2, = {T T, T G} ng vi ngi u l con trai, v do vy, xc sut T T xy ra l 1/2. Vi
TH3, = {T T, T G, GT }, nn T T xy ra vi xc sut 1/3, chnh l bi ton u ca chng
ta. Vi TH4, li gii kh hn v thng tin ngy th 3 dng nh khng lin quan.
Trc tin, gi s mi ngi con u thuc v mt trong hai nhm no , m tn sut ca nhm
P l p v tn sut ca nhm Q l q, vi p + q = n vi n l tng kh nng c th c. V d TH4,
hai nhm l P = "sinh vo ngy th ba" v Q = "khng sinh vo ngy th ba" c p = 1 v
q = 6. Nh vy, khng gian mu c 16 trng hp ng vi tn sut xy ra nh sau:
Hng/Ct
H1
H2
H3
H4

C1
TP TP
[p2 ]
TQ TP
[pq]
GP TP
[p2 ]
GQ TP
[pq]

C2
TP TQ
[pq]
TQ TQ
[p2 ]
GP TQ
[pq]
GQ TQ
[q 2 ]

C3
TP GP
[p2 ]
TQ GP
[pq]
GP GP
[p2 ]
GQ GP
[pq]

C4
TP GQ
[pq]
TQ GQ
[q 2 ]
GP GQ
[pq]
GQ GQ
[q 2 ]

Qua bng trn, ta thy tng tt c cc kh nng xy ra l n4 v ng vi mi "khi" 2 2, s kh


nng xy ra l n2 . Vi bi ton ca Martin (TH3), ta thy p n chnh l tng s kh nng ca
38

Tp ch Epsilon, S 09, 06/2016


khi 2 2 trn tri (l khi ton T ) chia cho ton b kh nng ca bng, sau khi loi b
khi 2 2 gc di phi (tc l khi ton G) nn xc sut xy ra l 3nn2 = 13 .
Quay li bi ton ngy th ba, vi rng buc kh nng P = "sinh vo ngy th ba" phi xy ra,
khng gian mu ca bi ton s l ton b cc hng H1 v ct C1 ca bng, ngha l ton
b cc c cha Tp . V yu cu ca bi ton chnh l cc (H1, C1), (H1, C2) v (H2, C1). Xc
sut (chng ti tm k hiu l ), do vy s l:
2
p2 + pq + pq
p2 + 2pq
p + 2q
2n p
= 2
=
=
=
=
2
2
2
p + pq + p + pq + pq + p + pq
3p + 4pq
3p + 4q
4n p
4

p
n
p
n

Khi p/n tin dn ti 0, s bng 1/2 v khi p/n tin dn n 1, s bng 1/3. Vi bi ton ngy
th ba (TH4), p = 1 v n = 7, ta c xc sut bng = 21/7
= 13/27. V nh vy, yu t "ngy
41/7
th ba", tht s c nh hng n kt qu cui cng!
Chng ta hy th cng xt iu kin sau: "c t nht mt ngi l con trai v sinh vo ngy 13
thng 6 v ri vo th ba, xc sut c hai u l con trai l bao nhiu?" B qua nm nhun,
0.49995.
chng ta c = 21/(7365)
41/(7365)

3. Cui cng th y c phi l nghch l?


Nh vy, chng ta cng nhau gii quyt v c kt qu ca cu hi: "Mt gia nh c hai ngi
con, bit rng c t nht mt ngi l con trai, hi kh nng c 2 ngi u l con trai l bao
nhiu" vi p s l 1/3. Tuy nhin, khi bi ton va ra i, vn c nhiu tranh ci, v chng ta
hy th cng nhau kho st cc phin bn khc nhau ca bi ton vi cng cu hi "xc sut
c hai u l con trai l bao nhiu":
"Trong tt c cc gia nh c hai con, m trong t nht mt ngi l con trai, mt gia nh
c chn ngu nhin hi." p n trong cu hi ny l 1/3, cng chnh l cu hi gc ca
Martin.
"Trong tt c cc gia nh c hai con, mt ngi con trong s c chn ngu nhin, v ngi
l con trai." p n lc ny li l 1/2.
Ni cch khc, ngoi vn l mt kt qu khc vi trc quan th cch pht biu bi ton cng
rt d dn n nhng bi ri gy tranh ci.
Quay li bi ton ngy th ba, vi yu cu "c t nht mt ngi l con trai v sinh vo th ba"
th kh nng c 2 u l con trai l 13/27. Tuy nhin, cu hi gc ca Foshee li l: "Trong ton
b cc gia nh c hai con m t nht c mt ngi l con trai v sinh vo ngy th ba, hy cho
bit t l cc gia nh m c hai ngi con u l con trai." V p n trong cu hi ny, theo
chnh tc gi, l 1/2.

39

Tp ch Epsilon, S 09, 06/2016

4. Li kt
Chuyn mc k ny ngn hn cc k trc do theo ch ca ngi vit, vn d cc nghch l
gy rt nhiu bi ri cho ngi c, nn nu dng qu nhiu bi ton s khin ngi c cng
kh tip cn hn, mt i bn cht ca ton hc gii tr. c c bi vit ny, chng ti tng
hp v s dng cc ngun tham kho chnh sau:
1. Wikipedia, Boy or Girl Paradox.
2. Gardner, Martin, 1954: The Second Scientific American Book of Mathematical Puzzles and
Diversions. Simon & Schuster.
3. Peter Lynch, The Two-Child Paradox: Dichotomy and Ambiguity, Irish Math. Soc. Bulletin 67
(2011), pp. 67-73.
Chng ti kt thc chuyn mc ti y v hn gp li cc bn trong s ti.

40

PHP BIN I FOURIER


C NGHA VT L G?
Job Bouwman
(Trn Nam Dng dch)
Hy tng tng bn g mt phm n dng cm, khi dy n s rung ln v
to ra mt sng m c dng hnh sin nh ta vn bit trong lng gic. By gi tng
tng ba phm c nhn cng mt lc, ln sng m ny s khng cn l hnh sin
na, m l mt ci g hn n v phc tp hn. Nhng n trong ln sng m
thanh ln xn ny ch l mt m hnh n gin: Sau tt c, hp m to ra ny ch l
ba phm nh cng lc, v v vy sng m thanh ln xn ch l kt qu ca s tng
hp ca ba nt (hay ba sng sin). V chnh l mt v d ca bin i Fourier.
Trong s ny, Epsilon tip tc gii thiu vi c gi phn bnh lun t bi vit ca
tc gi ca tc gi Job Bouwman c ng Epsilon s 7: Php bin i Fourier
c ngha vt l g?

1. Stephen Scholnik
Thc s ti s dng php bin i Fourier ... rt nhiu.
l mt cng c ton hc phn tch tn hiu, n khng c mt s tng t chnh xc v mt
vt l (IMHO). Nhng phi ni rng: S tng t tt nht m ti c th ngh ra l n cho ta bit
gi tr tn s trung bnh ca mt mu th ca, chng hn nh, mt bn ghi m. Nh vy nu nh
l bn ghi m ca mt bn nhc, ni chung n s cho chng ta bit nng lng m thanh trung
bnh (v pha) ca mi nt nhc trong ton bi. Nhng iu cng khng ng hn bi v mi
nt nhc c s ha m v php bin i Fourier khng th phn bit c s ha m gia nt La
v nt La trong mt qung tm cao hn.
Ngoi ra, nh ti ni, php bin i Fourier cho bn bit pha trung bnh ca mi tn s m
iu ny thng thng li khng c mt minh ha vt l no c (mc du bn c th nghe c
nhng bin i pha ng). T hn na l, mt nt nhc cng g mnh (tn cng sc bn hn v
ri xung), ph tn s ca n cng m. Nh vy c nhng tn s trong php bin i c nng
lng ng k nhng li khng tng ng vi mt nt nhc no c. Do tt c nhng l do , s
tng t hu nh l mt sai lm (thm vo sau ).
Ti thch cu tr li ca Lionel Chiron. ng ta a ra nhng v d ch thc v nhng d liu
quan st c th tng ng vi php bin i Fourier ca mt ci g c th. Ngoi ra cu tr li
ca ng ta lm ti nh rng mt bc nh ton k 3 chiu (th m bt c ai am m khoa hc u
bit v yu thch) l mt s tng t mc cao ca php bin i 2 chiu ca cnh m n xy
dng li.
41

Tp ch Epsilon, S 09, 06/2016

2. Konstantinos Konstantinides, K s in lm vic trn


lnh vc x l tn hiu v m ha video
Mt php bin i Fourier (FT) mt mnh n khng c ngha g c. l mt hm s. u ra
ca php bin i Fourier cho ta bin ca tn hiu trn min tn s. C khi lm vic vi min
thi gian tin hn, c khi lm vic vi min tn s tin hn. Chng hn nh, nu c ai bo vi
bn rng y l mt b lc thng thp, bn c th hiu rng b lc ny s lc b tt c nhng
tn s cao sau mt php lc tn, mc du biu din trong min thi gian (p ng xung) c th
kh phc tp.

Trong x l tn hiu, tch chp trong min thi gian tng ng vi php nhn trong min tn s.
iu ny gip ch rt nhiu khi bn phn tch p ng tn s ca mt h thng. Vy l php bin
i Fourier c th xem nh mt cng c chuyn i qua li gia min thi gian v min tn s.

3. Samuel O. Ronsin, PhD, Ton ng dng


Thm mt v ngha vt l, tht th v l php bin i Fourier (c th l di dng trc
giao) l cn bc hai ca ton t o chiu : f 7 (f ) = (x 7 f (x)) trong mt khng
gian L2 no , do vy F F = .
iu ny c ngha g? Vng, nu bn mun phn tch ton t o chiu mt hm no
thnh hai ton t ging nhau, php bin i Fourier chnh l ci bn mun tm. Du rng cn c
thm mt vi v d na ...

4. Norman Corbett, PhD, Ton ng dng


i vi cu tr li hin c: Php bin i Fourier c xy dng nh nhng tn s iu ha ...
bi nguyn ca mt vi tn s c bn. Mt trong nhng im tch ri vi m nhc l ch cc
nt nhc tun theo thang hm m (hy ngh v chic n piano ca bn). iu ny khng c
ngha l ph ca mi qu trnh vt l u b rng buc theo kiu nh vy.
Bn cnh , cc tnh cht ton hc ca php bin i Fourier m bo rng n c th x l d
liu m thanh, hnh nh, vt en trn mt tri, n tinh, ... v lm l ra nhng c im vt l c
bn.
Phi chng php bin i Fourier c mt nguyn nhn pht sinh c th? Vng! Joseph Fourier
khi u tt c khi nghin cu v s truyn nhit trong kh quyn. y phn tch tn hiu tn
s iu ha l ht sc tha ng ... (vng cho m hnh tuyn tnh).
Php bin i Fourier c ngha vt l g khng? Tt nhin l c. Hy xem xt nguyn nhn ban
u thc y s ra i ca n. Ni rng php bin i Fourier ch n gin l mt vn tru
tng cng ging nh ni Navier-Stokes khng c c s no trong thc ti.
42

Tp ch Epsilon, S 09, 06/2016

5. Singer Karthik, K s in x l tn hiu s v in t


s
Php bin i Fourier (FT) l mt m hnh/tru tng ha ton hc phn tch mt tn hiu thnh
nhng tn hiu c bn cng ging nh mt lng knh nhn vo nh sng trn v tch n ra thnh
nhng tn s nh sng khc nhau. Khng c s tng t no tt hn. Trc ht, mi tn hiu c
mt thnh phn tn s ch khng phi l nhng c im ca min thi gian. a s nhng ai
khng phi l k s gp kh khn vi khi nim tn s. Hiu khi nim ny trc v php bin
i Fourier s tr nn d hiu hn.
Ni tm li, php bin i Fourier cho php bn phn tch tn hiu trong mt min xc nh khc
gip ta hiu thu nhng thnh phn tn s c bn to thnh tn hiu m ta nghin cu. Nu nh
bn tht s mun phn tch ni dung ca mt tn hiu bn phi xem xt n trong min tn s.
th bin v tn s cho ta bit tn s no l ch o. T bn c th xc nh chng hn mt
tn s xc nh cha nhiu v thit k mt b lc lc nhiu i. Hoc gi bn mun lm suy
yu nhng tn s xc nh v khuch i nhng tn s cn li. N gip bn to hnh tn hiu nh
mong mun.

6. Jonathan Roberts
Php bin i Fourier l mt php bin i t min thi gian vo min tn s. Nu bn p dng
php bin i Fourier cho mt sng cosine bn s c mt on thng ng trn th. Mi sng
phc tun hon lin tc c th to thnh t tng ca nhng chui hnh sin trc giao. Bin i ca
nhng sng gin on pht sinh li gi l hin tng Gibbs. Vi hnh nh hu ch

Ngun: Steve on Image Processing, Frequency domain


43

Tp ch Epsilon, S 09, 06/2016

7. Henk Mulder
Php bin i Fourier o s tng quan ca mt tn hiu tun hon v mt chui nhng hm
iu ha hnh sin. y l cch m ton hc lm theo ng ngha en: tnh s tng quan ca mi
hm iu ha i vi tn hiu nhp vo.
iu ny c th c ngha l php bin i Fourier o xem tn hiu tun hon ging hm iu ha
ti mc no. i vi nhng hm iu ha ch o, bn c th nhn ra. Nhng nu nh c nhiu
hm iu ha th iu ny s kh khn hn. chnh l l do ti sao bn dng php bin i
Fourier c mt bc tranh chnh xc v s hin din ca tt c cc hm iu ha trong tn hiu
nhn vo.
Cn ni thm l tht th v khi ngi ch em ca php bin i Fourier, php bin i Laplace
thc hin chnh xc cch lm vi nhng hm m. N tnh tng quan ca hm m i vi
nhng thnh phn ca hm u vo v biu din hm ny di dng nhng hm m. V hm m
c o hm l chnh n, iu ny cho php bn x l d dng nhiu phng trnh vi phn.

8. Michael Kownacki
Lm vic trong mt x nghip lm giu uranium, ti thng xuyn dng php bin i Fourier
ly gi tr nh lng trong vic x l s c nhng my mc s s dng u ra ca gia
tc k. Mi vn hin ra nh mt tn s c bit, shaft alignment, s rung ca chong chng
turbine, ... Bng cch ly tch phn bnh phng ca php bin i Fourier trong mt vng tn s
c bit, ti c th tnh c nng lng ca mi vn c th.
Ti cng c th xc nh c chuyn ng ri lm gim hiu qu ca c qu trnh. Tt c nhng
th bn cn lm l lc nh mt ci van u vo thay i phn b p sut lm cho chuyn ng
ri bin mt. Mt khi s rung trong vng ri bin mt, bn bit rng van c b tr ng.
Mt vn nghim trng xy ra vi turbine l khi p sut khng u chung quanh chu vi ca n
(thng gi l t bin th cp). N lm cho chong chng turbine cht my mi khi chng quay,
to thnh s rung chong chng c th lm gy chong chng. i vi nhng my ln c tc
cao, iu ny c th khin cho ton b ci my n tung ng theo ngha en.

9. Mil Ford
Ti ngh rng mt v d trong ti ca ti, mt phn trong l xc nh cao ca thy triu
c th b ch cho cc bn.
Thy triu xut hin do lc hp dn ca mt tri, mt trng, ...
V mt trng quay chung quanh qu t v qu t quay chung quanh mt tri, chiu cao ca
thy triu c dng hm sin / cos
Ti dng mt phng php gi l Phn Tch Fourier Ri Rc to thnh trc Phn Tch Fourier
da trn nhng im d liu thay v hm.
V cn bn chng ta s so trng mt l nhng ng cong sin / cos, nh l php hi quy phi
tuyn tnh i vi cc im d liu v bin i n thnh tn s u ra ( th v ci bn nhn
c sau cng).
44

Tp ch Epsilon, S 09, 06/2016


Kt qu l, chng hn mt tn s na ngy, chng ta bit rng hiu ng thy triu l do s quay
ca tri t, bit tn s chng ta d dng tm ra bin Cn .
T bn c th vit thnh mt hm di chng hn nh C1 sin( ) + C2 sin( ) V hm
ny ra gip ta d on thy triu trong tng lai.
Tin hnh php bin i bn cn bn lm khi chuyn t ci thi gian sang khng gian tn s.

10. David Tung, mt nh doanh nghip


Php bin i Fourier kt hp hai khi nim vt l: Tun hon v c s ( l mt tn hiu tt c
th phn tch thnh cc thnh phn c lp v cc thnh phn ny c th kt hp li to thnh
nguyn tn hiu ban u).
Nhng php bin i Fourier to ra khi nim tn s, mt phn trong s hiu bit ca chng ta
v th gii vt l, nh vy c th cng ng khi ni rng tm tn s tng ng vi tin hiu chnh
l ngha vt l ca php bin i Fourier, nu chng ta qun i vn con-g-v-qu-trng.

11. Rod Schmidt


Trong tai trong c c tai cun xon li. Tng tng n khng xon na v tri di ra. N l mt
hnh nn di v hp. Khi m thanh vo tai, nhng tn s thp hn (bc sng di nht) to thnh
cng hng rung gn u ln m ca hnh nn. Nhng tn s cao (bc sng ngn nht) gy ra
nhng cng hng rung tng t gn nh ca hnh nn. Nh vy cc tn s c phn loi. Mi
tn s tng ng vi mt khong cch dc theo hnh nn.
i vi php bin i Fourier ngc ... hy hnh dung ra ng iu , ch c iu b phim c
chiu ngc li.

12. Lionel Chiron


Lin quan vi cu hi ca bn, php bin i Fourier c th xem nh mt hin tng t nhin..
chng hn n l trng hp ca tinh th hc khi nhng m quan st c l php bin i
Fourier ca cu trc tinh th di php ri X-quang, l trng hp ca MRI khi tn hiu ng
k (k-khng gian) l php bin i Fourier trc tip ca hnh nh m chng ta mun khm ph,
l trng hp ca Quang Hc Fourier: hin tng nhiu x sinh ra (trong mt chng mc gn
ng ph thuc vo khong cch) mt php bin i Fourier ca hnh nh trc mn chn.
iu ny l kh d v tt c nhng hin tng ni trn da trn mt s kin l c nhng hin
tng thin nhin c th c m hnh ha mt cch chnh xc nh mt tn s v mt bin ...
Tng qut hn C Hc Lng T s dng khi nim php quay trong khng gian tru tng v
can thip nhiu hn so vi vt l c in ... v do vy s xut hin t nhin ca php bin i
Fourier vn mang li mt cch nhn mi ca cng mt hin tng li cng t nhin hn na ...
Vi C Hc Lng T, khi nim tn s qu tht m rng sang c hc ... R rng n khng
ch gii hn trong quang hc v m thanh ...

45

Tp ch Epsilon, S 09, 06/2016

46

TP HP TR MT V NG DNG
Kiu nh Minh, Nguyn Quang Khi
(THPT chuyn Hng Vng, Ph Th)
Trong bi vit ny, cc tp hp c hiu l tp con ca R.

1. Tp hp tr mt
nh ngha 1. Cho hai tp hp A, X vi A X. Ta ni rng A tr mt trong X nu vi mi
x X v mi > 0 tn ti x A sao cho |x x| < . Mt cch tng ng, vi mi
x, y X, x < y tn ti x A sao cho x < x < y.
Ni mt cch nm na rng, tp A tr mt trong X nu mi phn t trong X u b kim sot bi
nhng phn t trong A vi khong cch b tu .
nh l 1.1. Tp hp A tr mt trong tp hp X khi v
 ch khi vi mi x X, tn ti dy (xn )
tho mn xn A, n N v lim xn = x A = X .
n+

Chng minh. Tht vy, nu x A th chn xn = x, n. Nu x


/ A, theo nh ngha ta c: Vi
mi x X v mi > 0, tn ti x A sao cho |x x| < . Mt khc d thy tn ti dy (n )
tho mn 0 < n < , n N sao cho mi n N, tn ti xn A tho mn |xn x| < n .
Suy ra
|xn x| = |xn x + x x| 6 |xn x| + |x x| < n + < 2,
do lim xn = x. Ngc li l hin nhin.
n+

Nhn xt. Nu A tr mt trong X th vi mi x X, tn ti hai dy (xn1 ) , (xn2 ) A tho


mn:
(i) xn1 < x < xn2 , n1 , n2 N.
(ii)

lim xn1 = lim xn2 = x.

n1 +

n2 +

nh l 1.2. Tp s hu t Q l tr mt trong tp s thc R.


Chng minh 1. Ta c nguyn l Archimet vi x, y R sao cho x > 0, y > 0 th lun tn ti
n N sao cho y 6 nx. Xt x, y R, x < y. t d = y x > 0. Theo nguyn l Archimet
tn ti n N sao cho nd > 1, suy ra n1 < d. Gi m = [nx] + 1 th m 1 6 nx < m suy ra
nx < m 6 nx + 1, suy ra tip x < m
6 x + n1 < x + d = y.
n
Chng minh 2. Ta cn ch ra a, b R, m
(m Z, n Z+ ) sao cho a < m
< b. Tht vy,
n
n
1
+
chn n Z sao cho on [na, nb] c di ln hn 1 hay nb na > 1 tc n > ba
. Khi
m
tn ti m (na, nb), suy ra na < m < nb suy ra a < n < b.
47

Tp ch Epsilon, S 09, 06/2016


Chng minh 3. Khng mt tng qut, ta c nh b v t ba = . Xt tp A = {x Q\x < b}.
Theo nh ngha v Supremum: > 0, x0 sao cho b < x0 < b. Nh vy ta ch cn chng
minh SupA = b. Gi SupA = s. Xt n N ta chng minh
s

1
1
6b6s+ .
n
n

Bt ng thc bn tri l hin nhin v x sao cho s


s Q th s + n1 A v s + n1 > s (v l). Nu s
/ Q th
w=

1
n

< x < b. Gi s s +

1
n

< b. Nu

1
[(n + 1) s]
+
Q,
n+1
n+1

v tho mn s < w < s + n1 . Do w < b hay w A (v l). Suy ra |b s| 6


lim |b s| = 0 tc b = s.

1
n

suy ra tip

Chng minh 4. K hiu [a] l phn nguyn ca a. Ta c


[a] 6 a < [a] + 1
Theo nh l gii hn kp ta c lim
n

[a]

a+ a
[10n x]+1
.
10n

[a] + 1
[a]
61<
(a > 0) .
a
a
= 1 = lim

[a]+1
.
a

Cho x l s thc dng, vi mi

x]
n N , t xn = [10
v x0 n =
Hai dy (xn ) v (x0 n ) l cc gi tr hu t gn ng
10n
di v gn ng trn ca x vi chnh xc 10n . D thy xn , x0 n Q v (xn ) tng cn (x0 n )
gim. Ngoi ra lim xn = x v lim x0 n = x. Nu x < 0 th hai dy s lp nh trn s tin ti
n+

n+

x. Bi vy hai dy s (xn ) v (x0 n ) s tin ti x.


Chng minh 5. Xt mt s thc x no . Ta chng minh rng > 0, r Q : |r x| < .
Tht vy, nu x Q th ta ch cn chn r = x. Ngc li, x
/ Q, xt biu din thp phn v hn
khng tun hon ca x, x = a0 a1 a2 an a . Ta thy > 0, n N : > 10n . Khi ,
chn r = a0 a1 a2 an+1 Q ta thy ngay |r x| < 10n < .
Ch . Tp s v t l tr mt trong tp s thc R. Tht vy, vi mi a, b Q, a < b. t
e = b a > 0, lun tm c
s t nhin n lnsao cho ne > 2. Khi n (b a) > 2, suy ra
nb > 2 + na. Chn q = 2 + na l s v t, do 2 l s v t v na l s hu t. Ta c

q
na < 2 + na < 2 + na < nb na < q < nb a < < b.
n
Vy tn ti m =

q
n

l s v t tho mn.

nh l 1.3. (Kronecker) Cho l mt s v t. K hiu {x} = x [x]. Khi tp hp


{{n} | n Z+ } l tr mt trong khong (0; 1).
Chng minh. Xt mt khong = (a; b) (0; 1), ta s chng minh n Z+ : {n} (a; b).
Tht vy, ta chia (0; 1) thnh mt s hu hn cc na khong 1 , 2 , . . . , n0 sao cho di
ca mi na khong u nh hn mt s thc dng m ta s la chn v sau. Khi , v ta c
th tm c v hn cc gi tr n, m sao cho n + m [0; 1] m li ch c hu hn cc khong
nn theo nguyn l Dirichlet th tn ti cc s nguyn dng n1 , n2 (n1 > n2 ) v m1 , m2 sao cho
cc s n1 + m1 v n2 + m2 thuc cng mt khong k no . Ta c
0 < (n1 n2 ) + (m1 m2 ) < .
48

Tp ch Epsilon, S 09, 06/2016


By gi ta chn tho mn <

ba
.
2

Ta c

ba
ba
>
> 2.
(n1 n2 ) + (m1 m2 )

Do s tn ti mt s nguyn k nm gia

a
(n1 n2 )+(m1 m2 )

b
,
(n1 n2 )+(m1 m2 )

tc l

a < k (n1 n2 ) + (m1 m2 ) < b.


By gi chn N = k (n1 n2 ) th s {N } (a; b) . Vy ta kt lun {{n} | n Z+ } tr mt
trong (0; 1). nh l c chng minh xong.
nh l 1.4. (Thc trin hm lin tc) Cho hai hm s f, g : X X, trong f lin tc,
g lin tc hoc n iu v A l tp tr mt trong X. Khi nu f (x) = g (x) , x A th
f (x) = g (x) , x X.
Chng minh. Ta xt hai trng hp:
a) g l hm lin tc: Theo nh l 1 th x X, (xn )n>0 A : lim xn = x. Ta c
n+

f (xn ) = g (xn ) , n N suy ra lim f (xn ) = lim g (xn ) f (x) = g (x).


n+

n+

b) g l hm n iu: Khng mt tng qut, gi s g l n iu tng. x X, xn1 , xn2 A


sao cho xn1 < x < xn2 v lim xn1 = lim xn2 = x. Ta c
n2 +

n2 +

f (xn1 ) = g (xn1 ) < g (x) < g (xn2 ) = f (xn2 ) .


Chuyn qua gii hn ta c f (x) = g (x).
Trong mi trng hp nh l u c chng minh.
Nhn xt. T nh l 2 v nh l 4 th ta c nghim ca phng trnh hm trn R khi bit hm
trn Q v tnh lin tc hoc n iu trn R. Tiu biu l lp phng trnh hm Cauchy.

2. Mt s v d minh ha
V dn1. Cho dy sothc dng (an ), tng v khng b chn. Chng minh rng tp hp
S1 = amn | m, n N tr mt trong R+ .
Chng minh. Nhn xt rng vi mi s > 0 v dy (an ) c tnh cht lim an = + th cng
n+

c lim an = +. Xt p, q R+ tho mn p > q. Khi y lim (p q) an = +. Do ,


n+

n+

tn ti n0 N sao cho
(p q) an0 > 2 pan0 > [pan0 1] > qan0 .
Chn m0 = [pan0 ] 1, ta c p >

m0
an0

> q. Vy S1 tr mt trong R+ .

Nhn xt. T v d ny suy ra cc tp

m
2n



| m, n N , amn |m, n N tr mt trong R+ .
49

Tp ch Epsilon, S 09, 06/2016


V d 2. Gi s N c
chia thnh hai tp con A v B, mi tp cha v hn phn t. t
S3 = ab | a A, b B . Chng minh rng tp S3 tr mt trong [0; +).
Chng minh. Do B c v hn phn t nn k N, > 0, b B sao cho b >
hn phn t nn

k
.
2

Do A c v

b (x + ) > b (x ) + k, x [0; +) a A : b (x + ) > a > b (x ) ,


suy ra
x+>


a

> x x
b

a
< .
b

Vy S3 tr mt trong [0; +) .
1


|
r

A
cng tr mt trong R+ .
r

V d 3. Cho (an )n>1 l dy s nguyn dng tho mn 0 < an+1 an < an , n N . t


o
n
S4 = aakl | k, l N , k > l . Chng minh rng tp S4 l tr mt trong (0; 1).
Nhn xt. Nu A tr mt trong R+ th B =

Chng minh. Ta chng minh nhn nh sau:


0 < x < y < 1, k, l N , k > l : x <

ak
< y.
al

Tht vy, t gi thit suy ra (an ) l dy s nguyn dng tng nn lim an = +. Suy ra rng
n+

x, y > 0, l N sao cho

a1
al

< x v

1
al

< y x. Xt cc phn t ca dy s hu hn

a1
a2
al1
al
<
< <
<
= 1.
al
al
al
al
Khi vi k = 1, 2, . . . , l 1, ta u c

ak
al
1
ak+1 ak

<
<
= < y x.
al
al
al
al
al
Hn na

a1
al

< x nn phi tn ti k sao cho x <

ak
al

< y.

V d 4. Cho dy s dng (an ) tng tho mn lim ann = k, k R+ v dy s dng (bn )


nn+
o

tng, khng b chn. Chng minh rng tp S5 = abm


|
m,
n

N
tr mt trong R+ .
n
an
n+ n

Chng minh. Xt p, q R+ , p > q. Do lim

= k nn chn > 0 sao cho

pq
p+q

k > . Ta

c, tn ti N sao cho
a
pq
2q
an
2p
n

k , n > N
k+<
<
k , n > N.
k <
n
p+q
p+q
n
p+q
t =

q
2q
k+
p+q

<

p
2q
k
p+q

= . Theo v d 1, tn ti v s b (m, n) sao cho <

n
bm

< . Ch

theo th d 1 ch c s tn ti hu hn, nhng s tn ti dn n s tn ti v hn v nu ta chn


< 1 < < n < th gia (i , j ) tn ti mt s, cho n + th tn ti v s s. Do
, c th chn n0 > N v m0 sao cho
<

n0
< ,
bm0
50

(2.1)

Tp ch Epsilon, S 09, 06/2016


m

2q
an
2p
k+< 0 <
k .
p+q
n0
p+q

Nhn v theo v (2.1) v (2.2) ta c q <

an0
bm0

(2.2)

< q. Vy S5 tr mt trong R+ .

Nhn xt. Ta thy rng:





1) Tp m! sin n1 |m, n N tr mt trong R+ .
an
n+ n

= + th bi ton khng cn ng na. Tht vy, chn an = 2n , bm = 2m

2) Nu lim

th S5 khng tr mt trong R+ .
an
n+ n
2k+1

3) Nu lim
n62

= 0 th bi ton khng cn ng na. Tht vy, chn an = 2k nu 22 6

1 v bm = 2m th S5 khng tr mt trong R+ .

V d 5. Chng minh rng tp S6 = {sin n | n N} tr mt trong [1; 1] .


Chng minh. Trc ht ta chng minh rng nu l mt s v t dng, th tp {n m | m, n N}
l tr mt trong R+ (c th xem v d 6) . lm iu , ly mt khong (a; b) , 0 < a < b ta s
ch ra rng khong ny cha t nht mt phn t ca tp cho. t = b a > 0, tn ti phn
t Rn sao cho
1
0 < Rn < 2 .
(2.3)
qn
Thc vy, ly mt s n l v ch rng (qn xn+1 + qn1 ) qn > qn2 . V lim qn = +, vi n
n+

1
qn

ln ta c < . T iu ny v (2.3) suy ra 0 < pn qn <


n0 N sao cho n0 (pn qn ) (a; b) .

1
qn

< vi n ln, do tn ti

Cho t [1; 1], tn ti s thc x sao cho t = sin x. T nhn xt trn suy ra tn ti cc dy cc
s nguyn dng {mn } v {kn } vi x = lim (mn 2kn ), S dng tnh lin tc ca hm
n+

sin x ta c

t = sin x = sin


lim (mn 2kn ) = lim sin mn .

n+

n+

Vy mi s thuc khong [1; 1] u l im gii hn ca tp S6 = {sin n | n N}.


V d 6. Cho u, v R+ . Chng minh rng tp hp {au + bv | a, b Z} tr mt trong R khi v
ch khi uv l mt s v t.



Chng minh. Ta
nn v au
+ b | a, b Z tr mt
v
 auc {au + bv | a, b Z} tr mt trong R 
trong R do v + b | a, b Z tr mt trong R suy ra a uv | a Z tr mt trong (0; 1)
v th uv l s v t.
Nhn xt. Vi u 6= 0, v 6= 0 th bi ton vn ng.
V d 7. Cho cc s thc dng a, b tho mn b [an] = a [bn] , n Z+ . Chng minh rng
a = b hoc c a v b u nguyn.
51

Tp ch Epsilon, S 09, 06/2016


Chng minh. Gi s a 6= b v c a, b u v t, ta c
b [an] = a [bn] ban b {an} = abn a {bn} b {an} = a {bn} .

(2.4)

Khng mt tng qut, gi s a > b, ta c


b
{bn}
b
=
, n Z+ > {bn} , n Z+ ,
a
{an}
a
iu ny v l do b l s v t nn {{bn} | n Z+ } l tr mt trong (0; 1). Do mt trong hai
s a, b l hu t. Gi s a Q, khi tn ti n0 Z+ sao cho an0 Z+ {an0 } = 0. Do
{bn0 } = 0 b Q. t a = pq , (p, q) = 1 v b = st , (s, t) = 1. Trong (2.4) cho n = q c
.
0 = a {bq} {bq} = 0 {bq} Z+ q .. t.
.
Tng t, ta c t .. q. Vy q = t, do (2.4) tng ng vi
 
 
p
s
n =p
n , n Z+ .
s
q
q
Vi q > 1, ta chn n 1s (mod q), suy ra
   
 
 
sn
1
1
p
1
p
=
= s
n =p
= .
q
q
q
q
q
q
t

pn = qk + r s

pn
q


=s

p
r
r
.
= s. p = s r p .. s.
q
q
q

.
Tng t c s .. p. Do p = s, v l do a 6= b, do vy q = 1 v a, b Z. Tm li hoc a = b
hoc a, b Z.
V d 8. Cho a, b l cc s thc dng tho mn [2an + b] chia ht cho 2 vi mi s nguyn
dng n. Chng minh rng a l s nguyn dng.
Chng minh. Nu a l s v t th ta c
.
.
[2an + b] = [2 {an} + b] + 2 [an] .. 2, n Z+ [2 {an} + b] .. 2, n Z+ .

(2.5)

Li do {{an} | n Z+ } l tr mt trong (0; 1). Suy ra tn ti s nguyn dng n m [b + 2 {an}]


khng chia ht cho 2. Tht vy:
Nu 2k + 1 > b > 2k th chn n sao cho 2k + 2 > 2 {an} + b > 2k + 1.
Nu 2k + 2 > b > 2k + 1 th chn n sao cho 2k + 2 > 2 {an} + b > 2k + 1. iu ny mu
thun vi (2.5) nn suy ra a phi l s hu t. Nu a khng nguyn th t a = pq , (p, q) = 1,
q Z+ , q > 1. Khi vit n = qt + r, 0 6 r 6 q 1 th
 


pr
.
[2an + b] = 2 pt +
+ b .. 2, n Z+ .
q
52

Tp ch Epsilon, S 09, 06/2016


.
.
Chn r = 0 th [2pt + b] .. 2 [b] .. 2. Do



2pr
.
+ {b} .. 2, t Z+ ,
[2an + b] = 2pt + [b] +
q
tng ng vi



2pr
.
+ {b} .. 2, t Z+ ,
q

hay l

  
pr
.
+ {b} .. 2, t Z+ .
2
q
n o
h n o
i
M 3 > 2 prq + {b} > 0 suy ra 2 prq + {b} {0; 2} . Ta xt hai trng hp sau:
h n o
i
Nu tn ti r0 6= 0 sao cho 2 prq0 + {b} = 0, th

1>2

pr0
q

1
+ {b} > 0 >
2


2

p2r0
q


+ {b} = 4

pr0
q

pr0
q

p2r0
q





pr0
pr0
= 2
=2
,
q
q

pr0
q

+ {b} > 2


+ {b} .

C lm nh vy suy ra tn ti s nguyn dng i m (v mi ln thc hin th gi tr biu


thc tng ln)


p 2i r0

+ {b} < 1
2
q



p 2i+1 r0

2
+ {b} > 1
q
Khi




 


p 2i+1 r0
p 2i r0
p 2i r0
2
+ {b} = 4
+ {b} < 2 2
+ {b} < 2.
q
q
q
Do



 
p 2i+1 r0
+ {b} = 1.
2
q

Gi r1 l s d 2i+1 r0 khi chia cho q, suy ra v l.


h n o
i
Nu khng tn ti r0 6= 0 sao cho 2 prq0 + {b} = 0, th vi mi r0 6= 0 ta c
 




pr0
pr0
2
+ {b} = 2 3 > 2
+ {b} > 2,
q
q
m

1 > {b} > 0 2

pr0
q


>1

pr0
q

1
>
2

53

p 2r0
q







pr0
pr0
pr0
= 2
=2
1 <
.
q
q
q

Tp ch Epsilon, S 09, 06/2016


C lm nh vy, suy ra tn ti s nguyn dng i sao cho (v mi ln thc hin th gi tr
biu thc gim xung)


i
pr

2
0
2

+ {b} > 2

q



pr0 2i+1

2
+ {b} < 2
q
o
n
i
Do 2 pr0q2 + {b} > 2, nn

 






pr0 2i+1
pr0 2i
pr0 2i
pr0 2i
2
+{b} = 2 2
1 +{b} = 4
+{b}2 > 2
> 1.
q
q
q
q
h n
o
i
pr0 2i+1
Suy ra 2
+ {b} = 1. Chn r1 l s d khi chia r0 2i+1 cho q suy ra iu v l.
q


Tm li a Z+ .
V d 9. t A =

 2m
3n


| m, n Z+ , 2m > 3n . Chng minh rng inf (A) = 1.

Chng minh. Ta t B = {log3 a | a A} . Khi ta phi chng minh inf (B) = log3 1 = 0. Ta
thy b > 0, b B suy ra inf (B) > 0. Li thy vi mi b B, b c dng nlog3 2 m. Vi mi
s nguyn dng n m n log3 2 > 1, xt m = [nlog3 2] Z+ . Khi , b = {nlog3 2}. Li thy
log3 2 l s v t dng nn theo nh l Kronecker th {{nlog3 2} |n Z+ } tr mt trong (0; 1).
Do tn ti dy s nguyn dng ni bi = {ni log3 2} 0, khi i +.
Do vy inf (B) = 0, suy ra inf (A) = 1.
V d 10. (Putnam 1995) Cho A (x) = {[nx] |n Z+ } . Chng minh rng nu a, b, c R+ th
A (a) , A (b) , A (c) , khng th l mt phn hoch ca Z+ .
Chng minh. Gi s phn chng nu a, b, c R+ th A (a) , A (b) , A (c) l mt phn hoch ca
Z+ . Ta xt cc trng hp sau:
Trong cc s a, b, c c t nht mt s hu t, gi s l a. t a = pq , (p, q) = 1, q Z+ . Ta
chng minh tn ti cc s nguyn dng m, n sao cho
[ma] = [nb] .

(2.6)

Nu b Q th tn ti m, n Z+ m
(

ma, nb Z+
ma = nb

+
Khi ta
h ciiu phi chng minh. Nu b 6 Q th chn m = qt, t Z . Khi
[ma] = qt pq = pt. Do b
/ Q nn theo nh l Kronecker, tn ti n0 Z+ sao cho

{n0 b} < p1 . Khi


[(pn0 ) b] = [p [n0 b] + p {n0 b}] = p [n0 b] + [p {n0 b}] = p [n0 b] .
Chn t = [n0 b] th ta c iu phi chng minh. Vy (2.6) ng v do A (a) A (b) 6= ,
tri vi gi thit phn chng.
54

Tp ch Epsilon, S 09, 06/2016


C ba s a, b, c u l v t. Xt n Z+ bt k, khi s cc s thuc A (a) m nh hn
hay bng n l s cc s m Z+ tho mn


n+1
n+1
m6
.
[am] 6 n am 1 < n m <
a
a


Do c n+1
s thuc A (a) v nh hn hay bng n. Tng t vi b, c. Khi (do gi
a
s)
X n + 1
= n, n Z+ ,
a
a,b,c
suy ra
X
a,b,c

Cho n + c

P
a,b,c

1
a

 n+1 
a

n+1

n
.
n+1

= 1, suy ra mt trong ba s a, b, c nh hn 3, gi s l a.

Ta nhc li nh l Beatty sau:


Cho , I + :

= 1. Khi A () , A () l mt phn hoch ca Z+ .

a
a
I + v a1
> 32 . Khi A (a) , A
Tr li bi ton, do a I + , a > 1 nn a1

a
mt phn hoch ca Z+ . Khi A (b) A (c) = A a1
.

Nhn xt. Cho I, >

3
2

v R+ sao cho A () A () , khi

a
a1

Z+ .

Chng minh. Do A () A () nn vi mi m Z+ th tn ti n Z+ tho mn


[m] = [n] n 1 < [m] < n

[m]
[m] 1
<n<
+ .

(2.7)

n
o
Ta thy nu tn ti m Z+ m 0 < [m]
< 1 1 th khng tn ti n Z+ tho mn

(2.7), do






[m]
1
m {m}
1
{m}
1
+
> 1 , m Z
> 1 mx
> 1 ,

x=



{m}
1
{mx}
> 1 , m Z+ .

(2.8)

Nu x I, khi vi mi m Z+ c

{m}
1
1 {m}
1
1
{mx}
>1
{mx} > 1 +
>1
{mx} > 1

2
{m}
1
{m} 1
1 > {mx}
{mx} +
>1
+ 1 > {mx}

1) Nu

1 6 0 th ta phi c {mx} > 1 1 , m Z+ , tri vi nh l Kronecker.

2) Nu 1 > 0 th ta c 1 1 > 2 1 do > 23 , suy ra vi mi m Z+ th


{mx}
/ 2 1; 1 1 tri vi nh l Kronecker.
55

Tp ch Epsilon, S 09, 06/2016


Do x
/ I suy ra x Q.
Nu x
/ Z+ , t x = pq , (p, q) = 1, q > 1. Xt m = qk + 1, k Z+ , khi

 


{m}
p
{(qk + 1) }
{mx}
=
(qk + 1)

 

p
{k q + }
=



{kp + }
= {x}
.

(2.9)

V I nn p I, do {{k (p)} | k Z+ } tr
n mt trn (0; 1) ,ot suy ra
{{k (p) + } | k Z+ } , tr mt trn (0; 1) , dn n {k(p)+}
| k Z+ tr mt trn


0; 1 . Suy ra tn ti k0 Z+ sao cho


1
1
1
1
{k0 p + }
> {x}+ 1 do 1 > {x} > {x} + 1;
> {x} + 1 .
1 > {x} >

Khi , t (2.7) xt m = qk0 + 1, ta c



 

{m}
{k0 p + }
{mx}
= {x}

{k0 p + }
= {x}


1
1
< {x} {x} + 1 = 1 ,

tri vi (2.9) Do x phi thuc Z+ , hay

Z+ v nhn xt c chng minh.

Tr li vi bi ton, do

A (b) A (c) = A

a
a1


A (b) , A (c) A

a
a1


,

a
a
m a1
> 23 nn tn ti b0 Z+ sao cho b = a1
b0 v c0 Z+ sao cho c =
a
d = a1 b0 c0 th
A (d) A (b) , A (d) A (c) ,

a
a1

c0 . t

iu ny v l do A (b) A (c) = .
Vy iu gi s ban u l sai, do ta c iu phi chng minh.
Nhn xt. Cho ai R+ (i = 1, 2, . . . , n) i mt phn bit, khi nu A (a1 ) , A (a2 ) , . . . ,
A (an ) l mt phn hoch ca Z+ th n {1; 2} .
Tip theo chng ta cng nghin cu mt s bi ton m cch gii c dng phng php ca
tp tr mt.

V d 11. (France TST 2005) Cho A N , A 6= sao cho x A th 4x, [ x] A. Chng


minh rng A = N .
56

Tp ch Epsilon, S 09, 06/2016


 
Chng
minh.
Ta
gi
a
l
s
nguyn
dng
b
nht
thuc
A.
Nu
a
>
1
th
a0 A v
0
0
 
a0 6 a0 < a0 , mu thun vi cch chn a0 v vy a0 = 1. T gi thit suy ra
i
h
h i
1
(2.10)
4n = 2n A, n (2n ) 2m A, m, n.
4n A, n
Ta c nhn xt sau

k N , m, n N : k 2 6 2n < (k + 1)2 .
Tht vy, chn m N 2m [log2 (k + 1) log2 k] > 2, khi
m

log2 (k + 1)2 log2 k 2 > 2 n N : log2 k 2 6 n < log2 (k + 1)2 ,


suy ra
h
i
1
m
m
k 2 6 2n < (k + 1)2 k = (2n ) 2m .
T (2.10) suy ra k A nn N A. Do A = N .
V d 12. (VMO 1997, bng A) Cho s t nhin n > 1 khng chia ht cho 1997. Xt hai dy s
(ai ) v (bj ) c xc nh nh sau:
ai = i +

ni
1997j
, i = 1, 2, . . . , 1996, bj = j +
, j = 1, 2, . . . , n 1.
1997
n

Xt tt c cc s ca hai dy trn v sp th t khng gim ta c c1 6 c2 6 6 c1995+n .


Chng minh rng ck+1 ck < 2, k = 1, 2, . . . , 1994 + n.
Chng minh. Thay 1997 bi s m khng l c ca n. Xt hai dy s


m
n
ai = i 1 +
, i = 0, 1, . . . , m, bj = j 1 +
, j = 0, 1, . . . , n.
m
n
Ta c
a0 = 0 < a1 < < am = m + n, b0 = 0 < b1 < < bn = m + n.
n
Nu n < m th ai+1 ai = 1 + m
< 2. Vi mi k m 1 6 k 6 m + n 2 th tn ti duy nht j
aj 6 ck < aj+1 vi 0 6 j 6 n 1. Khi ck+1 6 aj+1 v

ck+1 ck 6 aj+1 aj < 2.


Nu m < n th bj+1 bj = 1 + m
< 2. Vi mi k m 1 6 k 6 m + n 2 th tn ti duy nht j
n
bj 6 ck < bj+1 vi 0 6 j 6 n 1. Khi ck+1 6 bj+1 v
ck+1 ck 6 bj+1 bj < 2.
Ta c iu phi chng minh.
.
.
V d 13. Vi > 1 l mt s thc sao cho m, n N , m .. n [m] .. [n]. Chng minh
N .
Chng minh. Gi s = t + r, t N , 0 6 r 6 1. T gi thit k, n N sao cho
.
[kn] .. [n] , ta c
[kn] = [knt + knr] = k [nt] + [knr] [knr]

(mod nt),

.
suy ra k, n N . Li c [knr] .. [n] . C nh k, cho n + c nt + m [knr] b
chn nn suy ra {nr} = 0 r = 0 = t N . Ta c iu phi chng minh.
57

Tp ch Epsilon, S 09, 06/2016


V d 14. (IMO 1997) Cho x1 , x2 , . . . , xn R tho mn


n
X

n+1


xi = 1, |xi | 6
, 1 6 i 6 n.



2
i=1

Chng minh rng tn ti mt hon v (y1 , y2 , . . . , yn ) ca (x1 , x2 , . . . , xn ) sao cho



n
X n + 1


iyi 6
.



2
i=1
Chng minh. Vi mi hon v = (y1 , y2 , . . . , yn ) ca (x1 , x2 , ..., xn ), ta k hiu S () l gi
tr ca tng y1 + 2y2 + + nyn . t r = n+1
. Ta cn phi chng minh rng tn ti no
2
|S ()| 6 r. Gi 0 l hon v ng nht 0 = (x
, . . . , xn ) v gi l hon v o ngc
1 , x2

= (xn , xn1 , . . . , x1 ). Nu |S (
r hoc S 6 r, bi ton coi nh c gii xong,
0 )| 6

do vy, ta gi s |S (0 )| > r v S > r.

S (0 ) + S = (x1 + 2x2 + + nxn ) + (xn + 2xn1 + + nx1 )
= (n + 1) (x1 + x2 + + xn ) .




Ta suy ra S () + S = n + 1 = 2r. Nhng v |S (0 )| > r v S > r nn ta phi

c S (0 ) v S tri du nhau, ngha l mt s th ln hn r, cn mt s th nh hn r. T
0 , ta c th thu c m = bng cch chuyn hai phn t k nhau. Ni cch khc, tn ti
mt dy cc hon v 0 , 1 , . . . , m sao cho m = v vi mi i (i = 0, 1, . . . , m 1), hon
v i+1 c c t i bng cch hon chuyn hai s hng lin tip. iu ny c ngha rng nu
i = (y1 , y2 , . . . , yn ) , i+1 = (z1 , z2 , . . . , zn ) th tn ti mt ch s k {1, 2, . . . , n 1} sao
cho
zk = yk+1 , zk+1 = yk , zj = yj , j 6= k, k + 1.
Do gi thit |xi | 6 r, i = 1, 2, . . . , n nn ta c
|S (i+1 ) S (i )| = |kzk + (k + 1) zk+1 kyk (k + 1) yk+1 | = |yk yk+1 | 6 2r.
iu ny chng t rng khong cch gia hai s lin tip bt k trong dy s S (0 ) , . . . , S (m )
khng ln hn 2r. Mt khc, c th xem S (0 ) , S (m ) l nhng im nm trn ng thng
thc, nm ngoi on [r; r], t , suy ra rng t nht mt trong cc s S (i ) phi ri vo on
; ni cch khc; tn ti i S (i ) 6 r.
Trn y chng ti ni v khi nim tp hp tr mt cng nh mt s ng dng ca n trong
gii ton s cp nhm phc v cho cc k thi Olympic ton ph thng. Vic tip tc nghin cu
vn ny s c ni trong trng trnh ton bc i hc v cc bc hc tip theo.

3. Bi tp t luyn
Bi tp 1. Cho X, Y l hai tp con ca R sao cho X tr mt trong R v X Y. Chng minh
rng Y tr mt trong R.
Bi tp 2. Cho X R v X tr mt trong R, v A l tp con hu hn ca X. Chng minh
rng X\A tr mt trong R.
58

Tp ch Epsilon, S 09, 06/2016


 
Bi tp 3. Chng minh rng tn ti v s (m, n) Z2 sao cho n 2 = 2m .
Bi tp 4. (Isarel Hungarian 1998) Tm tt c R tho mn

m
1

n N , m Z : <
.
n
3n
Bi tp 5. Cho a > b l hai s nguyn
dng tho mn ab > 4. Chng minh rng tn ti s

nguyn dng l sao cho b < 2l = 2l1 < 2l < a.
n
o

Bi tp 6. Cho tp hp S =
a b | a, b N . Chng minh rng tp S tr mt trong R.
Bi tp 7. Cho cc dy s (an )n>0 v (bm )m>0 tho mn ng thi cc iu kin sau:
lim an = lim bm = +,

n+

m+

v
lim (bm+1 bm ) = 0.

m+

Chng minh rng tp S = {an bm | m, n N } l tr mt trong R.


Bi tp 8. Cho (an ) v (bn ) l hai dy s dng, tng ngt v khng b chn. c bit, tp hp
{|an+1 an | | n N } b chn.
n
o
am

Chng minh rng tp hp S = bn | m, n N tr mt trong R+ .


Bi tp 9. Cho hm f : R+ R+ tho mn cc iu kin sau:
(i) f kh vi trn R+ .
(ii) lim f (x) = +.
x+

(iii) |f 0 (x)| < M, x R+ \ (0; 1) .


Cho (bn ) l dy s dng, tng v khng b chn. Chng minh khi tp S =
n
o
tr mt trong R+ . Do vy tp 2mn | m, n N tr mt trong R+ .

f (m)
bn

| m, n N

Bi tp 10. Cho a, b Z+ sao cho s (an) = s (bn) , n Z+ . Chng minh rng log ab Z.
Trong , s (x) l tng cc ch s ca x vit trong h nh phn.

Ti liu tham kho


[1] Titu Andreescu, Gabriel Dospinescu, Problems from the book.
[2] Art of Problem Solving: http://artofproblemsolving.com

59

Tp ch Epsilon, S 09, 06/2016

60

ABOUT MEANS OF NON-NEGATIVE


NUMBERS AND POSITIVE DEFINITE
MATRICES
inh Trung Ha
(Department of Fundamental Sciences, Ho Chi Minh University of Food Industry)
Trong bi vit ngn ny, tc gi inh Trung Ha s gii thiu v l thuyt trung bnh
ca cc s khng m v cc ma trn xc nh dng. Khi u t cc khi nim
quen thuc nh trung bnh cng, trung bnh nhn, trung bnh iu ha ca hai s,
tc gi i n khi qut ha khi nim trung bnh. Tip theo bi vit gii thiu cc
tnh cht c trng ca hm n iu lin quan n cc trung bnh. Cui cng, tc
gi gii thiu nh ngha cc dng trung bnh ca hai ma trn. T cc nh ngha ny
v cc bt ng thc s v hm, cc vn tng ng vi ma trn s c t ra mt
cch t nhin (v tc gi cng vi mt s tc gi khc cng thu c mt s kt
qu theo hng ny).
Bi vit ny da trn cc bi ni chuyn ca TS inh Trung Ha ti Nng (cho
nhm hc sinh THPT chuyn L Qu n Nng chun b d thi HSG quc gia
nm 2014) v trong bui seminar dnh cho hc sinh v sinh vin ti trung tm Titan
Education (nm 2015).
y l bi vit ngn, vn phong d c nn chng ti nguyn bn ting Anh.
c gi tin theo di, chng ti xin chuyn ng v gii thch mt s thut ng ting
Anh.
Positive definite matrices = ma trn xc nh dng
Positive semidefinite = na xc nh dng
Successive iterations = cc bc lp lin tip
Arithmetic, geometric, harmonic means = trung bnh cng, trung bnh nhn, trung
bnh iu ha.
Two-variable function = hm 2 bin
Monotone increasing = n iu tng
One-to-one map = nh x 1 1
Reverse Cauchy inequality = bt ng thc Cauchy ngc
Convex function = hm li
Classification = phn loi
Quantum information theory = L thuyt thng tin lng t
Matrix optimization = Ti u ha ma trn
Hermitian matrices = ma trn Hermite
61

Tp ch Epsilon, S 09, 06/2016


Eigenvalues = gi tr ring
Eigenspaces = khng gian ring
Orthogonal projections = hnh chiu vung gc
Axiomatic approach = cch tip cn tin
Binary operation = php ton 2 ngi
Riemannian manifold = a tp Riemann
Weighted Heron mean = trung bnh Heron c trng
Norht and south poles = bc cc v nam cc
Mathematical aspects = kha cnh ton hc
Theory of operator and matrix = l thuyt ton t v ma trn
In this short we introduce the theory of means for non-negative numbers and positive definite
matrices.

1. Some information about means for scalars


1.1. Some well-known means
We list some (not all) definitions of the geometric mean as follows:
The unique solution of xa1 x = b;
The common list of the successive iterations of harmonic and arithmetic means:

ab = lim an = lim bn ,
2an bn
,
an +bn

n
bn+1 = an +b
.
2


a x
The maximum among real x for which
is positive semidefinite, or its determix b
nant is nonnegative: x2 6 ab.




The unique midpoint for the metric on R+ given by (x, y) = log xy :

where a0 = a, b0 = b, an+1 =

(a, b) = 2(a,

ab) = 2(b,

ab);

If x(t) is the solution


of x0 = b a1 x with initial condition x(0) = x0 > 0, then

limt x(t) = ab.


It is well-known that for two nonnegative numbers a, b and s [0, 1]
(
1

a1 + b1 1
as b1s + a1s bs
a+b
) 6 ab 6
6
,
2
2
2

where ( a +b
)1 is the harmonic mean,
2
metic mean.

as b1s +a1s bs
2

62

is the Heinz mean and

(1.1)
a+b
2

is the arith-

Tp ch Epsilon, S 09, 06/2016


The AM-GM inequality states that only the square has the smallest perimeter amongst all
rectangles of equal area. At the same time, we have the following geometric interpretation: The
geometric mean and the arithmetic mean are the midpoints of curves as b1s and sa + (1 s)b,
respectively, connecting a and b. And the Heinz mean is the curve connecting the geometric
mean and the arithmetic mean when s runs from 0 to 1.
Now, let us formulate a general approach of means. A mean M of nonnegative numbers is a map
from R+ R+ to R+ such that:
1) M (x, x) = x for every x R+ ;
2) M (x, y) = M (y, x) for every x, y R+ ;
3) If x < y, then x < M (x, y) < y;
4) If x < x0 and y < y0 , then M (x, y) < M (x0 , y0 );
5) M(x, y) is continuous;
6) M (tx, ty) = tM (x, y) (t, x, y R+ .
A two-variable function M (x, y) satisfying condition (6) can be reduced to a one-variable
function
f (x) := M (1, x).
(1.2)
Namely, M (x, y) is recovered from f as M (x, y) = xf (x1 y). Mention that the corresponding
to M function f is monotone increasing on R+ . And that relation (1.2) forms a one-to-one map
between means and monotone increasing functions on R+ .
It is obvious that:
The arithmetic mean is corresponding to
The geometric mean is corresponding to
The harmonic mean is corresponding to
The log-mean L(x, y) =

xy
log xlog y

1+t
;
2

t;

2t
;
1+t

is corresponding to

t1
.
log t

1.2. Monotone functions and means


Let f be a monotone increasing function on [0, ), then for any nonnegative number a, b,



a+b
f ( ab) 6 f
.
(1.3)
2
It is natural to ask that if inequality (1.3) holds for any pair of positive number a, b will the
function f be monotone increasing on [0, )? The answer is positive, and follows from the
elementary fact that for any positive numbers a 6 b there exist positive number x, y such that a
is arithmetic mean and b is geometric mean of x, y.
An interesting and useful reverse Cauchy inequality is as follows: For any positive number x, y,
x+y
1
6 xy + |x y|,
2
2
or
min{x, y} 6
63

xy.

(1.4)

Tp ch Epsilon, S 09, 06/2016


Proposition 1. A function f on [0, ) is monotone increasing if and only if the following
inequality


x+y
1

f(
)6f
xy + |x y| ,
(1.5)
2
2
hold for any pair of positive number x, y.

Chng minh. Firstly, we show that for 0 6 a 6 b 6 2a there exist positive number x, y such
that
x+y
1

a=
, b = xy + |x y|.
2
2

We can assume that x 6 a. It is easy to see that for 0 6 a 6 b 6 2a the equation


p
b = x(2a x) + a x,
or,
2x2 + 2(b 2a)x + (b a)2 = 0
has a positive solution. Consequently, if we have (1.5), then
x+y
6f
f (a) =
2


1
xy + |x y| = f (b).
2

For arbitrary a 6 b, it is obvious that there exist numbers a1 , a2 , . . . , am such that


a = a0 6 a1 6 6 am = b and ai 6 ai+1 6

2ai .

Apply above arguments, we can get f (a) 6 f (a1 ) 6 f (a2 ) 6 6 f (an ) = f (b).
Of course we can get new characterization of monotone increasing functions by using other
inequalities in (1.1).

1.3. Some classes of convex functions with means


Let M, N be arbitrary means and f be continuous on R+ . We call f to be M N -convex if for any
nonnegative x, y,
f (M (x, y)) 6 N (f (x), f (y)).
(1.6)
If M (x, y) = N (x, y) = sx + (1 s)y, then we have the class of convex functions, and
inequality (1.6) is no thing but Jensen inequality for convex functions. Such functions have many
applications in optimization, in information theory etc.
If M (x, y) = sx + (1 s)y, N (x, y) = xs y 1s , then we have the class of log-convex functions.
If M (x, y) = N (x, y) = xs y 1s , then we have the class of geometrically convex functions.
According to applications of several kinds of convexity many mathematicians give effort to study
the problem of classification of convex functions. And a such problem is more complicated for
matrices.
64

Tp ch Epsilon, S 09, 06/2016

2. A little about matrix means


In this section we talk about the matrix means that have many applications in quantum information
theory, matrix optimizations etc.
Let Mn be the space of n n complex matrices. For A, B Mn , the notation A 6 B means that
B A Mn+ (i.e., < (B A)x, x > > 0 for any vector x). The spectrum of a matrix A Mn
is denoted by (A). For a real-valued function f of a real variable and a matrix A Mnh , the
valueP
f (A) is understood by means of the functional calculus for Hermitian matrices, i.e. if
A=
i Pi (where i are eigenvalues P
of A, Pi are orthogonal projections on the eigenspaces of
i ), then the matrix f (A) is defined as f (i )Pi . We can understand f (A) in another way: Let
A = U diag(1 , 2 , . . . , n )U (where i are the eigenvalues of A), then
f (A) = U diag(f (1 ), . . . , f (n ))U.
Taking an axiomatic approach, Kubo and Ando [6] introduced the notions of connection and
mean. A binary operation defined on the set of positive definite matrices is called a connection
if
(i) A 6 C, B 6 D imply AB 6 BD;
(ii) C (AB)C 6 (C AC)(C BC);
(iii) An A and Bn B imply An Bn AB If II = I, then is called a mean.
For A, B > 0, the t-geometric mean of positive definite matrices A, B is
t
A]t B := A1/2 A1/2 BA1/2 A1/2 .

(2.1)

When t = 1/2, A]B := A]1/2 B is called the geometric mean of A and B, and it was first
introduced in [8] and is often denoted by A]B in the literature (the problem of defining the
geometric mean of two positive definite matrices was standing for more than 25 years because
we could not just take the square root of product of two positive definite matrices AB!!).
The harmonic A!B and arithmetic AB means are defined A!B = 2(A1 + B 1 )1 and
A+B
AB =
, respectively. A mean is called to be symmetric if AB = BA for any pair
2
of positive definite matrices A, B.
It is well-known that the arithmetic mean is the biggest among symmetric means. From the
general theory of matrix means we know that > and >!. And we still have the following
inequalities
A!B 6 A]B 6 AB.
(2.2)
The proof of (2.2) is not very difficult and based on the scalar case.
The t-geometric mean is interesting from the point of view of Riemannian geometry since the set
Pn of positive semidefinite matrices is a Riemannian manifold with the Riemannian metric [9]:
!1/2
n
X
(A, B) =
log2 i (A1 B)
, A, B Pn ,
i=1

which is the distance between A and B so that the curve (t) = A]t B, 0 6 t 6 1, is the unique
geodesic joining A and B in Pn . The picture is more interesting if we consider the weighted
Heron mean (1 ) A+B
+ A]B( [0, 1]) that connects two midpoints A+B
and A]B (this
2
2
model looks like the earth with A and B are the north and south poles, respectively!).
65

Tp ch Epsilon, S 09, 06/2016

3. Concluding comment
Many properties formulated in Subsection 1.1 have been studied for matrices [10]. Some interesting information related to the last part of Subsection 1.1 an be found in [1] (Petz is one
of the most active mathematicians in studying mathematical aspects of quantum information
theory) where the author discusses about means of more than 2 positive semidefinite matrices.
Matrix analogs of results in Subsection 1.2 recently was studied by Ando and Hiai [11] and by
the author [12]. At that time, problems of characterization of matrix convex functions in sense of
Subsection 1.3 are still in research interests of many mathematicians. Nowadays, the theory of
operator and matrix means is one of the most studied in matrix analysis with many applications
in machine learning and quantum theory etc.

Ti liu tham kho


[1] D.Petz. Means of positive matrices: Geometry and a conjecture. Annales Mathematicae et
Informaticae. 32 (2005) 129-139.
[2] F. Topse. Basic Concepts, Identities and Inequalities theToolkit of Information Theory.
Entropy, 3 (2001) 162.
[3] K. M. R.Audenaert, J. Calsamiglia, LI. Masanes, R. Munoz-Tapia, A. Acin, E. Bagan and
F. Verstraete, Discriminating States: The Quantum Chernoff Bound, Phys. Rev. Lett. 98
(2007) 160501.
[4] D. Bini, B. Meini, F. Poloni, An effective matrix geometric mean satisfying the Ando-LiMathias properties, Math. Comp. 79 (2010) 437-452.
[5] S. Furuichi, K. Yanagi, and K. Kuriyama, Fundamental properties of Tsallis relative entropy,
J. Math. Phys. 45(12) (2004) 4868-4877.
[6] F. Kubo and T. Ando, Means of positive linear operators, Math. Ann. 246(3) (1980) 205-224.
[7] H. Lee, Y. Lim, T. Yamazaki, Multi-variable weighted geometric means of positive definite
matrices, Linear Algebra Appl. 435(2) (2011) 307-322.
[8] W. Pusz, S.L. Woronowicz, Functional calculus for sesquilinear forms and the purification
map, Rep. Math. Phys., 8 (1975), 159-170.
[9] R. Bhatia, Positive Definite Matrices, Princeton University Press, New Jersey, 2007.
[10] J. D. Lawson, Y. Lim, The geometric mean, matrices, metrics, and more, Amer. Math.
Monthly 108 (2001), 797-812.
[11] T. Ando, F. Hiai, Log majorization and complementary Golden-Thompson type inequalities,
Linear Algebra Appl., 197/198 (1994), 113131.
[12] Dinh Trung Hoa. On characterization of operator monotone functions. Linear Algebra and
Its Applications. 487 (2015) 260-267.

66

IM KOSNITA V MT S NG THNG
I QUA N
Trnh Huy V (Lp 12 A1 Ton, trng THPT Chuyn KHTN, H Ni)

Bi vit nu ra nh ngha v mt s kt qu v im Kosnita. ng thi bi bo


cng s a ra chng minh thun ty cho mt s ng thng i qua im Kosnita.

1. nh ngha
nh ngha 2. im Kosnita K ca 4ABC bt k c xc nh l im ng quy ca ba
ng thng ni ba nh A, B, C tng ng vi tm ngoi tip ca 4OBC, 4OCA, 4OAB vi
O l tm ngoi tip ca 4ABC. im Kosnita l Kimberling center X(54) trong Encyclopedia
of Triangle Center (ETC) [1].

A
E

D
C

Hnh 10.1: im Kosnita

2. Mt s kt qu quen thuc
Sau y, tc gi s trnh by ra mt s kt qu quen thuc v im Kosnita m tc gi s dng
trong bi vit.
67

Tp ch Epsilon, S 09, 06/2016


Kt qu 1. 4ABC c tm ng trn Euler N v im Kosnita K. Khi , N v K ng gic
trong 4ABC.
nh ngha 3. Cho tam gic ABC vi tm ni tip I. ng thng Euler ca 4IBC, 4ICA,
4IAB, 4ABC ng quy ti mt im. im ng quy c gi l im Schiffler ca
4ABC.
Kt qu 2. Gi D, E, F ln lt l trung im ca cc cung nh BC, CA, AB ca 4ABC.
Khi , im Kosnita ca 4DEF cng l im Schiffler ca 4ABC.
Chng minh. X l trung im BC. V ng knh DJ. Ly cc im D0 , E 0 , F 0 ln lt i
xng D, E, F qua BC, CA, AB. Gi N, U, V, W tng ng l trung im ca IO, ID0 , IE 0 , IF 0 .
Ta c cc kt qu quen thuc: D, E, F tng ng l tm ngoi tip ca cc tam gic IBC , ICA
, IAB; I, N l trc tm v tm ng trn Euler ca 4DEF .
Ta c DI 2 = DB 2 = DC 2 = DX.DJ (theo h thc lng trong tam gic vung). T
DI
DO
=
ta c DI 2 = DX.DJ = 2DX.DO = DD0 .DO. Suy ra
. Do , 4DOI
DI
DD0
4DID0 (c.g.c). M hai tam gic ny c hai trung tuyn tng ng l DN v DU , do vy
N DI = U DO. Ni cch khc, DN v DU ng gic trong IDO. Mt khc, do I, O tng
ng l trc tm v tm ngoi tip ca 4DEF nn DH, DO ng gic trong EDF . T ta
thu c DN v DU ng gic trong EDF . Mt khc ta li c U l tm ng trn Euler
ca 4IBC nn DU chnh l ng thng Euler ca 4IBC. Vy DN v ng thng Euler
ca 4IBC ng gic trong 4DEF . Chng minh tng t ta cng c EN , ng thng
Euler ca 4ICA v F N , ng thng Euler ca 4IAB l hai cp ng thng ng gic
trong 4DEF . Do vy im Schiffler S ng gic vi N qua 4DEF . Theo Kt qu 1, im
Schiffler S ca 4ABC cng l im Kosnita ca 4DEF

J
E

O
F
N

D0

U
I

X
C

Hnh 10.2: im Kosnita ca 4DEF v im Schiffler ca 4ABC trng nhau

68

Tp ch Epsilon, S 09, 06/2016


Nhn xt. Kt qu 2 cn c cch chng minh khc ca Telv Cohl trong [4]. T kt qu ny
thu c h qu l im Kosnita ca 4DEF nm trn ng thng Euler ca 4ABC. Hn
na, da s trng nhau ca im Schiffler v im Kosnita ca 4ABC v 4DEF , ta cng
thu c chng minh ca kt qu sau:
Kt qu 3. Gi D, E, F ln lt l trung im ca cc cung nh BC, CA, AB ca ng trn
(O) ngoi tip 4ABC. H l trc tm ca 4ABC v K l im Kosnita ca 4DEF . Khi ,
OK
R
=
vi R, r tng ng l bn knh ca ng trn ngoi tip v ni tip tam gic
OH
2r + 3R
ABC.
G

Ha
E
A

L
F
I

H
X
C

Hnh 10.3: T s OK/OH

Trc ht ta nu ra mt b quen thuc, tc gi xin php khng nu chng minh ca b ny.


B 5. Cho 4ABC vi trc tm H, tm ngoi tip O v M l trung im BC. Khi o,
AH = 2OM .
Chng minh. L l hnh chiu ca I ln AC. X l trung im BC. Gi Ha l trc tm ca
4IBC. Do K nm trn ng thng Euler ca 4IBC nn K DHa . t DHa ct AH ti G.
DI
DB
XD
XD
Do 4ALI 4BXD(g.g) nn
=
=
=
. T p dng nh l Thales, ta
AI
AI
IL
r
IHa
DI
XD
2DX
XD
suy ra
=
=
. Theo b th
=
. Do vy, AG = 2(XD + r),
AG
DA
XD + r
AG
XD + r
69

Tp ch Epsilon, S 09, 06/2016


suy ra HG = AH + AG = 2OX + 2(XD + r) = 2(r + R). Theo nh l Thales suy ra
OK
OD
R
OK
R
=
=
hay
=
.
KH
HG
2(r + R)
OH
2r + 3R

3. Mt s ng thng i qua im Kosnita


Trong mc ny tc gi s pht biu mt s ng thng i qua im Kosnita. Hu ht cc ng
thng ny u c lit k trong ETC [1] nn tc gi ch pht biu li v ng thi a ra
nhng chng minh thun ty hnh hc cho chng.

3.1. ng thng i qua nh nghch o ca trc tm H qua ng


trn ngoi tip (O) v tm ca ng trn Taylor (X(186)X(389))
nh ngha 4. 4ABC vi A0 , B 0 , C 0 tng ng l chn ba ng cao. AB , AC ln lt l
hnh chiu ca A0 trn AB, AC. Xc inh cc im BC , BA , CA , CB tng t. Khi , 6 im
AB , AC , BC , BA , CA , CB cng nm trn mt ng trn. ng trn ny c gi l ng
trn Taylor [5]. Tm ng trn Taylor trong ETC l Kimberling center X(389) [1].

BA

CA
B0
AC

C0

AB

CB

A0

BC

Hnh 10.4: ng trn Taylor

Chng minh ng trn Taylor. Ta thc hin php bin i gc sau:


AAB AC = AA0 AC (A, AB , A0 , AC nm trn ng trn (AA0 ))
= ACB (cng ph A0 AC)
= AC 0 B 0 (B, C, B 0 , C 0 nm trn ng trn (BC))
= ACA BA (B 0 , C 0 , BA , CA nm trn ng trn(B 0 C 0 ))
Do vy 4 im AB , AC , BA , CA cng thuc mt ng trn. Chng minh tng t th BA BC CB AB
v CA CB BC AC u l cc t gic ni tip ng trn. Gi s ba ng trn ny khng trng
nhau th ta thy BC CB , CA AC , AB BA ln lt l cc trc ng phng ca 2 trong b 3 ng
trn {(AB AC BA CA ); (BA BC CB AB ); (CA CB BC AC )}, tc l BC CB , CA AC , AB BA phi ng
70

Tp ch Epsilon, S 09, 06/2016


quy (mu thun). Do vy, 3 ng trn ny trng nhau v 6 im AB , AC , BC , BA , CA , CB cng
thuc 1 ng trn.
Cc chng minh khc cho ng trn Taylor tham kho thm trong [6], [7].
nh ngha 5. Cho 4ABC vi D, E, F ln lt l trung im ca BC, CA, AB. Khi , tm
ni tip ca 4DEF c gi l im Spieker ca 4ABC.
Sau y tc gi s trnh by thm mt tnh cht lin quan n tm ng trn Taylor X(389) v
im Spieker.
B 6. 4ABC nhn vi A0 , B 0 , C 0 ln lt l ba chn ng cao. Khi , tm ca ng
trn Taylor ca 4ABC l im Spieker ca 4A0 B 0 C 0 .
A

BA

CA
B0
D
AC

C0
S
AB

E
CB

A0

BC

Hnh 10.5

Chng minh. S l tm ng trn Taylor ca 4ABC. Gi D, E, F ln lt l trung im ca


B 0 C 0 , C 0 A0 , A0 B 0 . Trc ht ta chng minh AB AC i qua E, F . Tht vy, do E l trung im ca
C 0 A0 nn EAB A0 = C 0 A0 AB = 90 BC 0 A0 = 90 ACB = A0 AAC = AC AB A0 .
Do , E AB AC . Tng t vi F suy ra AB AC i qua E, F . Chng minh tng t nh
trn ta c BA BC i qua F, D v CA CB i qua D, E. Mt khc d thy BA CA k CB BC nn
BA CA BC CB l mt hnh thang cn, m D = BA BC CA CB suy ra 4DBC CB l tam gic cn
v DS l phn gic ca BC DCB EDF . Tng t, ES l phn gic ca DEF . Vy S l
tm ni tip ca 4DEF v cng l im Spieker ca 4A0 B 0 C 0 .
Nhn xt. Kt qu trn ch ng khi 4ABC l tam gic nhn. Khi 4ABC t ln lt ti
A, B, C th tm ng trn Taylor ca 4ABC s l tm bng tip tng ng vi cc nh
D, E, F ca tam gic trung bnh DEF ca 4A0 B 0 C 0 .
Tc gi xin tip tc nu thm ra mt kt qu ni ting lin quan n im Spieker:
B 7. 4ABC vi tm ni tip I, trng tm G v im Spieker Sp v im Nagel Na . Khi
, I, G, Sp , Na cng nm trn mt ng thng v INa = 2ISp = 3IG.
y l mt nh l ni ting, bn c c th tham kho chng minh ca n t trang 7-12 trong
[6].
71

Tp ch Epsilon, S 09, 06/2016

E
J
X
K

Ho

S
G
H

R
Z

P
N

C
B

Hnh 10.6: nh l 3.1

nh l 1. Cho 4ABC vi trc tm H, ng trn ngoi tip (O) v 4DEF l tam gic to
bi ba chn ng cao ca 4ABC. S l tm ng trn Taylor ca 4ABC. H l nh nghch
o ca H qua (O). Khi , ng thng SH i qua im Kosnita K ca 4ABC.
Trong , nh nghch o H ca H ca (O) l Kimberling center X(186) trong ETC [1].
Trong bi vit ny tc gi s nu chng minh ca nh l 1 trong trng hp 4ABC nhn
(cc trng hp cn li chng minh tng t). Khi , tm ng trn Taylor chnh l im
Spieker ca tam gic DEF . Do vy, ta quy bi ton v vic chng minh ng thng i qua
im Spieker ca 4DEF v nh nghch o ca H qua (O) i qua im Kosnita K.

Chng minh nh l. N l tm ng trn Euler ca 4ABC v P l trung im ca N O.


X, Y, Z, J, L, M tng ng l trung im ca EF, F D, DE, HA, HB, HC. D thy H l tm
ni tip ca 4DEF (kt qu ni ting). Gi Ho , G ln lt l trc tm v trng tm ca 4DEF .
Ta c Ho , G, N thng hng trn ng thng Euler ca 4DEF v H, G, S thng hng,
HG = 2GS. Gi Q l trung im ca HK. r, R ln lt l bn knh ng trn ni tip v
ngoi tip ca 4DEF .

2 
2
OH
AH
OH
N H2
2

Ta c OA = OH.OH suy ra
=
=
=
. Mt khc, p dng
OH
AH
OA
R2
cng thc Euler v ch l H, N ln lt l tm ni tip v tm ngoi tip ca 4DEF ,
OH
R 2r
OP
R 2r
th N H 2 = Rr2 2Rr = R(R 2r). Do ,
=
. Vy
=
hay

OH
R
OH
4R

PH
3R + 2r
.
=

OH
4R
72

Tp ch Epsilon, S 09, 06/2016


1
Xt php v t tm H t s , A 7 J; B 7 L; C 7 M ; K 7 Q, do Q l im Kosnita ca
2
4JLM . Mt khc, ta c kt qu ni ting J, L, M ln lt l trung im cc cung EF, F D, DE
ca ng trn (DEF ). Theo kt qu 2.2, 2.3 th Q l im Schiffler ca 4DEF , Q Ho N v
R
3R
NQ
NQ
=
. Suy ra
=
. Mt khc, theo nh l Thales ta c, OK = 2N Q
N Ho
2r + 3R
NG
2r + 3R
3N G
v SP =
. Do vy,
2
3N G
PS
3N G
3 2r + 3R
2r + 3R
= 2 =
= .
=
OK
2N Q
4N Q
4
3R
4R
P H
PS
. Suy ra 4P H S 4OH K(c.g.c) nn P H S = OH K. Vy H , S, K
=

OH
OK
thng hng.

Vy

3.2. ng thng i qua Trung im HG v tm ng trn Euler ca


tam gic to bi ba ng tip tuyn qua A, B, C(X(156)X(381))
nh l 2. Cho 4ABC vi trc tm H, trng tm G v ng trn ngoi tip (O). Cc tip
tuyn ti A, B, C ca ng trn (O) ct nhau to thnh 4DEF . I l trung im ca HG. L
l tm ng trn Euler ca 4DEF . Khi , IL i qua im Kosnita K ca 4ABC
Trong , trung im I ca HG l Kimberling center X(381) ca 4ABC, tm ng trn
Euler L ca tam gic DEF l Kimberling center X(156) ca 4ABC trong ETC [1].
chng minh nh l 2, tc gi xin pht biu khng chng minh b quen thuc sau:
B 8. Cho 4ABC vi tm ngoi tip O, ng trn ni tip (I). Tip im ca (I) vi
BC, CA, AB tng ng l D, E, F . H l trc tm 4DEF . R, r ln lt l bn knh ca ng
trn ngoi tip v ni tip ca 4ABC. Khi , OI trng vi ng thng Euler IH ca 4DEF
IH
r
v
= .
OI
R
Chng minh nh l. Gi H , O , S ln lt l trc tm, tm ngoi tip v im Schiffler ca
4DEF . X, Y, Z tng ng l trung im cc cung EF, F D, DE ca ng trn (O ) ngoi
tip 4DEF . Theo kt qu 2 th S chnh l im Kosnita ca 4XY Z. t R, R ln lt l
bn knh ng trn ngoi tip cc tam gic ABC v DEF .
Do cng vung gc vi OD nn BC k Y Z. Tng t ta cng c CA k ZX; AB k XY . Do
4ABC v 4XY Z v t nhau, m O, O tng ng l tm ngoi tip ca 4ABC v 4XY Z;
K, S tng ng l im Kosnita ca 4ABC v 4XY Z. T suy ra OK k O S O H
OK
OA
R
O S
R
v =
=
. Mt khc, p dng kt qu 3 suy ra =
hay
O S
O X
R
O H
2R + 3R
O S
2R
=
. V vy, ta c:

O L
2R + 3R
OK
OK O S
R
2R
2R
=
=

O L
O S
O L
R 2R + 3R
2R + 3R

73

Tp ch Epsilon, S 09, 06/2016

F
G
I
B
Y

C
Z
H

Hnh 10.7: nh l 2

Mt khc d thy (O) cng chnh l ng trn ni tip ca 4DEF . Theo b 4 th OO


OH
R
trng vi ng thng Euler ca 4ABC, ni cch khc l O OH v
. M I l
=

OO
R
2
OI
= . Do ,
trung im ca HG nn HI = IG = GO hay
OH
3
OI
OI OH
2 R
2R
IO
2R
=

=
=
=
OO
OH OO
3 R
3R
IO
2R + 3R
OK
IO
= . Kt hp vi OK k O L suy ra 4IOK 4IO L(c.g.c), nn

IO
O L

[
\
OIK = O IL. T ta c I, K, L thng hng.
Do vy ta c

3.3. ng thng i qua trc tm H v im Spieker ca tam gic


to bi ba tip tuyn (X(4)X(6759))
cho thun tin, tc gi s s dng nhng k hiu ca dng trong vic pht biu v chng
minh nh l 2 pht biu nh l 3
nh l 3. Gi Sp l im Spieker ca 4DEF . Khi , ng thng HSp i qua im Kosnita
K ca 4ABC.
im Spieker ca tam gic to bi ba tip tuyn l Kimberling center X(6759) ca 4ABC.

74

Tp ch Epsilon, S 09, 06/2016


E

U
A

O
G
T
I
K

Sp
W

Hnh 10.8: nh l 3

Chng minh nh l. Gi G l trng tm ca 4DEF . Ta c G nm trn H O (ng thng


Euler ca tam gic DEF . Theo b 3, ch O l tm ni tip ca 4DEF v Sp l im
Spieker ca 4DEF , suy ra O, G , Sp thng hng v OG = 2G Sp . t IG ct OK ti T .
T php chng minh nh l 2 trn th ta c OK k O L. p dng nh l Thales, ta
OT
IT
TK
OT
O G
c =
= . T suy ra
=
. Mt khc, do O G = G L nn

O G
IG
GL
TK
GL
OT
OG
OI
OT = 2T K. T
=
= (= 2). Theo nh l Thales o suy ra HK k IT v
IH
TK
G Sp

KSp k T G . Theo tin Euclid th H, K, Sp thng hng.


Nhn xt. Kt qu v ng thng i qua 3 im H, K, Sp thc t ch l mt h qu ca nh l
2. Hn na, ta cng thu c h qu:
H qu 4.

HK
2R
=
HSp
2R + 3R

HK
OH
HSp
Chng minh. Tht vy, p dng nh l Thales cho IG k HSp th
=
=
. T
IT
OI
IG
HK
IT
IO
2R
suy ra
=
=
=
(do OK k O H ).

HSp
IG
IO
2R + 3R

3.4. ng thng i qua ng gic ca im De Longchamps v i


xng ca im Exeter qua tm ngoi tip O (X(64)X(378))
nh ngha 6. Cho 4ABC vi trc tm H v tm ngoi tip O. im De Longchamps ca
4ABC c xc nh l im i xng ca trc tm H qua tm ngoi tip O [8]. im de
Longchamps l Kimberling center X(20) trong ETC [1].
75

Tp ch Epsilon, S 09, 06/2016


A
J0

J
B0
C0
H

X
B

D
U

Hnh 10.9: B 5

B 9. Cho 4ABC ni tip ng trn (O). D l trung im ca BC v H l trc tm ca


4ABC. Tia DH ct (O) ti J. J 0 l mt im trn ng trn (O) sao cho JJ 0 k BC. Tip
tuyn ca (O) ti B, C ct nhau ti T . Khi , AJ 0 T = 90 .
Chng minh b . Dng ng knh AE ca (O). t U = AH (O). D thy U E k BC k
JJ 0 . Gi BB 0 , CC 0 l cc ng cao ca 4ABC. Ta c kt qu ni ting l AJ, BC, B 0 C 0 ng
quy ti mt im, gi im l X. Ta cng c A(BC, U X) = 1 (chm c bn). T suy ra
A(BC, U J) = 1. Do vy, t gic BJCU l t gic iu ha. Vi vy J, U, T l 3 im thng
hng. Mt khc, do U E k BC k JJ 0 nn bng php i xng qua trung trc BC, ta suy ra
J 0 , E, T thng hng. Do vy AJ 0 T = AJ 0 E = 90 .
T b trn ta chng minh c mt tnh cht lin quan n ng gic ca im de Longchamps
di y.
B 10. Cho 4ABC ni tip ng trn (O). im de Longchamps L v L0 l ng gic
ca L trong 4ABC. Cc tip tuyn ti A, B, C ca (O) ct nhau to thnh 4DEF . V ng
knh AA0 , BB 0 , CC 0 ca (O). Khi , DA0 k AL0 , EB 0 k BL0 , F C 0 k CL0 . Hn na, ta bit
DA0 , EB 0 , F C 0 ng quy ti im Nagel Na ca 4DEF (kt qu ni ting), th Na v L0 i
xng nhau qua O.
Chng minh b . t A0 H ct ng trn ngoi tip (O) ln th hai ti G. Ly G0 (O)
sao cho GG0 k BC. Theo b 5 th D, A0 , G0 thng hng. Mt khc, L i xng H qua O
nn AHA0 L l mt hnh bnh hnh. Suy ra A0 L k AH v A0 L BC. Dt A0 L (O) = J
(J 6= A0 ) th AJA0 = 90 , do AJ k BC k GG0 . Suy ra G0 A0 A = HA0 L = HAL
(do AHA0 L l mt hnh bnh hnh). M L, L0 v H, O l hai cp n gic trong 4ABC nn
L0 AO = HAL. Do vy L0 AO = G0 A0 A. Suy ra AL k D, A0 , G0 DA0 . Tng t
EB 0 k BL0 , F C 0 k CL0 .
Gi L i xng L qua O. Ta c AL0 A0 L l hnh bnh hnh v A0 L k A0 L. T suy ra
L DA0 . Tng t L EB 0 v L F C 0 . Vy L = DA0 EB 0 F C 0 nn L Na . Do
, Na v L i xng nhau qua O.
76

Tp ch Epsilon, S 09, 06/2016


E

J
Na

C0

B0

G0

H
L0

B
A0

Hnh 10.10: B 6

nh ngha 7. Cho 4ABC vi ng trn ngoi tip (O). Cc trung tuyn ca tam gic ABC
ct (O) ln lt ti X, Y, Z. Cc tip tuyn ca (O) ti A, B, C ct nhau to thnh tam gic
DEF . Khi , DX, EY, F Z ng quy ti mt im. im ng quy ny c gi l im Exeter
ca 4ABC [9]. im Exeter l Kimberling center X(22) trong ETC [1].
B 11. Cho 4ABC ni tip ng trn (O). ng trn ni tip (I) tip xc BC, CA, AB
ti D, E, F tng ng. Cc ng trn Mixtilinear ng vi cc nh A, B, C ln lt tip xc
(O) ti X, Y, Z. Ta c kt qu ni ting AD, BE, CF ng quy ti im Gergonne Ge ca
4ABC v AX, BY, CZ ng quy ti mt im J l ng gic ca Ge trong 4ABC. Hn na,
JI
r
J cn nm trn OI v
= vi R, r tng ng l bn knh ng trn (O) v (I).
JO
R
Bnh lun. B trn l mt kt qu ni ting nn tc gi xin php khng trnh by li chng
minh. Da vo b ny ta c th chng minh c tnh cht quan trng sau ca im Exeter.
B 12. im Exeter Ex nm trn ng thng Euler ca 4ABC. Hn na, ta cng c t s
R
Ex O
=
vi O, O l tm ngoi tip ca 4ABC, 4DEF v R, R l bn knh ca ng

Ex O
R
trn ngoi tip 4ABC v 4DEF .
Chng minh b . Gi H l trc tm ca 4ABC. M l trung im BC v X = AM (O).
Ta c cc t gic BOCD v ABXC ni tip nn M O M D = M B M C = M A M X. Do
, t gic AOXD l t gic ni tip. Vy XDO = XAO = AXO = ADO hay DO l
phn gic ca ADX. Suy ra DA v DX DEx l 2 ng ng gic trong EDF . Tng t
EB v EEx ; F C v F Ex l cc cp 2 ng ng gic trong 4DEF . Vy Ex l ng gic ca
im Gergonne Ge (ca 4DEF ) trong 4DEF . Theo b 7 th Ex nm trn ng thng
Ex O
R
OO , ng thng ny ng thi cng l ng thng Euler ca 4ABC, v
=
.

Ex O
R
77

Tp ch Epsilon, S 09, 06/2016


E

A
F
O

O
Ex
B

C
M

Hnh 10.11: B 8

nh l 5. Cho 4ABC vi im De Longchamps L v im Exeter Ex . L l im ng gic


ca L trong 4ABC. Ex0 i xng vi Ex qua O. Khi , L Ex0 i qua im Kosnita ca 4ABC.
Chng minh nh l. Gi H l trc tm ca 4ABC. O0 i xng O qua L. Cc tip tuyn ti
A, B, C ca (O) ct nhau to thnh tam gic DEF . Sp , Na ln lt l im Spieker v im
Nagel ca 4DEF . O l tm ngoi tip ca 4DEF . R, R ln lt l bn knh ng trn
ngoi tip ca 4ABC v 4DEF .
Ex O
R
OEx
R
Theo b 8 th
=
hay
=
. Mt khc, li theo b 4, th O OH

Ex O
R
OO
R + R
HO
R
OEx0
OEx
OEx OO
R
R
R
v
=
.
T

suy
ra
=
=

=
. Do vy,

OO
R
HO
HO
OO HO
R + R R
R + R
HEx0
R
=
, m HO0 = 3HO nn ta c
HO
R + R
HEx0
R
HEx0
R
=
=
=
(1)
0
0
0
HO
3(R + R )
Ex O
2R + 3R
Ch rng O, Sp , Na ln lt l tm ni tip, im Spieker v im Nagel ca 4DEF nn
ta c Sp l trung im ONa (xem b 3). Kt hp vi L i xng
qua O, nn ta
 Na 
HO
OS
1
p
c OL = ONa = 2OSp . T theo nh l Thales, do
=
=
nn ta c
OO0
OL
2
HK
2R
HK k L O0 v L O0 = 2HSp . Mt khc, ta li c
=
(h qu 4). Suy ra,
HSp
2R + 3R
HK
HK
R
=
=
(2).
L O 0
2HSp
2R + 3R
78

Tp ch Epsilon, S 09, 06/2016


E

Na

A
Sp

C0

Ex L

K
H

B0

O0

Ex0

A0

Hnh 10.12: nh l 5

HK
HEx0
= 0 . M KHEx0 = L O0 Ex0 (do HK k L O0 ). Do ,
0
0
Ex O
LO
4HKEx0 4OL Ex0 . Suy ra HEx0 K = O0 Ex0 L = 180 HEx0 L hay KEx0 L =
HEx0 K + HEx0 L = 180 . Do vy, L , Ex0 , K thng hng.
T (1) v (2) suy ra

3.5. ng thng i qua trc tm tam gic Pedal ca K v tm ngoi


tip O
A

E
F

H K

Hnh 10.13: nh l 6

79

Tp ch Epsilon, S 09, 06/2016


nh l 6. Cho tam gic ABC vi im Kosnita K v tm ngoi tip O. H l trc tm tam gic
pedal ca K ng vi tam gic ABC. Khi , ng thng OH i qua im Kosnita K.
nh l trn c tc gi xut trn din n AoPS trong [2]. chng minh nh l 6, ta
cn c mt b :
B 13. Cho 4ABC vi P l mt im bt k. Cc ng thng qua P ln lt vung gc
vi AP, BP, CP ct BC, CA, AB tng ng ti X, Y, Z. Khi , X, Y, Z thng hng.
y l mt kt qu ni ting nn tc gi s khng trnh by php chng minh.
Tr li chng minh nh l. Ta gi N l tm ng trn Euler ca 4ABC. Cc ng thng
qua N ln lt vung gc vi AN, BN, CN ct BC, CA, AB tng ng ti X, Y, Z. Theo b
1.1 th X, Y, Z cng nm trn mt ng thng. Sau y, chng minh nh l, tc gi s
chia ra chng minh hai bi ton nh sau y:
Bi ton 1. OK X, Y, Z.

L
Q

K
O

X
N
X
B

Oa

Hnh 10.14: Bi ton 1

Chng minh. Gi P, Q, R, L ln lt l trung im BC, CA, AB, OA. I, Oa tng ng l tm


ngoi tip ca 4AN L v 4OBC. Khi , ta c N, L l tm ngoi tip ca 4P QR v 4ARQ.
M A, D i xng nhau qua trung im qua QR nn L, N cng i xng qua trung im QR.
Mt khc, LN li trung trc ca QR. Suy ra L, N i xng qua QR, ni cch khc, QR l trung
trc ca LN . T suy ra I QR.
80

Tp ch Epsilon, S 09, 06/2016


Ly X 0 i xng A qua I th X 0 BC v AX 0 l ng knh ca (AN L). Suy ra AN X =
ALX = 90 . T ta c N X 0 AN, X 0 L AO v X 0 X. Do , XL AO nn X
thuc trung trc ca AO.
Xt php nghch o I qua ng trn (O). Gi X , Y , Z ln lt l nh ca X, Y, Z qua php
nghch o I. Do X l giao im ca trung trc AO vi BC nn X s l giao im ca ng
trn (A, AO) vi ng trn (OBC). T suy ra AOa l trung trc ca OX . M K AOa
nn KO = KX . Chng minh tng t suy ra KO = KX = KY = KZ hay K chnh l
tm ngoi tip ca 4X Y Z . Theo tnh cht ca php nghch o ta suy ra OK X, Y, Z.
Bi ton 2. HK X, Y, Z

E
F
H

K
N

X
B

Hnh 10.15: Bi ton 2

Chng minh. Theo kt qu 1 th N, K l hai im ng gic trong 4ABC. Do , ta c


AN EF, BN F D, CN DE. Suy ra N Y k F D HE, N Z k DE HF . Xt
4KEF v 4N Y Z c N A EF, Y E KE, ZF KF v N A, Y E, ZF ng quy ti A.
T suy ra 4KEF v 4N Y Z trc giao. Vy cc ng thng qua K, E, F ln lt vung
gc vi Y Z, N Y, N Z ng quy. M EH N Y v F H N Z. T ta c KH Y Z.
Ta gii quyt xong hai bi ton nh, p dng chng ta suy ra H, K, O thng hng trn mt
ng thng vung gc vi X, Y, Z. nh l 6 c chng minh xong.
81

Tp ch Epsilon, S 09, 06/2016


Nhn xt. C hai bi ton nh trn ngoi cc chng minh trn vn cn c cc cch tip cn v
chng minh khc trong [2] v [3]. Cng trong [2], Telv Cohl pht biu v chng minh thm
mt tnh cht lin quan n nh l 6 nh sau:
nh l 7. HO = 3HK
Tc gi xin dn li chng minh ca Telv Cohl cho nh l 7 trong [2] di y:

O
D

Ka

Qa
Ha

N P
H

M
Hb

Hc
C

Ka

OA

Hnh 10.16: nh l 7

Chng minh nh l 7. Gi T l trc tm ca 4ABC. Ka , Ha ln lt l tm ngoi tip v


trc tm ca 4AEF (Xc nh cc im Kb , Kc , Hb , Hc tng t). t J AN (ABC)
v Ka , OA i xng ca Ka , O qua EF, BC. T JBC = N AC = BAK = F EK

v JCB = N AB = CAK = EF K, ta c 4JBC 4KEF , do kt hp


vi COA B = BOC = 2BAC = F Ka E = EKa F (ch rng OA B = OA C v

Ka E = Ka F ) = 4JBC OA 4KEF Ka , do vy EKKa = OA JB = ACB =


EKD = Ka DK. V Ha i xng K qua trung im EF , nn KKa Ha Ka l mt hnh
bnh hnh, d t KKa BC ta suy ra Ka Ha BC = Ka Ha i qua trung im M ca
KT (v Ka l trung im ca KA). Tng t, ta chng minh c Kb Hb CA, Kc Hc AB
v M Kb Hb , M Kc Hc .
V Ha , Hb , Hc tng ng l i xng ca K qua trung im EF, F D, DE, nn 4DEF v
4Ha Hb Hc bng nhau (v v t), o vy kt hp vi Ha M k DK, Hb M k EK, Hc M k F K =
4DEF K, v 4Ha Hb Hc M bng nhau (v v t).
Gi P, Q l tm ng trn Euler v tm ngoi tip ca 4DEF . Gi Qa l i xng ca
k

Q qua EF v D l i xng ca D qua K. T KD = DK = M Ha = Ha M KD l


82

Tp ch Epsilon, S 09, 06/2016


mt hnh bnh hnh, nn D i xng M qua trung im ca EF , do P l trung im ca
k

DQa = 2P K = Qa D = QM . M M, Q l trung im ca KT, KN , ta suy ra 2M Q =


k

T N = 4P K = 2M Q = T N , do vy nu L HP OT ta c P K = N L = 3P K = LO
(v N O = N T ) = HO : HK = LO : P K = 3 : 1.

Ti liu tham kho


[1] C. Kimberling, Encyclopedia of Triangle Center, Part 1,
http://faculty.evansville.edu/ck6/encyclopedia/ETC.html
[2] HK passes through the circumcenter
http://www.artofproblemsolving.com/community/c6h1125044
[3] Geometry Problem (23)
http://www.artofproblemsolving.com/community/c6h373509
[4] Euler Lines Reflected
http://www.artofproblemsolving.com/community/c6h610293
[5] Taylor Circle
http://mathworld.wolfram.com/TaylorCircle.html
[6] Honsberger, R. , Episodes in Nineteenth and Twentieth Century Euclidean Geometry;
Washington, DC: Math. Assoc. Amer., 1995
[7] The Taylor Circle
http://www.cut-the-knot.org/triangle/Taylor.shtml
[8] De Longchamps point
http://mathworld.wolfram.com/deLongchampsPoint.html
[9] Exeter point
http://mathworld.wolfram.com/ExeterPoint.html

83

Tp ch Epsilon, S 09, 06/2016

84

NH L SAWAYAMA V THESBAULT TRONG


CC BI TON HNH HC THI OLYMPIC
Trn Quang Hng (T ton, trng THPT Chuyn KHTN, H Ni)
Dng nh Ngc (Lp 10 A1 Ton, trng THPT Chuyn KHTN, H Ni)

Bi vit tp trung vo vic pht trin cc ng dng nh l ni ting ca


Sawayama v Thbault trong cc bi ton hnh hc thi Olympic.

1. M u
nh l Thbault l mt trong nhng nh l p bc nht ca hnh hc phng. Nguyn liu ch
yu trong chng minh nh l ny l b ca Sawayama. V vy chng c gp chung thnh
tn gi l nh l Sawayama v Thbault xem [1,2,3]. B Sawayama c pht biu nh sau,
tham kho [2,3]
Bi ton 1 (B Sawayama). Cho tam gic ABC ni tip ng trn (O). D l mt im
thuc on BC. ng trn (K) tip xc DA, DC ln lt ti M, N v tip xc trong (O).
Chng minh rng M N i qua tm ni tip tam gic ABC.

A
F

K
M
O
I

85

Tp ch Epsilon, S 09, 06/2016


Chng minh. Gi phn gic gc BAC ct (O) ti E khc A. AE ct M N ti I. (K) tip xc
(O) ti F . D c E, N, F thng hng v EB 2 = EC 2 = EN.EF .
Ta li c F M N = 21 F KN = 12 F OE = F AE suy ra t gic AF IM ni tip. Suy
ra IF N = M F N M F I = DM N M F I = AF I M F I = AF M =
AIM = EIN . T 4EIN 4EF I suy ra EI 2 = EN.EF = EC 2 = EB 2 . Suy ra I
l tm ni tip tam gic ABC. Ta c iu phi chng minh.
nh l Thebault l mt h qu trc tip ca b trn
Bi ton 2 (nh l Thbault). Cho tam gic ABC ni tip ng trn (O). D l mt im
thuc on BC. ng trn (K) tip xc DA, DC v tip xc trong (O). ng trn (L) tip
xc DA, DB v tip xc trong (O). Chng minh rng KL i qua tm ni tip tam gic ABC.
Chng minh xem chi tit trong [1,4]. Sau y l mt pht biu khc ca b Sawayama trn t
gic ni tip
Bi ton 3. Cho t gic ABCD ni tip ng trn (O). Hai ng cho AC v BD ct nhau
ti E. ng trn (K) tip xc vi on EC, ED ti M, N v tip xc trong (O). Chng minh
rng M N i qua tm ni tip ca tam gic ACD v tam gic BCD.
Bi ton 4 (nh l Sawayama v Thbault m rng vi tm bng tip). Cho tam gic ABC
ni tip ng trn (O). D l mt im thuc tia i tia CB. ng trn (K) tip xc DA, DC
ln lt ti M, N v tip xc ngoi (O). Chng minh rng M N i qua tm bng tip ng vi
nh B ca tam gic ABC.
Chng minh. Gi phn gic ngoi ti nh A ct (O) ti E khc A. AE ct M N ti J. (K) tip
xc (O) ti F . D c E, N, F thng hng v EB 2 = EC 2 = EN.EF . Ta li c F M N =
1
F KN = 12 F OE = F BE = F AJ suy ra t gic AF M J ni tip. Suy ra EF J =
2
180 N F J = 180 N F M M F J = 180 JM A M AJ = M JA. T
4EF J 4EJN suy ra EJ 2 = EN.EF = EC 2 = EB 2 . Suy ra J l tm bng tip ng vi
nh B ca tam gic ABC. Ta c iu phi chng minh.
C hai pht biu trn u rt hay dng trong cc bi ton hnh hc khc nhau. Trn l mt vi bi
ton im qua nh l Sawayama v Thbault cng nh mt s cc m rng.

2. Mt s bi ton ng dng
nh l Sawayama v Thbault c th coi l mt b thng dng trong cc bi ton Olympic
kh, i khi vic dng n thng dng v hin nhin ti mc kh nhn ra vai tr ca n. Chng ta
hy bt u cc bi ton chn i tuyn Vit Nam 2014, tham kho [4]
d
Bi ton 5 (VNTST 2014). Cho tam gic ABC ni tip trong ng trn (O). Trn cung BC
khng cha A ca (O) ly im D. Gi s CD ct AB E v BD ct AC F . Gi (K) l
ng trn nm trong tam gic EBD, tip xc vi EB, ED v tip xc vi ng trn (O). Gi
(L) l ng trn nm trong tam gic F CD, tip xc vi F C, F D v tip xc vi ng trn
(O). Gi M l tip im ca (K) vi BE v N l tip im ca (L) vi CF . Chng minh rng
ng trn ng knh M N lun i qua mt im c nh khi D di chuyn.

86

Tp ch Epsilon, S 09, 06/2016

E
C

A
F

N
K
M
D

Chng minh. Gi (K) tip xc ED ti G v (L) tip xc F D ti H. Theo nh l Sawayama


v Thbault m rng cho tm bng tip th M G, N H i qua tm bng tip J ng vi nh A.
T theo tnh cht gc ca phn gic gc to bi (EK, LF ) = 21 (A + D) = 90 do
EK LF . Li d c M G EK LF N H t M G vung gc N H ti J nn ng
trn ng knh M N i qua J c nh.
Bi ton c th m rng th v hn nh sau, tham kho [4]
Bi ton 6. Cho tam gic ABC. Mt ng trn (O) bt k c nh i qua B, C. D l im di
chuyn trn (O) sao cho A, D khc pha BC. Gi s CD ct AB E v BD ct AC F . Gi
(K) l ng trn tip xc EB, ED ln lt ti M, N v tip xc trong (O). Gi (L) l ng
trn tip xc F C, F D ln lt ti P, Q v tip xc trong (O). Chng minh rng giao im ca
M N, P Q lun nm trn mt ng trn c nh khi D di chuyn.
Chng minh. Gi AB, AC ln lt ct (O) ti G, H khc B, C. p dng nh l Sawayama v
Thbault m rng vo tam gic GBC, HBC th M N i qua tm bng tip S ng vi nh G
ca tam gic BGC c nh v P Q i qua tm bng tip T ng vi nh H ca tam gic HBC
c nh. Gi M N ct P Q ti R ch EK M N, F L P Q, theo tnh cht gc ca phn gic
th 180 (M N, P Q) = (EK, F L) = 21 (A + D). V (O) c nh nn D khng i,
t R khng i v S, T c nh nn R thuc ng trn c nh i qua S, T .
87

Tp ch Epsilon, S 09, 06/2016

N
L
M

H
G
F
E

Cc bn hy lm hai bi ton sau luyn tp thm v dng ny


d cha
Bi ton 7. Cho tam gic ABC ni tip ng trn (O). P l im di chuyn trn cung BC
A ca (O). P B, P C ct CA, AB ln lt ti E, F . ng trn (K) tip xc on EA, EB v
d khng cha A ca
tip xc trong (O). ng trn (L) tip xc on F B, F C v tip xc BC
(O). (K) tip xc AC ti M v (L) tip xc AB ti N . Chng minh rng ng trn ng
knh M N lun i mt im c nh khi P di chuyn.
Bi ton 8. Cho tam gic ABC v ng trn (O) c nh i qua B, C. D l im di chuyn
d ca (O) sao cho D, A cng pha BC. DB, DC ct CA, AB ln lt ti E, F .
trn cung BC
ng trn (K) tip xc on EA, EB ti M, N v tip xc trong (O). ng trn (L) tip
xc on F B, F C ti P, Q v tip xc (O) ti mt im khng cng pha A so vi BC. Chng
minh rng giao im ca M N v P Q lun thuc mt ng trn c nh khi P di chuyn.
Bi ton tip sau tham kho [1,5]
Bi ton 9. Cho t gic ABCD ni tip ng trn (O) c AC ct BD ti E. ng trn (K)
tip xc on EA, ED v tip xc trong (O). ng trn (L) tip xc on EB, EC v tip
d v
xc trong (O). Chng minh rng trc ng phng ca (K) v (L) chia i cc cung AB
d ca (O).
CD
88

Tp ch Epsilon, S 09, 06/2016

G
H
A

P
M
K

N
E
T

Chng minh. Gi (K) tip xc AC, BD ti M, N . (L) tip xc AC, BD ti P, Q. Theo nh l


Sawayama v Thbault th M N i qua tm ni tip I ca tam gic DAB v P Q i qua tm ni
d v CD
d ca (O). Ta d chng
tip J ca tam gic CAB. Gi S, T l trung im cc cung AB
minh ST song song vi phn gic AEB v vung gc vi phn gic BEC. Cng d chng
minh t gic AIJB ni tip nn IJ ct EA, EB to thnh mt tam gic cn nn IJ ST . Hn
na DI, CJ i qua S v SI = SA = SB = SJ. T tam gic SIJ cn v ST IJ nn
ST chia i IJ. Trong hnh thang M N P Q c ST k M N k P Q v ST chia i IJ nn ST l
ng trung bnh nn ST i qua trung im ca M P, N Q, vy ST chnh l trc ng phng
ca (K) v (L). l iu phi chng minh.
Bi ton tip sau l mt h qu ca bi trn
Bi ton 10. Cho t gic ABCD ni tip ng trn (O) c AC ct BD ti E. ng trn
(K) tip xc on EA, ED v tip xc trong (O). ng trn (L) tip xc on EB, EC v
tip xc trong (O). Chng minh rng c mt tip tuyn chung ngoi ca (K) v (L) song song
AB.
Chng minh. Gi tip tuyn chung ngoi ca (K) v (L) gn AB hn ct (O) ti M, N . (K), (L)
tip xc (O) ti P, Q v tip xc M N ti G, H. Theo chng minh ca nh l Sawayama v
89

Tp ch Epsilon, S 09, 06/2016

S
Q
A

I
E

N
O

A
G

P
K

E
O

d
Thbault th P Q, QH i qua trung im S ca cung M
N . Li c SG.SP = SM 2 = SN 2 =
SH.SQ. T S thuc trc ng phng ca (K) v (L). Theo bi trc S l trung im
d Vy cung M
d
d c chung trung im S nn M N k AB. Ta c iu phi chng
cung AB.
N , AB
minh.
Bi ton trn li c mt h qu th v khc nh sau

90

Tp ch Epsilon, S 09, 06/2016


Bi ton 11. Cho tam gic ABC ni tip ng trn (O). D l mt im thuc on BC.
ng trn (K) tip xc DA, DC v tip xc trong (O). ng trn (L) tip xc DA, DB v
d khng cha
tip xc trong (O). Tip tuyn chung trong khc AD ca (K) v (L) ct cung BC
A ca (O) ti E. Chng minh rng DAB = EAC.
Chng minh. Gi AD ct (O) ti F khc A. Tip tuyn chung trong khc AD ca (K) v (L)
ct (O) ti G khc E. p dng bi trc vo t gic AEF G, ta thy BC l tip tuyn chung
ngoi ca (K) v (L) gn EF hn nn BC k EF . T d thy DAB = EAC. Ta c iu
phi chng minh.

G
K
O
L
B

Chng ta thu c mt h qu n gin ca bi ton trn nh sau


Bi ton 12. Cho tam gic ABC ni tip ng trn (O) vi ng cao AH. ng trn (K)
tip xc HA, HC v tip xc trong (O). ng trn (L) tip xc HA, HB v tip xc trong
(O). Chng minh rng tip tuyn chung trong khc AH ca (K) v (L) i qua im i tm ca
A trn (O).
Bi ton sau c mt phn l chn i Romani nm 2006 v b sung thm mt iu kin tham
kho [6,7,8], li gii sau da vo tng ca Telv Cohl trong [6]
Bi ton 13. Cho tam gic ABC ni tip ng trn (O) vi ng cao AH. ng trn (K)
tip xc HA, HC v tip xc trong (O). ng trn (L) tip xc HA, HB v tip xc trong
(O). Chng minh rng cc iu kin sau l tng ng
1) AB + AC = 2BC.
2) Tip tuyn chung trong khc AH ca (K) v (L) chia i BC.
3) KL i qua trc tm tam gic ABC.

91

Tp ch Epsilon, S 09, 06/2016

K
I

L N
B

Chng minh. Nh bi ton trn ta thy cc iu kin 2) v 3) l tng ng. Ta s chng minh
1) v 3) tng ng l xong, tht vy. Gi (K) tip xc HC, HA ti P, Q. Gi (L) tip xc
HB, HA ti M, N . Theo nh l Sawayama v Thbault th M N, P Q, KL ng quy ti tm ni
tip I ca tam gic ABC. D thy cc tam gic HM N v HP Q vung cn nn M I l phn gic
LM H v P I l phn gic KP H. T d c cc on thng bng nhau IL = IH = IK
suy ra I l trung im KL. Gi D l hnh chiu ca I ln BC th D l trung im M P . Gi tip
tuyn chung trong khc AH ca (K) v (L) ct BC ti R. Theo tnh cht tip tuyn ct nhau
ca hai ng trn ngoi nhau d c HM = RP nn D l trung im HR. Gi AI ct (O) ti
E khc A. Ta thy R l trung im BC khi v ch khi ER BC khi v ch khi I l trung im
AE. Theo h qu nh l Ptolemy iu ny tng ng AB + AC = 2BC. Ta c iu phi
chng minh.
Nhn xt 2.1. Vi tng tng t li gii trn bn c c th xut mt s vn tng qut
hn.
Bi ton tip tc sau xut hin trong k thi Olympic chuyn KHTN nm 2015 tham kho [9]
Bi ton 14. Cho t gic ABCD ni tip ng trn (O). Gi I, J l tm ni tip tam gic
BAD, CAD. Gi DI, AJ ln lt ct (O) ti S, T khc D, A. ng thng IJ ln lt ct
AB, CD ti M, N .
a) Chng minh rng SM v T N ct nhau trn ng trn (O).
b) Gi ng trn ngoi tip tam gic ABN ct CD ti P khc N . ng trn ngoi tip tam
gic CDM ct AB ti Q khc M . Chng minh rng P Q i qua tm ni tip hai tam gic ABC
v DBC.
Chng minh. a) D thy tam gic SAI v T DJ cn v c ASI = DT J nn hai tam gic
dng dng. Li d chng minh t gic AIJD ni tip nn M AI = IAD = DJN v
92

Tp ch Epsilon, S 09, 06/2016

U
L

I
O

K
J

N DJ = JDA = AIM . T hai tam gic M AI v N JD ng dng. T suy ra SM A


v T N J ng dng. Vy ASM = N T J do SM v T N ct nhau ti E trn ng trn
(O).
b) Gi AB ct CD ti G. GE ct (O) ti F khc E. Ta thy GC.GD = GE.GF = GM.GQ. T
t gic M QF E ni tip nn QF E = AM E = M AS +M SA = M BS +AF E =
SF A + ASE = EF S. T S, Q, F thng hng. Tng t T, P, F thng hng. T chng
minh trn SM A v T N J ng dng nn tam gic GM N cn suy ra GM = GN . Li c
GM.GQ = GN.GP nn GP = GQ suy ra P Q k M N k ST . T ng trn ngoi tip tam
gic F P Q tip xc (O). Vy theo nh l Poncelet nu P Q ct DB, AC ti U, V th ng trn
ngoi tip tam gic F U V cng tip xc (O) v tip xc DB, AC. T theo nh l Sawayama
v Thebaut th P Q i qua tm ni tip hai tam gic ABC v DBC.
Nhn xt. y l bi ton s dng hai b quan trng l nh l Sawayama v Thebaut v nh
l Poncelet. Ch rng vi nh l Sawayama v Thebaut th hiu mt cch cht ch phi pht
biu trn ng cho ca t gic ni tip, do vic s dng nh l Poncelet a v ng
trn F U V tip xc vi CA, BD l cn thit. Chng ti xin nhc li nh l Poncelet
Bi ton 15 (nh l Poncelet). Cho t gic ABCD ni tip ng trn (O). ng trn (K)
tip xc AB, CD ti M, N . ng trn (L) tip xc vi AC, BD ti P, Q. Chng minh rng
nu M, N, P, Q thng hng th (K), (L) v (O) ng trc.
Chng minh chi tit v m rng nh l ny xem trong [10]. Bi ton gc c th pht biu gn
li ch cn mt nh sau
Bi ton 16. Cho t gic ABCD ni tip ng trn (O). Gi I, J l tm ni tip tam gic
BAD, CAD. ng thng IJ ln lt ct AB, CD ti M, N . Gi ng trn ngoi tip tam
gic ABN ct CD ti P khc N . ng trn ngoi tip tam gic CDM ct AB ti Q khc
M . Chng minh rng P Q i qua tm ni tip hai tam gic ABC v DBC.
y l mt trong nhng khai thc th v ca nh l Sawayama v Thebaut. Khai thc ny hon
ton c th vit li trn cc ng cho nh sau
93

Tp ch Epsilon, S 09, 06/2016


Bi ton 17. Cho t gic ABCD ni tip ng trn (O). Gi I, J l tm ni tip tam gic
BAD, CAD. ng thng IJ ln lt ct AC, BD ti M, N . Gi ng trn ngoi tip tam
gic ACN ct BD ti P khc N . ng trn ngoi tip tam gic BDM ct AC ti Q khc M .
Chng minh rng P Q i qua tm bng tip gc A v D ca hai tam gic ABC v DBC.
Bi ton c mt m rng khc n gin nhng th v nh sau
Bi ton 18. Cho t gic ABCD ni tip ng trn (O). Gi S, T l trung im cc cung nh
d CD.
d M, N ln lt thuc AB, CD. Gi ng trn ngoi tip tam gic ABN ct CD ti
AB,
P khc N . ng trn ngoi tip tam gic CDM ct AB ti Q khc M . Chng minh rng SM
v T N ct nhau trn (O) khi v ch khi SQ v T P ct nhau trn (O).
Chng ta s chuyn sang mt vn tip theo, y l mt bi ton quan trng lin quan ti nhiu
bi ton khc, tham kho [21,22,23]
Bi ton 19. Cho t gic ABCD ni tip ng trn (O). Hai ng cho AC v BD ct nhau
ti E. ng trn (K) tip xc vi on EC, ED v tip xc trong (O) ti P . Chng minh rng
phn gic CP D i qua tm ni tip tam gic ECD v phn gic AP B i qua tm ni tip
tam gic EAB.

B
A
T
E

O L

I
K
C

Chng minh. Gi (K) tip xc on EC, ED ti M, N . Gi J, L l tm ni tip tam gic ACD


v BCD. Theo nh l Sawayama v Thebaut th M N i qua J, L. Hn na cc t gic P CM L
v P DN J ni tip. Gi ng trn ngoi tip hai t gic ny ct nhau ti I khc P . Ta c
DIP = DN P = N M P = LCP . T D, I, L thng hng. Tng t C, I, J thng
hng nn I l tm ni tip tam gic ECD. T d c DP I = IN E = IM E = CP I
94

Tp ch Epsilon, S 09, 06/2016


nn P I l phn gic CP D. Xt tam gic EAB c ng trn (O) qua A, B v (K) tip xc
EA, EB v tip xc trong (O) ti P . Theo phn va chng minh th phn gic AP B i qua
tm ni tip tam gic EAB.
Nhn xt 2.2. Bi ton c nhiu bin th khc nhau v d nh sau
Bi ton 20. Cho tam gic ABC mt ng trn (K) qua B, C v ct cnh CA, AB. ng
trn (L) tip xc on AB, AC v tip xc ngoi (K) ti P . Chng minh rng phn gic BP C
i qua tm ni tip tam gic ABC.
Cch chng minh cc bin th ny cc bn c th p dng mt cch hon ton tng t cch
chng minh bi ton trn nhng vi cc m rng ca nh l Sawayama v Thebaut cho cc tm
bng tip v tip xc ngoi. Bi ton trn dn ti mt bi ton ng dng rt p ca nh l
Sawayama v Thebaut ngh bi, Vladimir Zajic tham kho [11]
Bi ton 21. Cho tam gic ABC ni tip ng trn (O). P l mt im trn cnh BC, AP ct
(O) ti Q khc A. Gi (O1 ) l ng trn tip xc trong vi (O) v tip xc vi hai cnh P A,
P B. (O2 ) l ng trn tip xc trong vi (O) v tip xc P C, P A. Gi I1 , I2 l tm ng
trn ni tip cc tam gic P BQ, P CQ. Chng minh rng O1 O2 , I1 I2 , BC ng quy.

A
T2
O2
O
T1

O1
P

B
I1

I2

X1
Q

X2

Chng minh. Gi (O1 ), (O2 ) tip xc (O) ti T1 , T2 . Theo chng minh bi trc th phn gic
d
CT1 Q i qua tm I2 l tm ni tip tam gic P QC v i qua X2 l im chnh gia cung QC.
Tng t phn gic BT2 Q i qua tm I1 l tm ni tiptam gic P QB
 v i qua X1 l im
T
B
X
1
1
d T p dng Pascal cho su im
chnh gia cung QB.
thu c BC ct T1 T2
C T2 X2
ti S thuc I1 I2 . Mt khc d thy S thuc O1 O2 , nn ta c iu phi chng minh.
Nhn xt. S dng hng iu ha, ta cng d chng minh c O1 I1 , O2 I2 v AP ng quy.
Bi ton c tc gi m rng hn na, tham kho [12]
95

Tp ch Epsilon, S 09, 06/2016


Bi ton 22. Cho tam gic ABC ni tip ng trn (O) vi P, Q l hai im trn on
BC. AP, AQ ct (O) ti M, N khc A. Gi (I1 ), (I2 ), (I3 ), (I4 ) l ng trn ni tip cc tam
gic P AB, QAC, QAB, P AC. Gi (J1 ), (J2 ), (J3 ), (J4 ) l cc ng trn ln lt tip xc
vi cc on P B, P M ; QC, QN ; QB, QN ; P C, P M v tip xc trong (O). Chng minh rng
I1 I2 , I3 I4 , J1 J2 , J3 J4 v BC ng quy.
Li gii chi tit bi trn cc bn c th tham kho [12]. Bi ton sau tham kho trong [28]
Bi ton 23. Cho tam gic ABC nhn ni tip ng trn (O). ng trn (I) tip xc vi
cung nh AB v tip xc vi on AB ti Q. ng trn (J) tip xc vi cung nh AC v tip
xc vi on AC ti N . Tip tuyn chung ngoi khng ct on AQ, AN ca (I) v (J) tip
xc (I), (J) ln lt ti M, P . Chng minh rng M N, P Q ct nhau trn phn gic BAC khi
v ch khi M P k BC.

Q
P F

Chng minh. Gi M P ct (O) ti G, H v AK l ng cao ca tam gic AGH. Gi M N ct


P Q ti R. Theo nh l nh l Sawayama v Thbault th M N, P Q cng i qua tm ni tip ca
tam gic AGH nn R l tm ni tip ca tam gic AGH. T AR l phn gic GAH hay
cng l phn gic OAK do AO, AK ng gic trong GAH. T AR l phn gic BAC
khi v ch khi AO, AK ng gic trong BAC hay AK l ng cao ca tam gic AB. T
R nm trn phn gic BAC khi v ch khi M P k BC.
Qua mt s v d trn cc bn phn no thy c cc ng dng quan trng ca nh l Sawayama
v Thebaut trong cc bi ton thi Olympic.

3. Mt s bi ton lin quan


Phn ny ti xin ngh v su tp li mt s bi ton hay lin quan n ni dung ny c dn
link tham kho, cc bn hy t v hnh minh ha
Bi ton sau tham kho [13]
96

Tp ch Epsilon, S 09, 06/2016


Bi ton 24. Cho tam gic ABC ni tip (O) c P di chuyn trn cnh BC. ng trn (K)
tip xc P C ti M tip xc P A v tip xc trong (O). ng trn (L) tip xc P B ti N tip
xc P A v tip xc trong (O). AP ct (O) ti Q khc A. Chng minh rng ng trn ngoi
tip tam gic QM N lun i qua mt im c nh.
Li gii sau s dng tng ca Lym trong [13]

A'
X

F
I
Y

K
S

E
D'

M
J'

Li gii th nht. Gi X, Y l tip im ca (K) v (L) vi (O). E, F l tip im ca (K) v


(L) vi AP . (I) ni tip tam gic ABC v tip xc BC ti D. Gi ng trn A-Mixtilinear ca
tam gic ABC tip xc (O) ti J, D0 l tip im ca ng trn bng tip A vi BC, J 0 l
giao im ca AD0 vi (O). Theo tnh cht quen thuc ca ng trn Mixtilinear th JJ 0 k BC,
v vy theo tnh cht i xng th JD i qua A0 i xng A qua trung trc BC, nn P DJQ ni
d khng cha A nn Y N M X
tip. Mt khc v Y N, XM cng i qua im chnh gia cung BC
ni tip.H P S LK ti S, p dng nh l Sawayama v Thbault ta c K, L, I thng hng v
N IM = N SM = 90 . Theo tnh cht ca hai ng trn cng tip xc ng trn th ba,
ta c BC, XY, KL ng quy ti T , do T P.T D = T S.T I = T N.T M = T Y.T X = T B.T C
suy ra P Y XD ni tip. Kt hp vi XY QJ, P DJQ ni tip, ta chng minh c QJ i qua T ,
thu c T Q.T J = T Y.T X = T N.T M suy ra (QM N ) i qua J c nh.
tng li gii th nht kh trc tip v t nhin, tuy nhin vi li gii th hai ta nhn thy bi
ton thc cht ch l mt trng hp c bit ca mt kt qu rt p c xut bi Vladimir
Zajic, tham kho [26]
97

Tp ch Epsilon, S 09, 06/2016


B 14. Gi K l mt im bt k trn ng trn ngoi tip tam gic ABC, X bt k thuc
ng thng AK. T X k hai tip tuyn ti ng trn ni tip (I) ca tam gic ABC, ct BC
ln lt ti Y, Z. Khi (KY Z) i qua tip im J ca ng trn A-Mixtilinear ca tam gic
ABC vi (O).
Li gii sau da trn tng ca Jean-Louis tham kho [27]

L
A

I
F
Y
B

D
K
J

E
X

Ia

d khng cha A ca (O). Gi Ia l tm bng tip A


Chng minh. Gi s K nm trn cung BC
d khng cha A ca (O), T l giao im ca AX
ca tam gic ABC, E l im chnh gia BC
v BC. Ia T giao (BIC) ti F , IX giao Ia F ti D. T T K.T A = T B.T C = T F.Ia T suy ra
A, F, K, Ia cng thuc cng mt ng trn. Kt hp vi tnh cht quen thuc ca ng trn
Mixtilinear IJE = 90 , ta c IJK + IF K = IJE EJK + IF Ia + KF Ia =
90 EAK + 90 + KAIa = 180 nn JKF I ni tip. M IF CB, BCKJ ni tip, ta thu
c KJ, BC, IF ng quy ti S. Mt khc, gi XY ct AB ti L, p dng nh l Desargues
cho tam gic LY B v tam gic IDIa vi LB giao IIa ti A, LY giao ID ti X, BY giao Ia D
98

Tp ch Epsilon, S 09, 06/2016


ti T , v A, X, T thng hng nn LI, BIa , Y D ng quy ti tm bng tip L ca tam gic
LBY . Do IY D = 90 . Chng minh tng t ta c IZD = 90 , d thy I, F, Y, Z, D
thuc cng mt ng trn. Theo tnh cht phng tch th SY.SZ = SI.SF = SK.SJ, vy
Z, Y, K, J thuc cng mt ng trn.

F
I

L
E
Z
B

Li gii th hai. Gi D l tip im ca ng trn ni tip (I) vi BC. K tip tuyn N Z,


M Y ti (I). V tam gic P N F cn nn d thy ZIN + N F P = N ID + P N F = 90 ,
suy ra N Z song song vi AP . Tng t ta c M Y k AP k N Z do c th coi giao im ca
M Y v N Z nm trn AP . p dng b trn th (QM N ) i qua im c nh l tip im
ng trn A-Mixtilinear ca tam gic ABC vi (O).
Bi ton sau tham kho [14]
Bi ton 25. Cho tam gic ABC ni tip ng trn (O). P l mt im thuc on BC. ng
trn (K) tip xc P A, P C v tip xc trong (O) ti M . ng trn (L) tip xc P A, P B v
tip xc trong (O) ti N .
a) Chng minh rng M N lun i qua mt im c nh khi P thay i.
b) Chng minh rng ng trn ngoi tip tam gic P M N lun i qua mt im c nh khi P
thay i.
99

Tp ch Epsilon, S 09, 06/2016

L
J
S

N
B

D P

Chng minh. a) p dng nh l Sawayama v Thbault ta c KL i qua tm ni tip I ca


tam gic ABC. Theo tnh cht ca hai ng trn cng tip xc vi ng trn th ba, d thy
KL, BC, M N ng quy ti S. Gi OI ct M N ti J. p dng nh l Menelaus cho tam gic
R(I) R(L)
R(I)
JI
SI N L
IOL ct tuyn N JS ta c JO
= SL
. N O = R(L)
. Vy M N lun i qua J c nh
. R(O) = R(O)
vi J l ng gic ca im Nagel v cng l tm v t ngoi ca (O) v (I).
b) Gi D l hnh chiu ca I ln BC, t kt qu cch chng minh th nht ca Bi ton 23 ta c
P DN M ni tip, suy ra (P N M ) i qua D c nh.
Bi ton sau tham kho [15,16]
Bi ton 26 (Bulgaria 2010). Cho tam gic ABC ni tip ng trn (O). P l mt im thuc
on BC. ng trn (K) tip xc P A, P C v tip xc trong (O). ng trn (L) tip xc
P A, P B v tip xc trong (O). Chng minh rng bn knh ca (K) v (L) bng nhau khi v ch
khi P l tip im ca ng trn bng tip gc A vi BC.
Chng minh. Gi I l tm ni tip tam gic ABC, AP giao (O) ti D, S l im chnh gia
d khng cha A ca (O). H IQ BC ti Q. (K), (L) tip xc BC ti R, T v tip
cung BC
xc O ti M, N , d thy S, T, N thng hng, S, R, M thng hng. Tip tuyn chung trong khc
AP ca (K) v (L) ct (O) ti E, F sao cho E, D cng pha vi BC. Theo kt qu Bi ton
11 d thy AE, AP l hai ng ng gic. Ta c P l tip im ca ng trn bng tip gc
A vi BC khi v ch khi E l tip im ng trn A-Mixtilinear ca tam gic ABC vi (O)
hay tng ng vi T RDE ni tip. V DE k BC nn iu ny tng ng vi T i xng
R qua trung im BC. Ch ng thc ST.SN = SR.SM nn gi thit trn tng ng vi
ST
SR
OL
OK
= SM
hay ON
= OM
. iu ny xy ra khi v ch khi (K) v (L) c cng bn knh.
SN
Bi ton sau tham kho [17]
100

Tp ch Epsilon, S 09, 06/2016

O
L

I
N

M
B

E
S

Bi ton 27. Cho tam gic ABC ni tip ng trn (O). Hai ng trn lun trc giao i qua
A v I ct (O) ti M, N khc A. Chng minh rng ng thng M N lun i qua im c nh
khi hai ng trn thay i.
B 15. Cho tam gic ABC vi ng trn ngoi tip (O), tm ni tip I. P l mt im
thuc on BC. ng trn (K) tip xc P A, P C v tip xc trong (O) ti M . ng trn (L)
tip xc P A, P B v tip xc trong (O) ti N . Khi (AIN ) trc giao vi (AIM ).
Chng minh b . Gi (K), (L) tip xc BC ti X, Y . K tip tuyn chung trong khc AP
ca (K) v (L) ct (O) v BC ln lt ti A0 , Q sao cho A, A0 cng pha i vi BC. K
P S KL, theo cch chng minh th nht ca Bi ton 24 ta c LK, N M v BC ng quy
ti T th T S.T I = T N.T M = T B.T C nn ng trn ngoi tip tam gic BIC i qua S. Mt
khc gi S 0 l tm ni tip tam gic A0 BC th S 0 thuc (BIC) v theo nh l Thbault, S 0 cng
thuc KL, suy ra S 0 trng S. V P S KL nn IQ KL. Gi J l giao im hai tip tuyn ca
(O) ti M, N . D thy J thuc trc ng phng ca (K) v (L), kt hp vi d kin IQ k SP
v Y P = QX suy ra ng thng qua J vung gc KL chia i cc on thng XY, QP v
IS. M T S.T I = T B.T C = T N.T M suy ra I, S, M, N thuc mt ng trn, nn J l tm
(N SIM ). T AM I + AN I = M IN M AN = 180 21 M JN JM N = 90 ,
suy ra (AN I) trc giao vi (AM I).
Chng minh. p dng b trn d thy lun tn ti (K) v (L) cng ng thi tip xc BC
v (O) sao cho mt tip tuyn chung trong ca chng i qua A. p dng kt qu bi ton 25 ta
c M N lun i qua tm v t ngoi ca (I) v (O) c nh.
Bi ton sau tham kho [18]
101

Tp ch Epsilon, S 09, 06/2016

A'

N
T

Bi ton 28. Cho lc gic lng tip ABCDEF vi tm ni tip I v tm ngoi tip O. khi
cc ng chp AD, BE, CF ng quy ti G. Gi J, K, L, M, N, P l tm ni tip cc
tam gic GAB, GBC, GCD, GDE, GEF, GF A. ng trn tip xc on GA, GB v tip
xc trong (O) ti X. Tng t c cc im Y, Z, T, U, V . Chng minh rng by ng thng
XM, Y N, ZP, T J, U K, V L v OI ng quy.
Bi ton sau tham kho [19]
Bi ton 29. Cho tam gic ABC c tm ngoi tip O v tm ni tip I. ng trn (K) tip
xc AB, AC ti E, F v tip xc ngoi ng trn ngoi tip tam gic OBC. Chng minh rng
EF chia i AI.
Chng minh. D thy (K) c xc nh duy nht nn, gi s EF chia i AI. Ta s chng
minh (K) tip xc (BOC). Gi D l giao im th hai ca (BOC) vi AB, S l im chnh
d khng cha B ca (O). SB ct (BOC) ti Q th BQC = BOC = 2BSC
gia cung AC
nn QS = QC = QD, suy ra S l tm bng tip B ca tam gic BDC. D thy EF i
qua S. Do theo dng o ca b Sawayama m rng cho tm bng tip th (K) tip xc
(BOC).
Bi ton sau tham kho [20]
102

Tp ch Epsilon, S 09, 06/2016

O
K
L

I
M

N
B

E
K
D

F
S

O
I

B
C

Bi ton 30. Cho ng trn (O1 ) tip xc ngoi (O2 ) ti T . P Q l mt dy cung ca (O1 ).
P T, QT ct (O2 ) ti R, S. Tip tuyn qua P ca (O1 ) ct (O2 ) ti A, B sao cho A nm gia
P, B. Tip tuyn qua Q ca (O1 ) ct (O2 ) ti D, C sao cho D nm gia Q, C. SA giao P Q ti
F , RD giao P Q ti E. Chng minh rng EAF = 21 BAC.
103

Tp ch Epsilon, S 09, 06/2016

C
S
P
A

E
O1

O2

D
Q

d khng cha B ca (O2 ). M theo b


Chng minh. D thy S l im chnh gia cung CD
Sawayama m rng cho tm bng tip th P Q i qua tm bng tip C ca tam gic CAD, suy
ra F l tm bng tip tam gic CAD. Tng t ta c E l tm bng tip B ca tam gic BAD.
T EAF = DAE DAF = 90 21 DAB 90 + 12 DAC = 12 BAC.
Bi ton sau tham kho [24]
Bi ton 31. Cho tam gic ABC vi trc tm H, v ng cao AD, BE. K thuc DE sao cho
DK = DH. ng thng qua K vung gc DE ct AD ti I. M l trung im BC. Chng
minh rng BM = M I + IK.

Li gii ca Luis Gonzlez. K ng cao CF ca tam gic ABC, ta c H l tm ni tip


tam gic DEF . ng trn tm I bn knh IK tip xc DF ti L, d thy LK k BC. Gi
J l giao im ca KL v F C, ta c F JL = F CB = F EB = HEK suy ra JHEK
ni tip. Mt khc gi P l giao im ca F D vi ng trn ng knh BC, ch rng
F EM D ni tip v tam gic DHK cn, ta c F JE = HKE = 90 + 12 HDK =
90 + 41 F M E = 90 + 21 F P E, v vy J l tm ni tip tam gic P F E. p dng dng o
ca b Sawayama cho tam gic P F E, v LK i qua J nn (I) tip xc (M ) ti T . Do
M B = M T = M I + IT = M I + IK.
Bi ton sau tham kho [25]
104

Tp ch Epsilon, S 09, 06/2016

T
F

I
H
L

K
J
C

B
D

Bi ton 32. Cho tam gic ABC ni tip ng trn (O). D l im trn on BC. ng trn
(K) tip xc DA, DC v tip xc trong (O). ng trn (L) tip xc DA, DB v tip xc trong
(O). Tip tuyn chung trong ca (K) v (L) khc AD ct BC ti S. Tip tuyn chung ngoi ca
(K) v (L) khc BC ct AD ti T . Chng minh rng ST i qua tm ni tip tam gic ABC.

Gi gii. Gi I l tm ni tip tam gic ABC, p dng nh l Thbault ta c L, K, I thng


hng. Theo cch chng minh B 27.1 th SI KL, m ST KL do S, I, T thng
hng.

Li kt v cm n
Bi vit ny mang mt tnh cht l h thng li, tp hp v gii thiu cc bi ton lin quan ti
nh l Sawayama v Thbault trong cc k thi Olympic nn cc ti liu tham kho c chng
ti ghi rt k. Chng ti mun ni li cm n ti bn Nguyn Tin Dng sinh vin K50 i hc
Ngoi thng gip chng ti c li bn tho v a ra cc nhn xt, gp gi tr.

105

Tp ch Epsilon, S 09, 06/2016

T
K
I
L
B

Ti liu tham kho


[1] Nguyn Th Hng, Lng nh Nguyt, Lng Th Thanh Mai, o Th Qunh Nga. nh
l Sawayama v Thbault
http://analgeomatica.blogspot.com/2014/02/inh-ly-sawayama-va-thebault.html
[2] Forum Geometricorum, Sawayama and Thebaults theorem, Jean-Louis Ayme
http://forumgeom.fau.edu/FG2003volume3/FG200325.pdf
[3] Sawayama Thebaults Theorem
http://www.cut-the-knot.org/triangle/SawayamaTheBault.pdf
[4] Xung quanh mt bi ton hnh hc trong k thi chn i tuyn Vit Nam nm 2014
http://analgeomatica.blogspot.com/2014/03/xung-quanh-mot-bai-toan-hinh-hoctrong.html
[5] Parallel tangent
http://artofproblemsolving.com/community/c6h15945
[6] Metric relation describing the position of the incenter
http://artofproblemsolving.com/community/c6h88823
[7] circles and circles
http://artofproblemsolving.com/community/c6h118385
[8] b+c=2a
http://artofproblemsolving.com/community/c6h210518
[9] Hai bi hnh hc thi Olympic chuyn KHTN 2015
http://analgeomatica.blogspot.com/2015/05/hai-bai-hinh-hoc-thi-olympic-chuyen.html
106

Tp ch Epsilon, S 09, 06/2016


[10] Topic 3 circles with common tangency point
http://www.artofproblemsolving.com/community/q1h474157p4809875
[11] Ordinary and Thebault incircles.
http://www.artofproblemsolving.com/community/c6h244007p1341565
[12] Five concurrent lines
http://www.artofproblemsolving.com/community/c6h485516p2720041
[13] From mixtilinear incircles to the Thebault circles
http://www.artofproblemsolving.com/community/c6h213098p1176147
[14] Maybe Thebault have ovelooked
http://www.artofproblemsolving.com/community/c6h570686p3635527
[15] Prove that M = D - [Bulgaria NMO 2010]
http://www.artofproblemsolving.com/community/c6h349730
[16] On a particular case of Thebaults theorem
http://www.artofproblemsolving.com/community/c6h201858
[17] Exsimilar center
http://www.artofproblemsolving.com/community/c6h617532p3683327
[18] Seven concurrent lines
http://www.artofproblemsolving.com/community/c6h623162p3729318
[19] middle
http://www.artofproblemsolving.com/community/c6h517026p2914609
[20] A little hard for me
http://www.artofproblemsolving.com/community/c6h221652
[21] Concyclic points with triangle incenter
http://www.artofproblemsolving.com/community/c6h41667
[22] incenter of triangle
http://www.artofproblemsolving.com/community/c6h407366
[23] Fairly difficult
http://www.artofproblemsolving.com/community/c6h6086
[24] Prove that
http://www.artofproblemsolving.com/community/c6h503957p2840426
[25] Again with Thebaults circles
http://artofproblemsolving.com/community/q2h1086479p4806266
[26] Cevian and mixtilinear incircle
http://www.artofproblemsolving.com/community/c6h209898p1156398
[27] A new mixtilinear incircle adventure III
http://jl.ayme.pagesperso-orange.fr/vol4.html
107

Tp ch Epsilon, S 09, 06/2016


[28] Mi tun mt bi ton: Tun 2 thng 6 nm 2016
http://analgeomatica.blogspot.com/2016/06/moi-tuan-mot-bai-toan-tuan-2-thang-6.html

108

V BI TON TAM GIC 80-80-20


L Phc L
(Thnh ph H Ch Minh)
cp THCS, cc bn hc sinh yu Ton u quen thuc v cng kh s khi gp bi ton tnh gc. Hu
ht cc bi nh th u i hi phi k thm ng ph, im ph mi x l c.
Trong cc bi nh th, c l bi ton sau l ni ting v cng ph bin nht:
Cho tam gic ABC c gc A, B, C ln lt l 20 , 80 , 80 . Trn cnh AC, AB, ln lt ly cc im
D, E sao cho
BCE = 60 , CBD = 50 .
Tnh s o gc CED.
Bi ton ny c bit n ln u tin vo nm 1909 do nh Ton hc Langland ca M gii thiu. Di
y chng ti s gii thiu mt s cch chng minh cho bi ton cng mt s vn lin quan.
Cc li gii chng ti c tham kho ti trang web:
www.cut-the-knot.org/triangle/80-80-20/Classical1.shtml

Cch 1. (y l cch ph bin nht)

Qua E, dng ng thng song song vi BC ct AC ti F. Gi N l giao im ca BF, CE.


Ta c tam gic BCN cn ti N c BCN = 60 nn l tam gic u. T suy ra EN F =
BN C = 60 nn tam gic EN F cng u.
109

Tp ch Epsilon, S 09, 06/2016


Do , ta c EF = EN .
Tam gic BCD c BCD = 80 , CBD = 50 nn ta tnh c BDC = 50 hay tam gic
BCD cn ti C. Do , CD = CB = CN hay tam gic CN D cn ti C. T , ta cng c
CN D = CDN = 80 , suy ra
DN F = 180 (EN F + CN D) = 40
Suy ra DF N = CDN DN F = 40 hay tam gic DN F cn ti F.
Do , ta c DF = DN .
Suy ra DE l trung trc ca N F hay ED l phn gic ca N F E.
Vy ta c CED = 12 CEF = 30 .
Cch 2.

Dng hnh bnh hnh BCHE, khi , ta c BE = CH. Trn on CE, ly im N sao cho tam
gic BCN u th BC = CN = BN .
110

Tp ch Epsilon, S 09, 06/2016


Ch rng tam gic BCD cn ti C nn BC = CD. Do , BN = CD. Ta c
N BE = CBE CBN = 80 60 = 20 .
Do CH k AB nn ta cng c HCF = 20 .
Suy ra BN E = CDH(c.g.c) v CHD = BEN = 40 .
Mt khc CHF = 80 nn HD chnh l phn gic ca CHF.
R rng ECF = HCF = 20 nn CF l phn gic ca gc HCE. Do , D chnh l tm
ng trn ni tip tam gic CHE. T y suy ra
1
CED = CEF = 30 .
2
Cch 3. (y c l l cch n gin nht cho bi ton ny)

Trn AB, ly im F sao cho CB = CF. Khi , ta cng c BCF = 20 v F CD = 60 .


Ch rng CB = CD nn CD = CF , tc l tam gic CDF cn C nn n cng u.
Do DF = CF.
Bng bin i gc, ta cng c F CE = F EC = 40 nn tam gic F EC cn ti F.
T y suy ra F C = F D = F E nn F chnh l tm ng trn ngoi tip tam gic CDE.
Vy DEC = 12 DF C = 30 .
Cch 4.
111

Tp ch Epsilon, S 09, 06/2016

Gi O l tm ng trn ngoi tip tam gic BDE. Ta c


DBE = 30 DOE = 60 ,
m OD = OE nn tam gic ODE u. Do : EO = ED.
Bng bin i gc, ta cng c DCE = 80 60 = 20 .
D thy OC l trung trc ca BCD nn OCB =

80
2

= 40 nn

OCE = 60 40 = 20 .
Hai tam gic CDE v COE c cnh CE chung, ED = EO v DCE = OCE.
Tuy gc ny khng nm xen gia 2 cnh tng ng, nhng do gc DEC, OEC u nhn
(nm trong gc DEO nn nh hn 60 ), v th CDE = COE v dn n
1
CED = CEO = DEO = 30 .
2
Cch 5. (ca Maria Gelband, mt hc sinh THPT)
112

Tp ch Epsilon, S 09, 06/2016

Trn tia phn gic gc BCD, ly im M sao cho ABM = 30 .


Khi , ta c M D = M B v M BD = ABM + ABD = 30 + 30 = 60 , do , tam
gic BDM u. Khi , ta cng c CE l phn gic ca gc DCM .
V BE l phn gic ca gc DBM nn n cng l trung trc ca DM.
Trong tam gic CDM , E l giao im ca phn gic gc DCM v trung trc ca DM nn E
thuc ng trn ngoi tip tam gic DCM.
Do ,
CED = CM D =

113

60
= 30 .
2

Tp ch Epsilon, S 09, 06/2016

114

S DNG TNG TCH PHN


TNH GII HN DY S
Nguyn Ti Chung
(Trng THPT Chuyn Hng Vng, Gia Lai)

1. Gii thiu
C nhiu phng php chnh thng tnh gii hn dy s, chng hn nh s dng nh ngha
gii hn dy s, s dng nh l Weiertrass, s dng dy con, s dng nh l Lagrange,... Chuyn
ny s trnh by mt phng php m nu bit vn dng n mt cch linh hot chng ta c th
gii c nhiu bi ton v gii hn dy s lin quan n tng. c bit khi kt hp vi nguyn l
kp th phng php ny c sc cng ph rt ln, gii quyt c nhiu bi ton hay, kh v tng
qut.

2. Ni dung
2.1. Mt s kin thc thng dng
C s ca phng php s dng tng tch phn l nh ngha tch phn xc nh sau y.
nh ngha 2.1 (nh ngha tch phn xc nh). Cho hm s y = f (x) xc nh trn on
[a; b]. Xt mt phn hoch (php chia) T bt k ca on [a; b], tc l chia on [a; b] thnh n
on bi cc im chia:
a = x0 < x1 < x2 < < xn1 < xn = b.
Trn mi on [xi1 ; xi ] (i = 1, 2, . . . , n) ta ly bt k mt im i [xi1 ; xi ] v gi i =
xi xi1 l di ca on [xi1 ; xi ]. t d = max {1 , 2 , . . . , n }. Khi
n
X

f (i )i = f (1 )1 + f (2 )2 + + f (n )n

i=1

gi l tng tch phn ca hm s f (x) trn on [a; b] ng vi php phn hoch T . Tng ny
ph thuc vo phn hoch T , s khong chia n v ph thuc vo cch chn im i . Nu tn ti
n
P
lim
f (i )i (l mt s xc nh), khng ph thuc vo php phn hoch on [a; b] v cch
d0 i=1

chn im i th gii hn ny gi l tch phn xc nh ca hm s f (x) trn on [a; b] v k


Zb
hiu l f (x)dx. Khi hm s f (x) c gi l kh tch trn on [a; b].
a

115

Tp ch Epsilon, S 09, 06/2016


Lu .
Nh vy nu hm s f (x) kh tch trn on [a; b] th mi php phn hoch T ca on
[a; b] v mi cch chn cc im k [xk1 ; xk ] (k l s nguyn dng), ta lun c:
Zb
f (x)dx = lim

d0

n
X

f (i )i .

i=1

Trong thc hnh, chng ta c bit ch n mt s lp hm kh tch n gin v d nhn


bit sau y:
Nu hm s y = f (x) lin tc trn on [a; b] th kh tch trn on .
Nu hm s y = f (x) b chn trn on [a; b] v ch c mt s hu hn im gin
on th kh tch trn on .
Nu hm s y = f (x) b chn v n iu trn on [a; b] th kh tch trn on .
Ch 2.1. Cho on [a; b] R. Khi vi mi > 0, > 0 sao cho + = 1, ta c
a + b [a; b] .

2.2. Tnh gii hn ca dy s Sn =

n
P

xi

i=1

C rt nhiu bi ton gii hn dy s m vic tnh tng trc tip Sn rt kh khn hoc khng th
thc hin c. Tuy nhin c th bin i tng ny v dng tng tch phn (ca hm f (x) trn
on [a; b] ng vi mt php phn hoch T no ). Vy tnh gii hn ca mt tng nh tch
phn xc nh ta thng tin hnh theo cc bc nh sau:
Bc 1: Bin i tng Sn thnh biu thc
n

Sn =

baX
ba
f (a + i
).
n i=1
n

Bc 2: Chn c hm f (x) kh tch trn on [a; b].


Lp php phn hoch u trn [a; b] bi cc im chia
a = x0 < x1 < x2 < < xn1 < xn = b.
Lp tng tch phn ca hm f (x) trn [a; b]:
n

baX
ba
f (a + i
).
Sn =
n i=1
n
Zb
Bc 3: Khi lim Sn =

f (x)dx.

n+

116

Tp ch Epsilon, S 09, 06/2016


c bit: Nu a = 0 v b = 1 th cc bc trn tr thnh:
Bc 1: Bin i tng Sn thnh:
  
 
 
n
 n  1 X
1
2
i
1
f
+f
+ + f
=
f
.
Sn =
n
n
n
n
n i=1
n
Bc 2: Ch ra hm f (x) kh tch trn on [0; 1].
Lp php phn hoch u trn [0; 1] bi cc im chia
0 = x0 < x1 < x2 < < xn1 < xn = 1.
Lp tng tch phn ca hm f (x) trn [0; 1]:
n

Sn =

1X i
f ( ).
n i=1 n

Z1
Bc 3: Khi lim Sn =

f (x)dx.

n+

Bi ton 2.1. Cho Sn =

1
2
n
+ 2 + + 2 . Tm lim Sn .
2
n+
n
n
n

Li gii. Ta c:
1
Sn =
n

1
2
n
+ + +
n n
n

1Xi
=
.
n i=1 n

Xt hm s f (x) = x kh tch trn [0; 1]. Chia on [0; 1] bi cc im chia xi =

i
,i =
n

0, 1, . . . , n , tc l chia on [0; 1] thnh n on u nhau:


x0 =

0
1
2
n1
n
= 0 < x1 = < x2 = < < xn1 =
< xn = = 1.
n
n
n
n
n

i
Chn i = [xi1 ; xi ], i = 1, 2, . . . , n. Khi tng tch phn ca hm s f (x) trn [0; 1]
n
ng vi php phn hoch ni trn l


n
1 1
2
n
1Xi
Sn =
+ + +
=
.
n n n
n
n i=1 n
Ta c
n

lim Sn = lim

n+

n+

1Xi
n i=1 n

Z1

!
=

1
x2
1
xdx =
= .

2 0 2

i
Lu . Trong bi ton 2.1 ny, ta phi chn i = trng vi u mt phi ca on [xi1 ; xi ],
n
i
cho vi f (x) = x th f (i ) = .
n
117

Tp ch Epsilon, S 09, 06/2016


Bi ton 2.2 (Olympic ton Sinh vin ton quc nm hc 2007-2008). Tm lim Sn bit
n+

12008 + 22008 + + n2008


Sn =
.
n2009
Li gii. Ta c:
1
Sn =
n

" 
#
 2008
n  2008
2008
 n 2008
1
2
1X i
+
+ +
=
.
n
n
n
n i=1 n

Xt hm s f (x) = x2008 . Khi f (x) kh tch trn on [0; 1]. Chia u on [0; 1] bi cc
i
im chia xi = (i = 0, 1, . . . , n) v chn
n
i =

i
[xi1 ; xi ], i = 1, 2, . . . , n.
n

Khi tng tch phn ca hm s f (x) trn [0; 1] l


Z1
lim Sn =

Z1
f (x)dx =

n+

x
0

n
1P
i
f ( ) = Sn . Vy
n i=1 n

2008

1
x2009
1
dx =
=
.

2009 0 2009

Bi ton 2.3. Chng minh rng gii hn

2
n

sin
sin
sin

n+1 +
n + 1 + +
n+1
lim

n+
1
2
n
l mt s dng.
(
Li gii. Xt hm s f (x) =

sin x
nu x (0; ]
x
1
nu x = 0.

sin x
= 1 = f (0) nn hm s f lin tc trn on [0; ], do hm s f
x0
x0
x
kh tch trn on [0; ]. Hn na:
V lim+ f (x) = lim+

Z
f (x)dx > 0 (do f (x) > 0, x [0; ]) .
0

Nh vy:

i
i
n sin
n sin
X
n + 1 = lim X
n+1
lim

i
n+
n+
i
n
+
1
i=1
i=1
n+1
118

Tp ch Epsilon, S 09, 06/2016


n

= lim

n+

X
f
n + 1 i=1

i
n+1

!

Z
=

f (x)dx > 0.
0

Ta c iu phi chng minh.


Bi ton 2.4. Tnh lim Sn , vi:
n+

Sn =

2
n+
3

1
8
n+
3

1
14
n+
3

+ +

1
, n = 1, 2, . . .
6n 4
n+
3

Phn tch. Ta bin i:


n

1 X
Sn = .c
c i=1

1
1 cX
1
= .
,
6i 4
6i
4
c n i=1
n+
1+
3
3n

1
1+x
v on ly tch phn s l [d; d + c]. Xt php phn hoch u on [d; d + c] bi cc im chia
vi c s l di ca on ly tch phn. Ta d on hm s di du tch phn s l f (x) =

x0 = d, x1 = d +
Khi i = xi xi1 =

c.1
c.2
c.n
, x2 = d +
, . . . , xn = d +
= d + c.
n
n
n

c
, i = 1, 2, . . . , n. Gi s i [xi1 ; xi ], suy ra i c dng:
n

i = axi1 + bxi , vi a > 0, b > 0, a + b = 1.


6i 4
= axi1 + bxi . Tc l tm a v b sao cho
3n




4
c(i 1)
2i
ci
(a + b)ci ac

=a d+

+ d(a + b).
+b d+
=
n
3n
n
n
n
n

Ta cn tm a v b sao cho:

Suy ra:

(a + b)c = 2
4
ac =

d(a + b) = 0

c=2
do a+b=1
2
1

a= , b=

3
3

d = 0.

Bi vy ta c li gii nh sau.
Li gii. Ta c:
Sn =

n
X
i=1

1
1 2
=
6i 4
2 n
n+
3
119

n
X
i=1

.
6i 4
1+
3n

Tp ch Epsilon, S 09, 06/2016


1
trn on [0; 2]. R rng f (x) lin tc trn [0; 2] nn n kh tch trn
1+x
on . Xt php phn hoch u on [0; 2] bi cc im chia
Xt hm s f (x) =

2.1
2.2
2.(n 1)
2.n
, x2 =
, . . . , xn1 =
, xn =
= 2.
n
n
n
n
2
= , i = 1, 2, . . . Xt:
n
2
1
i = xi1 + xi [xi1 ; xi ].
3
3

x0 = 0, x1 =
Khi i = xi xi1

Khi :

6i 4
2 2 (i 1) 1 2i
+ . =
, f (i ) =
i = .
3
n
3 n
3n

1
.
6i 4
1+
3n

Suy ra:

n
1
1X
=
f (i )i .
6i 4 2
i=1 1 +
i=1
3n
y chnh l tng tch phn ca hm s f (x) trn on [0; 2] (ng vi php phn hoch on
[0; 2] v php chn im i nh ni trn). Do theo nh ngha tch phn ta c

1 2
Sn =
2 n

n
X

1
lim Sn =
n+
2

Z2
f (x)dx =

1
ln 3 = ln 3.
2

1k + 3k + + (2n 1)k
Bi ton 2.5. Tnh lim
n+
nk+1
Li gii. Ta c:
!
1 1k + 3k + + (2n 1)k
1k + 3k + + (2n 1)k
=
nk+1
n
nk
!
 k  k

k
2k
1
3
2n 1
=
+
+ +
.
n
2n
2n
2n


2i 1
1 i1
i
Do
=
+
l trung im ca on
2n
2
n
n


i1 i
;
, i = 1, 2, . . . , n
n n
 k  k

k 
k !
2k
1
3
2n 3
2n 1
nn
+
+ +
+
l tng tch phn ca hm
n
2n
2n
2n
2n
s f (x) = 2k .xk trn on [0; 1] ng vi php phn hoch u on [0; 1] bi cc im chia
i
2i 1
xi = (i = 0, 1, . . . , n) v chn i =
l trung im ca on
n
2n
[xi1 ; xi ], i = 1, 2, . . . , n.
120

Tp ch Epsilon, S 09, 06/2016


Nh vy:
1k + 3k + + (2n 1)k
lim
=
n+
nk+1

Z1
0

1
xk+1
2k
.
2 .x dx = 2
=
k + 1 0 k + 1
k

Bi ton 2.6. Tnh lim Sn , vi:


n+

Sn = 2n

n1
X
i=1

1
, n = 0, 1, 2, . . .
(2i + 1)2 + 4n2

1
, ta c
Li gii. Xt hm s f (x) =
1 + x2

Z1

Z1
f (x)dx =

dx

= . Ta chia on [0; 1]
2
1+x
4

thnh 2n phn bng nhau bi cc im chia


x0 = 0, x1 =

2
2n 1
2n
1
, x2 =
, . . . , x2n1 =
, x2n =
= 1.
2n
2n
2n
2n

1
, i = 1, 2, . . . , 2n. Trn cc on [x2i ; x2i+1 ] hay [x2i+1 ; x2i+2 ]
2n
2i + 1
(i = 0, 1, . . . , n 1) ta u chn im i =
. Th th tng tch phn ca hm f (x) trn
2n
[0; 1] l
Khi i = xi xi1 =

f (0 )1 + f (1 )2 + f (2 )3 + + f (2n2 )2n1 + f (2n1 )2n

1
3
5
(2n 1)2
1+ 2
1+ 2
1+ 2
1+
4n
4n
4n
4n2


1
4n2
4n2
4n2
4n2
=
+
+
+ +
2n 1 + 4n2 32 + 4n2 52 + 4n2
(2n 1)2 + 4n2
n1
1 X
4n2
=
= Sn .
2n i=0 (2i + 1)2 + 4n2
=

2n

Z1
Vy lim Sn =

Z1
f (x)dx =

n+

+ +

dx

= .
2
1+x
4

Lu .
Trong cch chn trn th i c chn chnh l u mt chung ca hai on lin k nhau.
p dng thnh cng phng php tng tch phn, chng ta phi bin i
Sn = x1 + x2 + + xn
thnh tng tch phn ca mt hm no :
121

Tp ch Epsilon, S 09, 06/2016


Trn on [0; 1], [0; a] hay [a; b] ph hp;
Thng vi php chia u min ly tch phn (rt cn tha s

1
);
n

Chn im trung gian i mt tri, mt phi, u mt ca hai on lin k nhau,


hay trung im on th i, thm ch phi chng minh i chnh l mt im no
ca on th i.
Nhiu khi tng Sn = x1 + x2 + + xn khng phi l tng tch phn, m ch "gn nh"
vy, trong trng hp ny phng n a ra thng l i tng; thay th cc s hng trong
tng; thm bt lng thch hp a v tng tch phn ca mt hm s no

2.3. Phng php i tng


Trong tng tch phn ca hm s f c biu thc

n
P

f (i ), tuy nhin trong mt s bi tp i

i=1

khng phi l chy t 1 n n, m l mt qu trnh khc. Lc ta phi i tng, tc l


thay i bi mt biu thc no .
Cng c khi gii hn cn tnh khng phi l tng tch phn ca bt k mt hm no, lc
ta hy c gng thay th s hng tng qut ca tng bi mt s hng khc. Phng php
y thng l s dng tnh lin tc ca hm s (xem bi ton 2.9 trang 122) hoc s
dng nh l Lagrange (xem bi ton 2.10 trang 122, bi ton 2.11 trang 122).

Bi ton 2.7. Tnh lim

n+

2n1
X
k=n

!
1
.
k2

Li gii. Ta s i tng k chy t n n 2n 1 bng tng i chy t 0 nh sau:


n

2n1
X
k=n

vi f l hm s f (x) =

n1

X
1
1
=
n
2 (do thay k bi i + n)
k2
(i
+
n)
i=0
n1
n1  
1X
i
1
1X
=
,
f

2 =
n i=0
n i=0
n
i
1+
n

1
. Chia on [0; 1] bi cc im chia
(1 + x)2
xi =

i
, i = 0, 1, . . . , n,
n

tc l chia on [0; 1] thnh n on u nhau:


x0 =

0
1
2
n1
n
= 0 < x1 = < x2 = < < xn1 =
< xn = = 1.
n
n
n
n
n
122

Tp ch Epsilon, S 09, 06/2016


i
[xi ; xi+1 ], i = 0, 1, 2, . . . , n 1. Khi tng tch phn ca hm s f (x) trn
n
n1  
1X
i
. Nh vy:
[0; 1] ng vi php phn hoch ni trn l
f
n i=0
n
Chn i =

lim

n+

2n1
X

1
k2

k=n

2n1
X

Bi ton 2.8. Tnh lim

n+

Li gii. Trong un =

k=n

2n1
X
k=n

Z1
Z1
n1  
1X
i
dx
= lim
f
= f (x)dx =
n+ n
n
(1 + x)2
i=0
0
0
1

1
1
1
=
= +1= .

x+1 0
2
2

1
.
2k + 1

1
, thay k bi i + n, ta c:
2k + 1
un =

2n1
X
k=n

t vn =

n1
X
i=0

n1

X
1
1
=
.
2k + 1
2i
+
2n
+
1
i=0

1
, khi :
2i + 2n
n1

1 1X
vn =
2 n i=0

n1

1 1X
=
f
i
2 n i=0
1+
n
1

 
i
,
n

1
. Nh vy:
1+x
1
Z1
Z1

1
1
1
1
dx
lim vn =
f (x)dx =
= ln(1 + x) = ln 2.
n+
2
2
1+x
2
2
0

vi f l hm s f (x) =

Li c:


n1 
X
1
1


|un vn | =




2i
+
2n
+
1
2i
+
2n
i=0


n1
X

1
n


=
, n = 1, 2, . . .
6

2n(2n + 1)
(2i
+
2n)(2i
+
2n
+
1)
i=0
n
nn s dng nguyn l kp suy ra lim |un vn | = 0.
n+ 2n(2n + 1)
n+
Nh vy:











1
1
1
un ln 2 = (un vn ) + vn ln 2 6 |un vn | + vn ln 2 .






2
2
2
M lim

T y li s dng nguyn l kp ta c lim un =


n+

123

1
ln 2.
2

Tp ch Epsilon, S 09, 06/2016


Bi ton 2.9. Tm gii hn:

1
n

2
+
lim
n+
n+1

2
n

1
2

n+

+ +

n
n

n+

1
n

Li gii. Bin i tng di du gii hn v dng


1

2n
+
Sn =
n+1

2n
n+

1
2

+ +

2n
n+

1
n

i
n
X
2n

i=1

i
n
X
1 2n
=
.
.
1
1
n
i=1
n+
1+
i
ni

Dng nh v phi khng phi l tng tch phn ca bt k hm no trn on [0; 1] vi php
chia u thnh n on v chn im i mt tri, mt phi cng nh trung im thng thng.
Tuy nhin d chng minh rng


i
i
i1
i
2n
2n
1
2 n =
6
6 2 n , i = 1, 2, . . . , n do 1 < 1 +
62 .
1
2
ni
1+
ni


i1 i
x
V hm s g(x) = 2 lin tc trn R nn tn ti i
;

n n
i

2i =

2n
1 .
1 + ni

n 1
n 1
P
P
.2i . M
.2i l tng tch phn ca hm s g(x) = 2x trn on [0; 1] vi
n
n
i=1
i=1
php chia u on [0; 1] thnh n on v cch chn im i nh ch. Do

T Sn =

Z1
lim Sn =

n+

1
2x
1
2 dx =
=
.

ln 2 0 ln 2
x

Bi ton 2.10. Gi s hm s f (x) c o hm kh tch v b chn trn on [a; b]. t




n
baX
ba
Sn =
f a+i
. Chng minh rng:
n i=1
n

Zb

lim n Sn

f (x)dx =

n+

ba
[f (b) f (a)].
2

Bi ton 2.11. Gi s hm s f (x) c o hm cp hai kh tch v b chn trn on [a; b]. t




n
baX
ba
Sn =
f a + (2i 1)
. Chng minh rng:
n i=1
2n

lim n2

Zb

f (x)dx Sn =

n+

124

(b a)2 0
[f (b) f 0 (a)].
24

Tp ch Epsilon, S 09, 06/2016


Lu . Bi ton 2.10, bi ton 2.11 l nhng kt qu quan trng, nh hng cho vic tnh gii
hn ca mt s dy s lin quan n tng v xy dng h thng bi tp. Cn vic gii bi ton
2.10 v bi ton 2.11 bn c c th tham kho trong ti liu tham kho sch Chuyn kho Dy
s (NXB HQG H Ni) xut bn nm 2013 ca cng tc gi.
(K sau ng tip)

125

Tp ch Epsilon, S 09, 06/2016

126

SNG TO VI MT BI TON HNH HC


TRUNG HC C S
Nguyn Ngc Giang (TP H Ch Minh)

Trung hc c s, cc bi ton v ng phn gic v bi ton v ng trung


tuyn chim vai tr quan trng. S d chng quan trng v chng c mt s tnh cht
p v c p dng nhiu trong gii ton. Sng to ton bc hc ny kh hn
cc bc hc cp trn v cng c gii ton cn tng i t. i vi hnh hc, th vic
sng to ton ngoi vic tm cc cch gii p dng cc nh l quen thuc nh Thales,
Menelaus, . . . th phng php v ng ph l phng php c yu thch. Thm
vo , cc phng php tng t ha, khi qut ha cng l cc phng php c
a dng. Xin gii thiu vi cc bn nhng iu va ni qua bi ton hay sau y
Bi ton 1. Cho 4ABC ng trung tuyn BM ct ng phn gic CD ti P . Chng minh
rng
AC
PC

=1
P D BC
Sau y l cc cch gii bi ton 1:
Cch 1 V DK k BM . Ta c
MC
MA
PC
=
=
PD
MK
MK
AB
AB
=
=
+1
BD
BC

127

Tp ch Epsilon, S 09, 06/2016


Cch 2 V DI k AC. Ta c
PC
MC
MA
=
=
PD
DI
DI
AB
AB
=
=
+1
BD
BC

Cch 3 K AN k BM . Suy ra P M l ng trung bnh ca 4AN C, tc l P N = P C. Ta c


PN
PC
=
.
PD
PD
AD
AC
ND
=
=
(do AN k BP )
M
DP
BD
BC
ND
AC

+1=
+1
DP
BC
PC
PN
AC

=
=
+1
PD
PD
BC

128

Tp ch Epsilon, S 09, 06/2016


Cch 4 K AH k P C.
PC
AH
Suy ra
=
(do 4AM H = 4CM P nn P C = AH)
PD
DP

PC
AC
=
+ 1(do DP k AH)
PD
BC
Cch 5 Trn tia i ca tia M B ly im E sao cho M B = M E.
Do M A = M C AB k CE v AB = CE.
Hay

PC
CE
AB
AC
=
=
=
+1
PD
BD
BD
BC
Cch 6 V CS k BM .
Suy ra BM l ng trung bnh ca 4ACS nn BS = AB.
Ta c

129

Tp ch Epsilon, S 09, 06/2016

PC
BS
AB
Do BM k CS
=
=
. Hay
PD
BD
BD
PC
AB
AC
=
=
+ 1.
PD
BD
BC
AB
PC
SBP C
SABP
v
.
Cch 7
=
=
PD
SDBP
BD
SDBP

M SABM = SBM C , SAP M = SCP M SABP = SBP C


PC
AB
AC
Suy ra
=
=
+1
PD
BD
BC
Cch 8 Ko di AP ct BC ti N .
M A N C DB
p dng nh l Ceva ta c

= 1(1)
M C N B DA
V M l trung im AC nn M A = M C (2)

130

Tp ch Epsilon, S 09, 06/2016

NC
DA
=
(3)
NB
DB
PC
MC NC
p dng nh l van Aubel ta c
=
+
(4)
PD
MA NB
AC
PC
=1+
.
T (1), (2), (3), (4)
PD
BC
Suy ra iu phi chng minh.
Bi ton tng t bi ton 1 l bi ton sau
Vy

Bi ton 2 Cho 4ABC, M l im nm trn AC sao cho


ng phn gic CD ca 4ABC ti P . Chng minh rng

PC
3AC

=3
PD
BC

Cch 1 V DK k BM . Ta c

PC
MC
3M A
=
=
PD
MK
MK
3AB
3AC
=
=
+3
BD
BC
131

1
AM
= . ng thng BM ct
MC
3

Tp ch Epsilon, S 09, 06/2016

Cch 2 V DI k AC. Ta c
MC
3M A
PC
=
=
PD
DI
DI
3AB
3AC
=
=
+3
BD
BC

Cch 3 K AN k BM .
Suy ra P M l ng thng song song vi cnh AN ca 4AN C.
Theo nh l Thales ta c 3P N = P C.

132

Tp ch Epsilon, S 09, 06/2016

PC
3P N
=
.
PD
PD
ND
AD
AC
M
=
=
(do AN k BP )
DP
BD
BC
3N D
3AC
Suy ra
+3=
+3
DP
BC
PC
3P N
3AC
Suy ra
=
=
+3
PD
PD
BC
MC
PC
=
=3
Cch 4 K AH k P C. Theo nh l Thales,
AH
MA
Vy

PC
3AH
Suy ra
=
.
PD
DP
PC
3AC
Hay
=
+ 3 (do DP k AH)
PD
BC
MA
MB
AB
1
Cch 5 Trn tia i ca tia M B ly im E sao cho
=
=
= .
MC
ME
CE
3
133

Tp ch Epsilon, S 09, 06/2016


Vy theo nh l Thales o, ta c AB k CE.

PC
CE
3AB
3AC
=
=
=
+3
PD
BD
BD
BC
Cch 6 V CS k BM .

MC
BS
Theo nh l Thales, ta c BM l ng thng song song vi CS nn
=
= 3. M
AB
MA
BS
3AB
PC
=
=
(1)
BM k CS
PD
BD
BD
3AB
3AC
Ta c
=
+ 3(2)
BD
BC

PC
3AC
T (1), (2), ta c
=
+3
PD
BC
PC
SBP C AB
SABP
Cch 7
=
,
=
PD
SDBP BD
SDBP
134

Tp ch Epsilon, S 09, 06/2016


1
1
1
m SABM = SBM C , SAP M = SCP M SABP = SBP C .
3
3
3

PC
3AB
3AC
=
=
+3
PD
BD
BC
M A N C DB
Cch 8 Ko di AP ct BC ti N . p dng nh l Ceva ta c

= 1(1)
M C N B DA
1
Theo gi thit M A = M C
3
NC
3DA
Vy
=
(2)
NB
DB
MC NC
PC
=
+
(3)
p dng nh l van Aubel, suy ra
PD
MA NB
Suy ra

T (1), (2), (3)

PC
3AC
=3+
Bi ton tng qut ca bi ton 1 v bi ton 2 l bi ton
PD
BC
135

Tp ch Epsilon, S 09, 06/2016


MC
= n(n > 0) ng thng BM
MA
ct ng phn gic CD ca 4ABC ti P . Chng minh rng
Bi ton 3 Cho 4ABC, M l im nm trn AC sao cho

PC
nAC
=
+n
PD
BC

Cch 1 V DK k BM . Ta c

MC
nM A
PC
=
=
PD
MK
MK
nAB
nAC
=
=
+n
BD
BC

Cch 2 V DI k AC. Ta c

PC
MC
nM A
=
=
PD
DI
DI
nAB
nAC
=
=
+n
BD
BC
136

Tp ch Epsilon, S 09, 06/2016

Cch 3 K AN k BM .
Suy ra P M l ng thng song song vi cnh AN ca 4AN C. Theo nh l Thales ta c
P C = nP N .

PC
nP N
=
.
PD
PD
ND
AD
AC
M
=
=
(do AN k BP ).
DP
BD
BC
Vy

137

Tp ch Epsilon, S 09, 06/2016

nAC
nN D
+n=
+n
DP
BC
PC
nP N
nAC

=
=
+n
PD
PD
BC
MC
PC
=
= n. Suy ra P C = nAH.
Cch 4 K AH k P C. Theo nh l Thales ta c
AH
MA
PC
nAH

=
.
PD
DP
PC
nAB
nAC
Hay
=
=
+n
PD
BD
BC
ME
CE
MC
=
=
= n.
Cch 5 Trn tia i ca M B ly im E sao cho
MA
MB
AB

Theo nh l Thales o, ta c AB k CE.


PC
CE
nAB
nAC

=
=
=
+n
PD
BD
BD
BC
138

Tp ch Epsilon, S 09, 06/2016


Cch 6 V CS k BM .
Theo nh l Thales, ta c BM l ng thng song song vi CS nn
k CS.
PC
BS
nAB

=
=
(1)
PD
BD
BD
nAC
nAB
=
+ n(2)
Ta c
BD
BC

BS
MC
=
= n m BM
AB
MA

PC
nAC
T (2), (3), ta c
=
+n
PD
BC
PC
SBP C AB
SABP
1
1
Cch 7
=
,
=
. M SABM = SBM C , SAP M = SCP M SABP =
PD
SDBP BD
SDBP
n
n
1
SBP C .
n

139

Tp ch Epsilon, S 09, 06/2016


PC
nAB
nAC
=
=
+n
PD
BD
BC
Cch 8 Ko di AP ct BC ti N . p dng nh l Ceva ta c
M A N C DB

= 1(1)
M C N B DA
NC
nDA
Theo gi thit M C = nM A. Vy
=
NB
DB

p dng nh l van Aubel ta c


MC NC
PC
=
+
(3)
PD
MA NB
PC
nAC
=n+
.
PD
BC
Chng ta va c mt s khm ph th v xoay quanh bi ton v giao im ca ng trung
tuyn v ng phn gic. Cc cch gii khc nhau, cc bi ton tng t v m rng em
n cho chng ta nhiu iu th v. Bi vit ny cn trao i g thm? Mong c s chia s ca
cc bn.
Sau y l mt s bi ton luyn tp
DB
2CB
Bi ton 4 Cho 4ABC c trung tuyn BM . D l im trn AB sao cho
=
. Gi P
DA
CA
PC
CA
l giao im ca CD v BM . Tnh t s
theo
.
PD
CB
DB
nCB
Bi ton 5 Cho 4ABC c trung tuyn BM . D l im trn AB sao cho
=
. Gi P
DA
CA
PC
CA
l giao im ca CD v BM . Tnh t s
theo
.
PD
CB
T (1),(2),(3),

Ti liu tham kho


[1]Phan c Chnh, Tn Thn, V Hu Bnh, Trn nh Chu, Ng Hu Dng, Phm Gia c,
Nguyn Duy Thun(2011), Ton 8 tp 1, Nh xut bn Gio dc Vit Nam.

140

Tp ch Epsilon, S 09, 06/2016


[2]Phan c Chnh, Tn Thn, Nguyn Huy oan, L Vn Hng, Trng Cng Thnh, Nguyn
Hu Tho (2011), Ton 8 tp 2, Nh xut bn Gio dc Vit Nam.
[3]Nguyn c Tn (2005), Gii bng nhiu cch cc bi ton lp 9, Nh xut bn Tng hp
Thnh ph H Ch Minh.

141

Tp ch Epsilon, S 09, 06/2016

142

NU BN KHNG NUI DNG, AM M


S T B BN" - TR CHUYN V CON
NG N VI CMU CA PHM HY HIU
(Trung Dng - Tp ch Ngi th)

c Google ba ln gi li mi lm vic. Nghin cu trong lnh vc kh thi s hin nay ca


cng ngh cao: Tr thng minh nhn to. Xc nh ch lm Google mt nm, sau dnh thi
gian cho chng trnh tin s ca i hc Carnegie Mellon (CMU) ... Phm Hy Hiu l mt trong
nhng trng hp ngi Vit tr n lc khng nh bn thn tm c v tr ca mnh trong
th gii ngy cng phng v xch li gn nhau hn nh nhng bc tin v bo ca cng ngh.
Th nhng cng t ngi bit rng, c c nhng hoch nh y, ngi bn tr ny khng
t ln b thch thc bi nhng bi ton hc ba t ra t chnh cuc sng ...
Bo ch trong nc dn ngun tin t cc du hc sinh Vit Nam ti M, v giai thoi ba ln bn
nhn c li mi ca Google (trong hai ln u t chi). La chn v con ng n vi
g khng l Google ca bn thc s nh th no? hn l mt quyt nh y cn nhc bi
bn cng c nhng c hi n t Apple, Microsoft?
Con ng n vi Google ca ti rt di. Ti thay i t ch mun lm cho Google v danh
ting ca cng ty ny n ch ng lm cho Google v ngh rng mnh s hc c nhiu kinh
nghim c ch t h.
Nh hi nm nht i hc, b ng v thiu thng tin v cc cng ty, ti nh liu gi h s xin
thc tp Google. H ni khng phng vn v ti cha tng c kinh nghim lp trnh. Thy
vy, ti hc mt vi lp lp trnh Stanford ri nm hai xin vic li. Nm th hai i hc, ti
c phng vn v vt qua tt c cc cu hi ca h, nhng cui cng khng c nhn v h
ni ti khng hp vi cc n thc tp. Lc ny ti tht s khng hiu Google mun g khi
tuyn mt ng vin na.
Cng trong thi gian ny, suy ngh ca ti v vic hc dn thay i. Ti nhn ra mnh phi hc
ci m mnh thch v c gng tr thnh mt ngi c nng lc, ch khng phi hc kim
mt cng vic hay gy n tng vi mt cng ty no c. Ngh th, ti khng quan tm g n
Google na. Ti bt u hc v tr tu nhn to, thch g hc , v c gng tin b.
Nm ba, ti li np h s cho Google. H tuyn ti vo thc tp, nhng bng mt quy trnh m
theo ti l v l. Th nht, cc cu hi phng vn ca Google, theo ti, l nhng cu hi xut
hin y ry trong cc sch gio khoa hoc bin i cht t. Th hai, theo suy lun ca ti, vi
cc cu hi d, phn ln cc ng vin u vt qua vng phng vn. Google cho cc k s ca
mnh c li h s ca mi ngi vt qua vng phng vn chn xem ai hp vi cc n
thc tp. Nhng ai khng hp s b loi. Vi ti, hp vi cc n thc tp l mt thc o
y cm tnh v thiu cng bng. Theo , nu ti khng c nhn (iu ny xy ra nm
hai i hc ca ti), ti khng hiu ti sao. Cn khi ti c nhn, ti ngh vic mnh c nhn
chng c gi tr g c. V ngh vy nn ti t chi Google, ng thi ni vi h nhng suy ngh
ti va ni trn.
143

Tp ch Epsilon, S 09, 06/2016


Sau khi tt nghip, ti li c Google nhn lm vic chnh thc. Ln ny, ti cm thy cc cu
hi phng vn ca h bt u lm kh mnh. Nhng khi ti hi h cn gi chnh sch hp
vi n khng v bit h vn cn gi, ti ni vi h rng ti khng cm thy thoi mi nu
cng hin cho mt cng ty i x vi ng vin ca mnh nh vy, v ti li t chi h.

Phm Hy Hiu trong mu o i Cardinal ca i hc Stanford tham d vng chung kt Cuc thi lp trnh
ACM dnh cho sinh vin i hc ton th gii t chc ti Nga nm 2014. Vit Nam c mt s sinh vin
tng tham gia cuc thi tin hc quc t (IOI) c tham gia vng chung kt th gii trong mu o cc i
hc MIT (M), NUS (Singapore), ... nhng chuyn t thi ton quc t (IMO) sang thi ACM th Phm Hy
Hiu l trng hp c bit - nh CTV.

n thng 3.2016 va ri, Google mt ln na mi ti lm vic. Ti li hi chnh sch tuyn


dng ca h, v khi bit rng h thay i, ti bt u cn nhc vic lm s vo lm cho
Google. Lc ny, ti so snh Google vi Facebook, Apple, Microsoft v vic i hc tin s i
hc Carnegie Mellon (CMU). y l mt quyt nh kh khn. Ti hi xin li khuyn t gio s
hng dn mnh Stanford cng nh nhiu bc n anh v c cc gio s c th s hng dn
mnh CMU. Cui cng, sau khi suy ngh v nghe cc li khuyn, ti quyt nh s lm vic
Google Brain trong mt nm ri s vo CMU lm tin s.
B quyt lt vo mt xanh ca nhng cng ty hng u l g? Bi thc t ngay c vic
xt h s trao hc bng cc trng i hc nhm n nhng hc sinh gii v ti nng ch khng
cn quan nim con nh ngho hc gii, trong khi cc cng ty nhm n nhng ng vin c
nng lc thc s, ranh gii v vng min, chng tc hay quc gia c v rt m. Bn c ng tnh
quan im ?
B quyt ca ti l ng quan tm n cc cng ty . C ln ti c nghe mt bui ni chuyn
ca GS. Ng Bo Chu i hc KHTN TP.HCM. C mt ngi hi gio s rng ng c t k
hoch n khi no th nhn c gii thng Fields khng. Cu tr li ca gio s lm ti rt tm
c: Khng bn . Ti ngh nhng ai i nghin cu ton v Gii thng Fields th mt l khng
c, hai l rt d vo bnh vin tm thn. Cu ni ny tr thnh kim ch nam cho nh
144

Tp ch Epsilon, S 09, 06/2016


hng hc tp ca ti. Ti hc ci m ti thch v hc theo s am m ca mnh. Ti hc v ti
mun nng cao nng lc ca bn thn, c th cng hin cho lnh vc mnh yu thch. Ti ngh
khi bn c nng lc, Apple, Facebook, Google, Microsoft, hay bt c ci g tng t, s t tm
n. Ngc li, nu bn hc v nhng iu th c hi ca bn s gim i nhiu.
Ti khng ng tnh vi quan im ranh gii vng min, chng tc c v rt m. Ti tin rng
th gii ca chng ta ngy cng phng v gn nhau hn, nhng chng mc no , cc ranh
gii trn vn cn . Ti hy vng trong tng lai, cc ranh gii ny m dn v tin ti ch c hi
m ra cho tt c mi ngi.
Hy k mt cht v cng vic hin nay i bn?
Ti ang tham gia nghin cu sau i hc v tr tu nhn to i hc Stanford. Hng nghin
cu ca ti l cc ng dng ca mng n-rn gip my tnh hiu c ngn ng ca con ngi.
Ti tp trung vo cc
nghin cu a ngn ng,
nhm gip my tnh hiu
v giao tip c vi ngi
dng bng nhiu ngn ng
khc nhau. Hin nay, nhiu
tr tu nhn to tng tc
vi con ngi bng ting
Anh tt hn so vi cc
ngn ng khc. Nguyn
nhn l do s ph bin ca
ting Anh, khin ngi ta
c th d dng thu thp
d liu cho my tnh hc.
Tuy nhin, nu my tnh c
th s dng cc ngn ng
khc, cng tt nh ting
Anh, th ngi dn nhiu
nc trn th gii s c th
tip cn cc cng ngh ny.
Khi cc cng ngh a ngn ng tr nn hon thin hn, ti tin rng con ngi khng nht thit
phi hc ting Anh ch nhm mc ch tip cn cng ngh. Thay vo , h c th u t thi
gian rn luyn tht gii mt k nng no , sau da vo my tnh tip nhn v truyn t
kin thc ca mnh cho ton th gii.
T thng 6.2016, ti s bt u lm vic Google v sau mt nm s bc vo chng trnh
tin s ca CMU. C th hng nghin cu ca ti s thay i. Ti khng ni trc c, phi
nh vo duyn s thi.
Va qua mi ngi quan tm s kin tr thng minh nhn to chin thng con ngi. Bn c
th cho mi ngi hnh dung mt cht v AlphaGo? Theo bn, ti sao chin thng c gii
nghin cu tr tu nhn to nh gi cao?
AlphaGo l mt h thng s dng hai mng n-rn tng tc vi nhau. Mt mng n-rn tm
ra nc c tt nht cho mi lt i, cn mng kia nh gi th c xem li th hay thit hi ca
chng trnh so vi i th nh th no.
145

Tp ch Epsilon, S 09, 06/2016


Chin thng ca AlphaGo c nh gi cao v Go (c vy) l mt tr chi c cho rng rt
kh i vi tr tu nhn to. Trc AlphaGo, cc chuyn gia v tr tu nhn to vn tin rng sm
mun g my tnh cng s chin thng con ngi v c vy, tuy nhin h d on rng iu s
xy ra trong vng 10 nm, tc l khong nm 2025.
Thnh cng ca AlphaGo cho thy tc pht trin ca tr tu nhn to vt qua d on
ca h, v th n c nh gi cao. AlphaGo cng m ra nhiu hng i mi trong nghin cu
tr tu nhn to. Cc thut ton dng trong tr tu nhn to ny cng c th c ng dng
gip my tnh hiu c ngn ng t nhin hay nhn din c ging ni v hnh nh. Ngoi ra,
thnh cng ca AlphaGo cng khin cc nghin cu v tr tu nhn to nhn c nhiu s quan
tm hn ca cng ng. Thng qua , cc nghin cu ny c th thu ht nhiu ngun u t, t
to ra tim nng pht trin thm.
L ngi ang gp phn gip cho nhng c my tr nn thng minh, bit t duy v cm xc, c
bao gi bn lo ngi con ngi s b chnh my mc iu kin li? Bi, theo mt cch hiu no ,
con ngi ang lm bing suy ngh m ph thuc vo cng ngh; hin chng nghin smartphone
hay mng x hi ang lm con ngi t giao tip vi nhau, tng rt gn nhng li rt xa?
Tr tu nhn to ng l ang pht trin rt nhanh, nhng chng cn rt xa mi n ch c th
iu khin c con ngi. Lo lng v iu trong lc ny s ch khin cc nghin cu tr tu
nhn to mt tp trung mt cch khng cn thit. Bn thn ti khng cho rng con ngi ang
lm bing suy ngh hay ph thuc vo cng ngh. Trc khi c my tnh, con ngi phi t thc
hin rt nhiu php ton. Sau khi cc ng dng tnh ton c pht trin, chng ta s dng cc
ng dng ny thay mnh, nhng chng ta u c ni mnh ph thuc vo cc ng dng ny? Tr
tu nhn to cng vy. Ti ngh chng ch l nhng phn mm gip chng ta lm vic hiu qu
hn, tit kim cng sc v thi gian tp trung cho nhng cng vic quan trng hn.
Bn c di theo hot ng cng nh i sng ton hc trong nc? Nhng chuyn ng mang
li cho bn suy nh g v nhng th h n em ang tip bc?
Ti khng c chuyn mn v ton cao cp nn khng hiu c nhiu v cc nghin cu v hot
ng chuyn su v ton, c trong v ngoi nc. Hot ng ton hc duy nht trong nc m
ti cn theo di c l phong tro olympic ton. Ti thy by gi vic dy v hc ton olympic
trong nc ci m hn. Cc em hc sinh do c nhiu c hi hn tip xc vi cc ti liu
cng nh tham gia cc hot ng giao lu v hc hi. Do , cht lng dy v hc ang tng
ln. Ti ngh nu cc em bit kin tr vi am m v nh hng ca mnh, cc em s i xa hn
th h chng ti.
Ton hc ang h tr cho bn nh th no trong cng vic hin nay nghin cu v tr tu nhn
to? Thut ton hc su - Deep Learning c v l hng i kh thi thng cho vic nghin cu
tr tu nhn to? C hi ny c qu tm cho cc bn tr Vit Nam?

Vic bn bc ra
khi trng i hc
nh th no quan
trng hn rt nhiu
so vi vic bn
bc vo trng
i hc no.

Ton xy dng cho ti mt u c bit t duy logic v khng


s cng thc. iu ny c th hi bun ci, nhng hin nay c
nhiu k s phn mm v thm ch l cc nh nghin cu rt s
nhn thy cc phng trnh ton hc. iu ny dn n nhiu sn
phm cng ngh c to ra mt cch my mc m khng hiu
ti sao mnh lm nh th li cho ra kt qu tt. iu ny v lu
di s to ra ro cn cho s pht trin cc thut ton v sn phm
mi. Ti cm thy mnh may mn v chng nhng khng s ton
m cn thch th o su tm hiu v bn cht ton hc ca nhiu
thut trong tr tu nhn to.
146

Tp ch Epsilon, S 09, 06/2016


Deep learning ng l hng i thi thng hin nay v c th
l trong nhiu nm na.Thut ton ny chuyn cc kh khn trong tr tu nhn to t vic
thu thp d liu, vn mt thi gian v tn km, thnh vic lp trnh c nhng mng n-rn
ln, c kh nng x l nhiu d liu n gin mt cch nhanh v hiu qu.Vi s pht trin ca
phn cng my tnh, vic gii quyt nhn chung l kh thi hn. V th deep learning mi t c
nhiu thnh tu nh vy.
Deep learning cng khng h qu tm so vi cc bn tr Vit Nam. Ngc li, n ang tr nn
ph bin v d tip cn hn nhiu. Ly v d, nm 2012, TS. L Vit Quc cng Google cng b
mt phng php dng deep learning xy dng khi nim con mo t cc video YouTube
bng cch kt hp 16.000 CPU ca rt nhiu my tnh. Gi ca m hnh ny thi im khong
1 triu USD. Tuy nhin, ch vi thng sau , mt nhm nghin cu ca Stanford cng b mt
m hnh deep learning khc, c th cho ra kt qu tng t nhng ch c gi khong 20 nghn
USD. l nm 2012, cn hin nay cc thit b ny c th mua c vi vi nghn USD. Chi
ph ny, theo ti, khng phi l qu tm i vi nhiu c s nghin cu ca nc ta hin nay.
Mt khi c c thit b, lm c hay khng hon ton ph thuc vo con ngi.
u l la chn kh nht ca bn trong cng vic v cuc sng?
iu kh nht ti tng la chn l t chi hc bng ca i hc NUS (Singapore) ng thi
nh mt nm hc ting Anh v tm hc bng i M. Tht ra y khng phi l la chn
ca ti m l nh hng ca b m. Cng v lu di, ti v gia nh cng nhn ra y l mt
quyt nh sai lm. N li nhiu hu qu cho bn thn ti m n by gi ti vn ang phi
khc phc.
V bn dung ho v gi c s t lp, thch nghi vi mi trng mi nh th no? Nu c
la chn li, bn c ph nhn c m trc y v Singapore l hi lng mn, mang tnh ho
hc ca tui tr?
By gi mi khi ngh li chuyn Singapore, ti vn cha vt qua c cm gic tic nui. i
du hc i hc M, ti phi tr ci gi qu t i vi ti: mt nm khng c hc v lm
nhng iu mnh thch. Thi cp ba, ti ch u t cho cc cuc thi hc sinh gii nn trnh
ting Anh khng vt qua cc k thi chun ha vo i hc ca M nh TOEFL v SAT.
Ti cng khng hiu bit g v phong tc v vn ha ca ngi M vit mt essay tuyn sinh
cho h c. Ci gi ca tt c nhng iu ny l mt nm b ra hc nhng iu ti cha tng
quan tm trc .
Mt nm ny i vi ti tai hi v cng. Ti phi hc nhng iu ti cho l v ngha. Tai hi hn,
hc ting Anh v vn ha M, ti phi cm bn thn mnh khng c hc ton, bi ch cn
ng vo ton, ti s d dng b ra vi gi tm hiu mt vn mnh thch m qun i nhim
v chnh: hc ting Anh. Nhng ai tng thi ton olympiad u hiu, ch cn vi tun khng
luyn tp, k nng tng rn luyn nhiu nm s mt dn i, ng ni g n mt nm nh ti.
iu ti t nht, ti cm thy lng t trng ca mnh b xc phm. Ti tham gia vo mt vi
nhm t vn dnh cho cc bn tr xin vo cc i hc ln M. Trong cc nhm ny, ti c
nghe rng cc thnh tch trong nhng cuc thi olympiad khng c gi tr trong mt cc nh tuyn
sinh. H ch quan tm n im SAT cao.
147

Tp ch Epsilon, S 09, 06/2016

Phm Hy Hiu (o trng) nhn gii thng Ben Wegbreit dnh cho lun vn tt nghip xut sc nht ca
khoa Khoa hc my tnh, i hc Stanford 2015 - nh t liu ca gia nh.

Tuy nhin im SAT ca ti khng cao. Ti thi 4 ln mi c 2.090, im s nm trong 25%


thp nht trong kha hc ti Stanford ca ti. Khi ti vo Stanford, im SAT khng cao m
k nng ton ca ti cng gim st. Hi nm nht, c ln mt gio s ni chuyn vi ti c
cp n b Cauchy-Davenport. Ti nh tn ca b ny, nhng khng th nh ni ni dung
ca n. V gio s ny ngc nhin v sao ti c th i din Vit Nam ti IMO m khng bit b
quan trng ny. Ti xu h v cng. Ti k iu ny thy rng, ti phi b li nhng iu
mnh mt rt nhiu thi gian v cng sc hc nh th no. Ti cn gp nhiu rc ri khc
vi Stanford. T ngn ng, vn ho, cch sinh hot, cch ho nhp. Ti sng qua thi i hc vi
nhiu bun hn l vui.
i li, ti tin rng nu i Singapore, iu u tin l ti khng phi mt mt nm hc SAT,
TOEFL, ... Xung quanh ti cng s c nhiu bn b m ti quen t cp 3 hn. Nhng ngi
bn , ti tin l h hiu ti v c th gip ti cng hc tp, cng tin b, tt hn nhiu so vi
thc t m ti phi tri qua.
Ti xin gi n bn thn ti ca 6 nm trc, cng nh nhng bn tr ang ngh v i hc, du
hc, v.v. mt li nhn nh: Hy hi bn thn cc bn thch iu g v c gng lm iu tht
tt. ng v ci tn Stanford, MIT hay Harvard hay bt c g m gc li am m ca mnh,
lm mt iu mnh khng thch. Bi v nu bn khng nui dng, am m s t b bn. Cuc
i ca cc bn c khong 4 nm trng i hc v phn sau di hn rt nhiu. V th, vic
bn bc ra khi trng i hc nh th no quan trng hn rt nhiu so vi vic bn bc vo
trng i hc no. D cc bn thch ton, tin hc, vn hc, piano, nhy hin i hay hot ng
x hi, am m ca cc bn u ng trn trng nh nhau. Hy u t vo am m . Chnh
nhng iu ny s l cha kho cc bn thnh cng.
Ai l ngi nh hng ti bn, n la chn con ng i, nhng c m sau ny ca bn nhiu
nht?
148

Tp ch Epsilon, S 09, 06/2016


Ti nhn c nhiu nh hng trong vic nh hnh tnh cch t chnh b m ca mnh. Ti hc
t m ti c tnh phi bo v quan im ca mnh cho n khi bit c mnh sai. Tht ra
chnh s cng nhc ny cng khin m ti v ti nhiu ln bt ng quan im. Cng ln, ti
cng bo v c quan im ca mnh trc m. Ngc li, ti hc t b mnh c tnh coi trng
nng lc ca con ngi v phn no quan nim v s c - mt i. Khi ti lm vic trong
mt nhm v t c phn thng, ti thng t ng gp phn ca mnh vo mt cuc lin
hoan hoc vo qu chung. Bi l ti quan im rng ci qu nht, l k nng t c phn
thng kia, th mnh hc c ri.
L ngi c tri nghim mi trng hc kh c bit, bi trong nc bn c hc ti nhng
trng Top u ca thnh ph, v i du hc vi hc bng ton phn i hc v thc s ca
Stanford. Dnh c nhiu phn thng gi tr trong thi gian i hc, c bit l bng danh d
Stanford. Nhng phn thng ngu nhin n vi bn hay l nhng mc tiu bn ngm
n t trc? Bn c thy bn thn mnh may mn? B quyt sng v hc ni x ngi, duy tr
c phong l g?
Ti ngh rng mnh gp nhiu may mn. Ngi ngoi nhn
vo ti by gi c th s thy cc gii thng, nhng li mi
lm vic v nhng hc bng. Nhng bt c ai thn vi ti s
hiu ti tri qua nhng thi khc kh khn nh th no.
Ti cha tng dm ngh mnh s t c nhng g ti ang
c hin nay. Ti cm thy nhng iu n vi ti mt cch
ngu nhin, nh mt n ci ca s phn, khuyn khch ti
hy tip tc theo ui con ng mnh chn.
Hin ti, ti vn ang tm b quyt sng vui v v hc tp,
lm vic hiu qu x ngi. Ti ngh iu quan trng nht
l xc nh c mt am m, mt mc tiu gn lin vi am
m v mt nh hng t c mc tiu ny. Nhn vo
tng lai, ti ch thy mnh cn b sung nhiu hiu bit v
tr tu nhn to, trau di cc k nng lp trnh v pht trin
cc k thut nghin cu khoa hc ca bn thn. Ti khng
th bit mnh s i n u, nhng ti ngh cuc i nh th
mi th v.

Khi ti lm vic trong


mt nhm v t
c phn thng,
ti thng t ng gp
phn ca mnh vo mt
cuc lin hoan hoc
vo qu chung. Bi l
ti quan im rng ci
qu nht, l k nng
t c phn thng
kia, th mnh hc
c ri.

Nhiu ngi nh gi bn l ngi ti nhng cng y c tnh, khng ngi bc l quan im c


nhn. Tnh cch mnh m c hnh thnh nh nhng yu t g?
Cm n nhng ngi nh gi cao v ti. Ti cha ngh mnh l ngi ti nng, cn c tnh
ca ti hnh thnh khi ti suy ngh v hu qu ca nhng ln mnh im lng. V d, trn ti
tr li vic khng i Singapore gy ra cho ti nhng hu qu nh th no. Mt trong nhng l do
l v ti im lng, khng ni ln quan im ca mnh khi b m ti ni ti phi i M. Ti ngh
nu c nghe nhng iu ti ng l nn ni, b m ti s ngh li v gip ti c c quyt
nh tt hn cho cuc sng ca mnh. V vy, by gi ti lun phi ni ln nhng g mnh ngh.
Nh th, d ti c sai, ti cng c c hi bo v quan im ca mnh cho n khi ti nhn ra
nhng quan im l sai.
Phng chm, hay trit l cuc sng ca bn l g? Bn c la chn mt cu chm ngn no
cho ring mnh?
Ti quan nim mnh phi tm c mt vic mnh thch lm v lm tht gii vic . y l
cch tt nht cng hin cho x hi. Ti thng xuyn gp b tc trong vic tm ra con ng
149

Tp ch Epsilon, S 09, 06/2016


cho mnh, t nhng quyt nh c nhn cho n cc vn nghin cu, hc tp. Hin nay, ti
ngh iu tt nht nn lm l bc ln pha trc. C th bn s vp ng nhng t nhng sai lm
ca mnh, bn s tm ra con ng tt cho bn thn. V th, ti rt thch cu ca L Tn Trn
i ny lm g c ng. Ngi ta i mi th thnh ng thi. Ti cng thch mt phin bn
vn hc mng ca cu ni ny, nghe c v th hn: C i. Pha trc s c ng.
Dnh c khng t vinh quang nhng c khi no bn thy mnh tht bi hay b tc? Bn
vt qua iu nh th no?
Ti tht bi nhiu hn thnh cng rt nhiu ln. i vi ti, tht bi rt kh chu, nhng khng
bao gi l du chm ht. Tht bi lun sa cha c. Ti lm th nghim hun luyn cho my
tnh hc bng mt thut ton mi, ri nhn ra thut ton ca mnh hon ton sai v my...
chng hc c g c. l loi tht bi khng lm mt qu nhiu thi gian sa. N lm tng
thm kinh nghim ca ti, ti ch cn ghi li v t ni vi mnh rng sau ny ng dng thut
ton na.

Gio dc gia nh nh hng nhiu n tnh cch ca Hy Hiu trong nghin cu. Trong nh Hy Hiu
cng m v em trai - nh T liu gia nh.

Cng c loi tht bi cn nhiu thi gian hn sa cha. Trc y, c mt ln ti n xin gio
s hng dn ca mnh vit th gii thiu xin vo chng trnh Tin s. Lc kinh nghim
nghin cu ca ti cha nhiu. Thy ni thng vi ti l nu ti cha vit c mt bi bo khoa
hc vi thy th thy c vit g i na, c hi ti u vo Tin s cng rt thp. Ti nh hm
gp thy xong, mnh i lang thang khp ni, chng thit lm vic g c. Nhng cng nh m
ti hiu ra tm quan trng ca vic cng b nhng cng trnh trong chng ng nghin cu ca
mt nh khoa hc. Mt nm sau , sau khi lm v cng b mt s nghin cu, ti gp li v
gio s ny v hi xin th gii thiu. Thy ng vit v ti t c nhng g mnh mun.
150

Tp ch Epsilon, S 09, 06/2016


Ti ngh im mu cht vt qua tht bi l kim sot c cm xc ca bn thn, tm ra
nhng iu mnh lm cha ng dn n tht bi v sau ny ng lm nh vy na. Ni th
d, nhng bn thn ti nhiu khi cng khng lm c.
Cn trong cng vic, mc tiu ti thng m bn hng ti l g? Hy chia s v la chn khi
n vi chng trnh Tin s ca i hc Carnegie Mellon (CMU) i Hiu? c m no m bn
ang tip tc eo ui?
C nhiu l do khin CMU l mt mi trng nghin cu l tng i vi ti. Th nht, CMU
c nhiu chuyn gia hng u v lnh vc hp ca ti l x l ngn ng t nhin. Khi la chn
trng lm tin s, khng nhng phi ngh n danh ting ca ngi gio s hng dn m
cn phi ngh n vic v gio s c thi gian dnh cho mnh khng, c th quan tm n
ti ca mnh khng, ... Ti thy CMU c nhng iu ny. Th hai, CMU nm thnh ph
Pittsburgh, mt ni tng i cch bit vi cc thnh ph ln khc ca M nhng li khng c
lp. Ti hy vng s cch bit ny c th gip ti tp trung vo vic nghin cu m khng b phn
tm v nhng yu t bn ngoi.
Ti tng m c mnh tr thnh mt gio s dn dt cc nghin cu tr tu nhn to v ging
dy li kin thc mnh tch lu c cho cc th h sau. Tuy nhin, cng ngy ti cng nhn ra c
rt nhiu cch cng hin cho x hi. Ti khng bit liu mnh c th tr thnh mt gio s
c khng ci cn phi nh vo ci duyn na. Hin ti ti ch mun hc c tht nhiu
m rng hiu bit, trau di cc k nng.

151

Tp ch Epsilon, S 09, 06/2016

152

V PHONG TRO OLYMPIC TON


SAUDI ARABIA
V Quc B Cn
(Archimedes Academy)
Hai nm tr li y, chng ti c may mn c sang thm v lm vic cng i tuyn Ton
Saudi Arabia mt thi gian. Nhng ngy thng lm vic ti y cho chng ti hiu thm v
phong tro hc Ton ph thng x Hi gio ny.
Bi vit di y xin c chia s nhng cm nhn ca chng ti v cch cc thnh vin i
tuyn Olympic nc bn hc Ton. Nhng chia s y c phn hi ch quan nhng hy vng
rng bn c s c ci nhn khi qut v phong tro nc bn, cng nh thy c nhng ci
hay, ci cha hay rt kinh nghim cho mnh.
Trc ht, xin c ni qua v cch thc t chc tuyn chn v bi dng hc sinh gii Saudi
Arabia. Trc y, Saudi Arabia t chc tuyn chn nhng em hc sinh c nng khiu v Ton
hc bi dng Riyadh. Sau , trc mi k thi Quc t quan trng, cc em s c lm bi thi
loi, nhng bn c kt qu lm bi tt nht s c chn vo i tuyn i thi vi hy vng mang
vinh quang v cho nc nh.
T nm 2014 n nay, Saudi Arabia c chng trnh hp tc hun luyn vi thy L Anh Vinh,
nhng kha hc tp trung c chuyn v trng i hc Khoa hc v Cng ngh mang tn nh
vua Abdullah (gi tt l KAUST). Cc hc sinh s c tp hun trc tip vi cc hun luyn
vin t Vit Nam sang v cc ch lin quan n Olympic Ton. V ngoi nhng kha hc tp
trung ny, hc sinh cng c rn luyn trc tuyn nh vi nhng bi tp c thy Vinh trc
tip bin son v a ln website ca mnh hng tun.

Cc em hc sinh level 3, 4 cng cc hun luyn vin


153

Tp ch Epsilon, S 09, 06/2016


Tip theo, chng ti s phn tch nhng im tt v im cha tt v phong tro hc Ton y.
im tt: Cc em hc sinh rt thn thin, nng ng, thch o su suy ngh nhng vn m
gio vin giao v thng xin thm thi gian tm cch gii cc bi ton khi gio vin yu cu
c cha tip tc bi ging.
S tng tc ca cc em vi gio vin thc s rt tt. Chng ti rt bt ng bui ging u
tin ca mnh, khi hu nh em no cng mun chia s kin v vn m chng ti ging. Nh
m vn cng thm sng t, lp hc tr nn si ng hn hn, khin ngi thy cng tr
nn ho hng hn khi ging bi.
iu ny tri ngc vi Vit Nam, hc sinh thng rt r hn khi trao i vi gio vin. Nhiu
em ch tip nhn bi ging ca thy c mt cch th ng m khng dm t vn hay a ra
kin c nhn. C l hc sinh Vit Nam ta nn thay i dn cch hc tr nn tch cc hn.

Cc em hc sinh level 4+ cng cc hun luyn vin


im cha tt: Tng tc tt trn lp l vy, nhng kh nng t hc ca hc sinh y cha
c tt. Hc sinh kh li ghi chp bi v khi hc trn lp, do khi v nh thng khng xem
li bi hiu r vn hn, v cc em. . . hay qun. S rt kh nh v hiu nhng l thuyt
kh nu nh chng ta khng nghin ngm thng xuyn. Chng ti vn thng. . . th cc em
bng cch cho li nhng bi ton dy bui trc vo cc bui tip theo cch bui u hi xa
mt cht, v s tht l c rt t em nhn ra cc bi ton .
Mt iu ng ni na l cc em khng t gic cao khi nh, khng tm kim ti liu c
thm. Cng nh cp trn, thy Vinh c a ln website ca mnh cc bi tp hc sinh rn
luyn thm, nhng c rt t em chu lm v np bi (d iu gn nh l bt buc). R rng l
nu ch ph thuc vo nhng iu gio vin ging trn lp th kh c th c nhng tin b ng
k c.
Hc sinh y ch thch th sc nhng bi ton trong thi Olympic Ton thay v tp trung xy
dng cho mnh mt nn tng Ton tht tt ri mi luyn cc bi Olympic. Mi khi chng ti
ging l thuyt, nhiu em t ra ngn ngm, khng tp trung, c em cn lm vic ring. C bi
154

Tp ch Epsilon, S 09, 06/2016


ton no thy cho m dng dp khc Olympic Ton l cc em khng c gng lm, c bn
bin minh rng nhng bi nh th s khng bao gi xut hin trong cc k thi Ton.
iu ny tri ngc vi cch hc Ton khoa hc l phi thm nhun tng trong cc l thuyt
Ton hc ri mi luyn cc bi Olympic tip cn cc tng mi. Mt ci cy s khng th
ng vng nu phn gc b su mt.
Tuy nhin, mt iu ng mng l sau hai nm lm vic cng cc bn i tuyn Saudi, chng ti
cng dn tc ng v ci thin c t tng ca cc em. Nhiu em bt u ghi chp bi v
cn thn v xem li, c em cng tm n cc din n Ton m chng ti gii thiu tm
kim thm t liu t trau di, rn luyn.
C mt iu hn ch na thuc v vn tn gio, Saudi Arabia l mt quc gia Hi gio vi
100% dn s theo o Hi, m tn Hi gio mi ngy phi cu nguyn 5 ln t sng sm cho
n ti. Do , thi gian hc ca hc sinh b nh hng kh ln, gn mt phn ba thi gian cc
bui hc ch dnh cho vic cu nguyn.
Ngoi ra, cng do tn gio nn Saudi, nam n nu khng phi cng huyt thng hoc c quan
h v chng th khng c ni chuyn t do vi nhau, khng c ngi gn nhau. C c mt i
ng cnh st tn gio chuyn kim sot vic ny. Do , nam n khng c hc chung vi nhau.
KAUST th mi vic thong hn mt cht v y hc vin a phn l du hc sinh quc t,
nam n c th hc chung vi nhau. Nhng do t tng tn gio thm vo cc em nn hc
sinh nam v hc sinh n thng khng trao i kin vi nhau, thm ch c nhng bi ton cc
em n lm c, gio vin yu cu ln bng trnh by li cho cc bn khc cng nghe th cc
em nam cng thng khng tp trung lng nghe bn trnh by. iu ny cng gy nh hng t
nhiu n hiu qu hc tp ca lp hc.
l iu chng ti nhn thy c khi lm vic y, tuy nhin do y l vn thuc v tn
gio, vn ha ca nc bn nn chng ta ch c th tn trng m khng th gp .
Mc d cn nhiu im cha tt, nhng mi vic vn ang chuyn bin theo chiu hng tch
cc. iu th hin r rng qua kt qu ca i tuyn Saudi cc k thi Quc t. i tuyn
t kt qu tt hn so vi cc nm trc, thm ch c huy chng vng BMO, JBMO,
APMO, iu cha tng c nhng nm trc y.
Tin rng vi vic i mi t tng ca hc sinh, phong tro Olympic Ton Saudi Arabia s
ngy cng pht trin tch cc hn. Bi vit xin c kt thc y.

155

Tp ch Epsilon, S 09, 06/2016

156

BI TON HAY LI GII P


Trn Nam Dng
(i hc Khoa hc T nhin - HQG TP.HCM)

LI GII THIU
Chuyn mc ny c ly cm hng t bi vit ca thy Nguyn Duy Lin s
bo trc v bi ton s 6 trong k thi IMO 2001 vi 5 cch gii khc nhau. Mc
ny s dnh vit v nhng bi ton hay, li gii p v nhng cu chuyn th v
xung quanh nhng bi ton v li gii .
Tn ca chuyn mc c mn t tn ca mt nhm nhng ngi yu ton trn
Facebook do anh Nguyn Vn Li sng lp Bi ton hay Li gii p am m
ton hc. Chuyn mc ghi nhn cc c ca bn c v s chn ng mi k 1, 2
bi ton.
S ny chng ti s gii thiu vi bn c v bi ton s 6 trong thi ton quc
t nm 1995 vi nhng li gii sau ny c a vo nhng sch gio khoa v
phng php gii ton.
Bi ton. (IMO 1995, bi ton 6) Cho p l s nguyn t l. C bao nhiu tp con A gm p phn
t ca tp hp {1, 2, . . . , 2p} c tng cc phn t chia ht cho p?
Chng minh. t X = {1, 2, . . . , p}, Y = {p + 1, p + 2, . . . , 2p}. Vi tp con A X ta
nh ngha php tnh tin theo s k l k + A = {y X : y k + x (mod p), vi x A no
}. Ta cng nh ngha iu tng t trong Y.
By gi ta ch rng nu P {1, 2, . . . , 2p} v |P | = p th P = X, P = Y hoc 0 <
|P X|, |P Y |.
Tip theo ta xt P 6= X, Y v nh ngha
k + P = (k + (P X)) (P Y ), k X.
Ch rng v 0 < |P X| < p nn tng ca cc phn t ca k + P s ly mi thng d modulo
p, trong ch c mt thng d l tha mn bi ton ca chng ta. Ta c th nh ngha quan h
tng ng trn cc tp hp P {1, 2, . . . , 2p} vi p phn t, P 6= X, Y nh sau:
P Q (P Y = Q Y

& k + P X = Q X, k X).

Quan h tng ng ny xc nh mt phn hoch ca h cc tp con p phn t ca {1, 2, . . . , 2p}


khc vi X v Y. Mi mt lp tng ng ca phn hoch c p phn t v t mi lp c ng
mt phn t tha mn iu kin ca bi ton.
Suy ra p s ca bi ton l

p
C2p
2
p

+ 2.
157

Tp ch Epsilon, S 09, 06/2016


Chng minh. Gi l cn nguyn thy bc p ca n v. Khi
2p
Y

(x i ) = (xp 1)2 = x2p 2xp + 1.

i=1

So snh h s ca xp ta c
2=

i1 +i2 ++ip

p1
X

nj j .

j=0

Trong tng th nht tnh theo tt c cc tp con p phn t {i1 , i2 , . . . , ip } ca {1, 2, . . . , 2p}
v nj trong tng th hai l s cc tp con sao cho
i1 + i2 + + in j

(mod p).

T y suy ra rng l nghim ca


G(x) = (n0 2) +

p1
X

n j xj ,

j=1

l mt a thc c bc p 1. V a thc ti tiu ca trn trng cc s hu t l F (x) =


1 + x + + xp1 , cng c bc p 1. T y suy ra G(x) phi c dng k F (x) vi k l hng
s. Nh th n0 2 = n1 = n2 = = np1 . V
p
n0 + n1 + + np1 = C2p
,

nn t y ta suy ra n0 =

p
C2p
2
p

+ 2.

Li gii th nht, s dng phn hoch v song nh, thuc v tc gi bi ton, Marcin Kuczma,
trng on Ba Lan. Li gii th hai, s dng s phc, c th l cn n v, thuc v Roberto
Dvornicich, trng on . Nikolay Nikolov, hc sinh ngi Bulgaria c trao gii c bit
cho li gii ca mnh, kh ging vi li gii th hai nhng trnh by c ng hn na, ch trong 2
dng. Nikolay ot 2 huy chng vng v 1 huy chng bc trong cc k IMO trc v ti
k IMO nm 1995 ot gii qun qun vi s im tuyt i. on Vit Nam nm c bn
Ng c Tun gii c bi ny.

158

NG NHT THC BRAHMAGUPTA FIBONACCI V NG DNG


Trn Nam Dng
(i hc Khoa hc T nhin - HQG TP.HCM)
Mt hc sinh chuyn t lp 7 ln lp 8 t l s c yu cu chng minh hng ng thc:
(a2 + b2 )(c2 + d2 ) = (ac + bd)2 + (ad bc)2 ,
v s chng gp my kh khn. Bn y s khng ng rng ngy xa, chng minh n, nh ton
hc ni ting Fibonacci phi mt n 5 trang giy. V cng khng ng rng, hng ng thc
ny, m dng tng qut hn s c tn gi l ng nht thc Brahmagupta - Fibonacci, s c
nhiu ng dng p , hiu qu v bt ng. Chng ta cng i vo mt chuyn tham quan n
vi lch s ca ng nht thc p ny, nhng ng dng kinh in v nhng bi ton mi nht
cn nng hi. iu c bit l cc bn hc sinh THCS c th hiu c a s cc bi ton v v
d trong bi vit ny.

1. i dng lch s
Phn dn dt lch s ny ch yu c ly t [4].
ng nht thc m ngy nay thng c gi l ng nht thc Brahmagupta - Fibonacci rt
n gin nhng c mt lch s th v v c nhng ng dng bt ng
(a2 + b2 )(c2 + d2 ) = (ac + bd)2 + (ad bc)2 = (ac bd)2 + (ad + bc)2 .
Ngi ta tin rng ngi u tin nhc n ng nht thc ny l Diophantus (th k th III
trc cng nguyn) ngi vit trong cun S hc ca mnh (Quyn III, Bi ton 19) [Stillwell,
trang 174]: 65 mt cch t nhin c chia thnh hai bnh phng bng hai cch ... v 65 l
tch ca 13 v 5, mi mt s ny u l tng ca hai bnh phng.
John Stillwell cp rng nm 950 sau cng nguyn, nh ton hc vng Ba T al-Khazin
(900 971 SCN) gii thch khng nh ca Diophantus trong dng tng qut hn gn nh
ging vi cng thc nu trn. Mt gii thch tng t cng xut hin trong cun sch ca
Fibonacci Liber Quadratorum vo nm 1225. Lc ngn ng ton hc cn vo thi th u
nn Fibonacci phi mt 5 trang giy chng minh cng thc ny.
tng ca Diophantus c tng qut ha trc bi nh ton hc n Brahmagupta
(597 668 SCN), ngi chng minh
(a2 + N b2 )(c2 + N d2 ) = (ac + bd)2 + N (ad bc)2 = (ac bd)2 + N (ad + bc)2 .
V s dng n gii phng trnh Pell x2 N y 2 = 1.
159

Tp ch Epsilon, S 09, 06/2016


Cng thc, trong dng c bn ca n l trng hp ring (khi n = 2) ca ng nht thc
Lagrange
! n !
!2
n
n
X
X
X
X
a2i
b2i =
ai b i +
(ak bj aj bk )2 .
i=1

i=1

i=1

16i<j6n

ng vi mi n > 2, v d, vi n = 3 c dng sau


(a2 + b2 + c2 )(x2 + y 2 + z 2 ) = (ax + by + cz)2 + (ay bx)2 + (az cx)2 + (bz cy)2 ,
v c s dng chng minh cc tnh cht ca im i trung.
Tuy nhin, ng nht thc Brahmagupta - Fibonacci (n = 2) s hu nhng tnh cht c bit
khng cn ng vi ng nht thc Lagrange, cng nh trng hp n = 3 ca n. l cch m
Diophantus c th pht biu tng qut ha ca bi ton 19 ca ng:
Tch ca hai s nguyn, mi s l tng hai bnh phng, cng l tng ca hai bnh phng.
Ta khng th ni iu ny cho tng ca ba bnh phng. Nhng vo nm 1748, Euler thng bo
v ng nht thc bn bnh phng:
(a21 + a22 + a23 + a24 )(b21 + b22 + b23 + b24 ) = (a1 b1 a2 b2 a3 b3 a4 b4 )2
+ (a1 b2 + a2 b1 + a3 b4 a4 b3 )2 + (a1 b3 a2 b4 + a3 b1 + a4 b2 )2 + (a1 b4 + a2 b3 a3 b2 + a4 b1 )2 .
Trng hp n = 2 lin quan n l thuyt s phc m c th din t nh l chun
(m-un, gi tr tuyt i) ca hai s phc bng tch cc chun ca chng. Vai tr tng t cng
c dnh cho ng nht thc bn bnh phng trong l thuyt quaternion v ng nht thc
tm bnh phng Degen trong l thuyt octonion [Conway and Smith].
Bi tp 1. Hy chng minh ng nht thc Brahmangupta - Fibonacci
(a2 + N b2 )(c2 + N d2 ) = (ac + bd)2 + N (ad bc)2 = (ac bd)2 + N (ad + bc)2 .

2. ng dng trong bi ton biu din s nguyn


Bi ton biu din s nguyn l bi ton tm iu kin mt s nguyn c th biu din c
di mt dng cho trc, v d di dng tng ca hai bnh phng (s 0, 1, 2, 4, 5 biu din
c nhng 3 th khng), tng ca ba bnh phng (s 0, 1, 2, 3, 4, 5, 6 biu din c, nhng
7 th khng). nh l Langrange li ni rng mt s t nhin bt k c th biu din c di
dng tng ca bn bnh phng.
Tuy nhin, chng ta s tm thi khng i xa nh vy, v cc kt qu nu trn tuy l nhng kt qu
kinh in nhng cng l hi qu sc i vi hc sinh THCS, i tng chnh ca bi vit ny. Ta
s i vo mt s vn c th v c gii hn hn.
Bi ton 1. Ta ni s nguyn dng n l biu din c nu tn ti cc s nguyn a, b sao cho
n = a2 + b 2 .
a) Chng minh rng nu m, n l biu din c th mn cng biu din c.
160

Tp ch Epsilon, S 09, 06/2016


b) Chng minh rng nu 2m biu din c th m cng biu din c.
c) Xt xem trong cc s 2013, 2014, 2015, 2016, 2017 s no biu din c.
Chng minh. a) Kt lun ny chnh l kt qu m Diophantus pht biu v c chng minh
trc tip nh ng nht thc Brahmagupta - Fibonacci.
b) Gi s 2m = a2 + b2 th a, b cng tnh chn l, suy ra a + b v a b l s chn v ta c

2 
2
a+b
ab
m=
+
,
2
2
t suy ra iu phi chng minh.
ng thc trn chnh l ng nht thc Brahmagupta - Fibonacci p dng cho 12 =
v 2m = a2 + b2 . Ta lu th thut nh c th hu ch cho cc tnh hung sau.


1 2
2


1 2
2

c) Ta c mt s chnh phng chia 3 d 0 hoc 1 v chia 4 d 0 hoc 1. Do nu a2 + b2 chia


ht cho 3 th a v b u phi chia ht cho 3, do a2 + b2 chia ht cho 9. Cng t s c dng
4k + 3 khng th biu din di dng tng ca hai bnh phng. p dng tnh cht ny ta suy ra
cc s sau y khng biu din c:
S 2013 (chia ht cho 3 m khng chia ht cho 9), 2015 (c dng 4k + 3).
S 2014 = 2 1007 khng biu din c do 1007 c dng 4k + 3 khng biu din c (p
dng tnh cht b)). S 2016 = 27 63 khng biu din c do 63 c dng 4k + 3 khng biu
din c (p dng nhiu ln tnh cht b).
S 2017 biu din c (ta dng thut ton n tham th t trn xung di v tm c
2017 = 442 + 92 ).
Bi ton 2. Cho m v n l cc s nguyn dng phn bit. Hy biu din m6 + n6 di dng
tng ca hai bnh phng khc vi m6 v n6 .
Chng minh. Ta c
3

m6 + n6 = (m2 ) + (n2 ) = (m2 + n2 )(m4 m2 n2 + n4 )


h
i
2
= (m2 + n2 ) (m2 n2 ) + (mn)2 .
By gii p dng ng nht thc Brahmagupta - Fibonacci cho a = m, b = n, c = m2 n2 v
d = mn ta thu c
2

m6 + n6 = (m3 mn2 + mn2 ) + (m2 n m2 n + n3 ) .


y khng phi l iu m ta mong mun v biu din ny trng vi biu din ban u. Nhng
khng c g mt mt y nu ta rng hng ng thc Brahmagupta - Fibonacci s c dng
khc i mt cht nu ta thay d bng d
(a2 + b2 )(c2 + d2 ) = (ac bd)2 + (ad + bc)2 .
Lc ny ta c th vit
2

m6 + n6 = (m3 mn2 mn2 ) + (m2 n m2 n + n3 )


2

= (m3 2mn2 ) + (n3 2m2 n) .


y chnh l kt qu cn tm.
161

Tp ch Epsilon, S 09, 06/2016


Ta c th thy rng yu cu m v n nguyn dng c th b qua: ng nht thc m ta thu c
l ng nht thc i s v ng vi mi bin s (hay n s) m v n.
Bi ton trong thi v ch ton Moscow s t ra mt vn kh khn hn mt cht:
Bi ton 3. (Moscow MO) Chng minh rng vi mi n > 3, tn ti cc s l x, y sao cho
7x2 + y 2 = 2n .
Chng minh. Nu khng c iu kin x, y l th bi ny c th gii n gin nh vo quy np.
Ch vi n = 3 ta c biu din 7 12 + 12 = 23 v vi n = 4 th 7 12 + 32 = 24 v nu
7x2 + y 2 = 2n th 7(2x)2 + (2y)2 = 2n+2 . Tuy nhin, vi iu kin x, y l th bc chuyn t n
ln n + 2 khng dng c. Ta dng n gii php gn ging vi gii php bi 1, vi ch l
 2  2
1
1
7
+
= 2.
2
2
T , nu gi s 2n = 7x2 + y 2 vi x, y l, th p dng ng nht thc Brahmagupta - Fibonacci
ta c
"  
 2 #
2
1
1
n+1
n
2
=22 = 7
+
(7x2 + y 2 )
2
2
2

2

xy
7x + y
+7
=
2
2


2
2
7x y
x+y
=
+7
.
2
2
Do x, y u l nn cc s trong ngoc u nguyn. Tuy nhin, theo yu cu bi, ta cn
chng l. n y ta ch cn xt 2 trng hp
Nu x, y cng s d mod 4 th ta chn biu din th 2.
Nu x, y khc s d mod 4 th ta chn biu din th 1.
Trong mi trng hp phng trnh 2n+1 = 7x2 + y 2 c nghim x, y l. Theo nguyn l quy
np ton hc ta c iu phi chng minh.
Bi tp 2. (Saudi Arabia TST 2016, Juniors) Cho k l s nguyn dng. Chng minh rng tn
ti cc s nguyn x, y khng s no chia ht cho 3 sao cho x2 + 2y 2 = 3k .
Bi tp 3. (VMO 2010) Chng minh rng vi mi s nguyn dng n, phng trnh
x2 + 15y 2 = 4n ,
c t nht n nghim nguyn khng m.
Bi tp 4. (Saudi Arabia Training Camp 2016) Cho S l tp hp tt c cc s t nhin biu din
c di dng x2 + 3y 2 vi x, y l cc s nguyn. Chng minh cc tnh cht sau y ca S
i) Nu m S, n S th mn S.
ii) Nu N S v 2 | N th

N
4

S.

iii) Nu N S, s nguyn t p S v p|N th


162

N
p

S.

Tp ch Epsilon, S 09, 06/2016

3. ng dng trong phng trnh Pell


Theo mt s nh nghin cu lch s ton hc th phng trnh Pell, tc l phng trnh nghim
nguyn dng
x2 N y 2 = 1,
(3.1)
vi N l s nguyn dng khng chnh phng ng ra phi c gi l phng trnh Brahmagupta - Bhaskara. Tuy nhin chng ta s khng i su v kha cnh lch s phn tch xem
hai ng tin xa nh th no trong vic gii phng trnh (3.1) m ch nu ra ng dng c bn
ca ng nht thc Brahmagupta-Fibonacci trong vic gii phng trnh Pell v phng trnh
dng Pell.
Bi ton 4. Chng minh rng phng trnh
x2 2y 2 = 1,

(3.2)

c v s nghim nguyn khng m.


Chng minh. Ta thy rng nu a2 2b2 = 1 v x2 2y 2 = 1 th p dng ng nht thc
Brahmagupta - Fibonacci ta c
1 = (a2 2b2 )(x2 2y 2 ) = (ax + 2by)2 2(ay + bx)2 .

(3.3)

T y d dng suy ra nu (3.2) c nghim th (3.2) s c v s nghim. D thy x = 3, y = 2 l


nghim ca (3.2) do (3.2) c v s nghim. Hn na, nu p dng (3.3) cho a = 3, b = 2 v
x, y l mt nghim nguyn khng m ca (3.2) th ta thy: Nu (x, y) l nghim ca (3.2) th
(3x + 4y, 2x + 3y) cng l nghim ca (3.2).
V d t cp (3, 2) theo cch ny ta ln lt c cc nghim (17, 12), (99, 70), (577, 408), . . .
iu th v l cch xy dng ny s vt ht tt c cc nghim nguyn khng m ca phng trnh
(3.2) (tt nhin phi b sung nghim hin nhin x = 1, y = 0). Tuy nhin chng ta s khng
chng minh s kin ny.
Cng t cch chng minh trn, ta thy rng phng trnh (3.1) nu c nghim nguyn dng s
c v s nghim nguyn dng. Hn na, nu (3.1) c nghim nguyn dng v phng trnh
x2 N y 2 = a,

(3.4)

c nghim nguyn dng th (3.4) s c v s nghim nguyn dng.


Ngi ta chng minh c rng nu N l s nguyn dng khng chnh phng trnh phng
trnh (3.1) lun c nghim nguyn dng, v theo nh lp lun trn, c v s nghim nguyn
dng. Hn th na, ta c th tm c tt c nhng nghim . Kt qu trn vn ny thuc v
Langrange (1767 1838) da vo l thuyt lin phn s. Tuy nhin, ta cng s khng i su nh
vy m xem xt mt v d p dng s tn ti v s nghim ca phng trnh dng Pell.
Bi ton 5. (EGMO 2016) Gi S l tp hp cc s nguyn dng sao cho n4 c c s
nm trong on [n2 + 1, n2 + 2n]. Chng minh rng c v s cc phn t ca S c dng
7m, 7m+1, 7m+2, 7m+5, 7m+6 nhng khng c phn t no ca S c dng 7m+3, 7m+4.
163

Tp ch Epsilon, S 09, 06/2016


Chng minh. Mu cht ca li gii ny l nhn xt
n2 + k | n4 n2 + k | n4 k 2 + k 2 n2 + k | k 2 .
Tip theo, s dng iu kin k thuc [n2 + 1, n2 + 2n] d dng suy ra

k2
n2 +k

{2, 3}.

Cui cng ta chng minh cc phng trnh k 2 = 2(n2 + k), k 2 = 3(n2 + k) khng c nghim
n c dng 7m + 3, 7m + 4 (dng m-un) v c v s nghim dng 7k, 7k + 1, 7k + 2, 7k + 5
(phng trnh th nht) v dng 7k, 7k + 6 (phng trnh th hai) bng cch s dng cng thc
Brahmagupta - Fibonacci.
Bi tp 5. (VMO 1999) Cho hai dy s (xn ), (yn ) xc nh nh sau
x1 = 1, x2 = 4, xn+2 = 3xn+1 xn , vi mi n > 1,
y1 = 1, y2 = 2, yn+2 = 3yn+1 yn , vi mi n > 1.
Chng minh rng cc s nguyn dng a, b tha mn phng trnh a2 5b2 = 4 khi v ch
khi tn ti s nguyn dng k sao cho a = xk , b = yk .

4. Mt s ng dng khc
ng nht thc Lagrange, trng hp tng qut ca ng nht thc Brahmagupta - Fibonacci
cho ta h qu l bt ng thc Cauchy - Schwarz, mt bt ng thc c in c nhiu ng dng
quan trng.
Bt ng thc Cauchy-Schwarz trng hp n = 2: Nu a, b, c, d l cc s thc bt k th ta c
bt ng thc
(a2 + b2 )(c2 + d2 ) > (ac + bd)2 .
Du bng xy ra khi v ch khi a : b = c : d.
Cng i theo hng bt ng thc, nhng thin v biu din, nh l sau y l mt kt qu tng
t vi kt qu trong l thuyt s (v iu kin cn v mt s nguyn dng c th biu din
di dng tng bnh phng ca hai s nguyn) dnh cho a thc. Ha ra iu kin cn v
mt a thc vi h s thc c th biu din c di dng tng bnh phng ca hai a thc
vi h s thc l P (x) > 0 vi mi x. Tuy nhin, kt qu ny chng minh n gin hn kt qu
trong l thuyt s rt nhiu. Chng minh thc t ch dng n ng nht thc Brahmagupta Fibonacci.
Bi ton 6. Cho P (x) l a thc vi h s thc sao cho P (x) > 0 vi mi x thuc R. Chng
minh rng tn ti cc a thc Q1 (x) v Q2 (x) vi h s thc sao cho
P (x) = Q21 (x) + Q22 (x).
Chng minh. V P (x) khng m nn hoc P (x) khng c nghim thc, hoc mi nghim thc
ca n c bi chn. T y suy ra
P (x) = c

n
Y

(x2 + pk x + qk ),

k=1

trong c v pk , qk , k = 1, 2, . . . , n l cc s thc, vi c > 0 v p2k 4qk 6 0, k = 1, 2, . . . , n.


164

Tp ch Epsilon, S 09, 06/2016


Nhng vi mi k, phng php gom bnh phng ng chng t rng
!2
p
2

2
4q

p
p
k
k
k
x2 + p k x + q k = x +
+
,
2
2
do
P (x) = c

n
Y

(Uk2 (x) + Vk2 (x)).

k=1

By gi, p dng ng nht thc Brahmagupta - Fibonacci nhiu ln, ta c th bin s cc tha
s t n tr v 1.
Cui cng, ta s nu ra mt ng dng th v ca ng nht thc bn bnh phng ca Euler.
Bi ton 7. Chng minh rng:
a) Phng trnh x2 + y 2 + z 2 = 7 khng c nghim hu t.
b) Phng trnh sau khng c nghim hu t
x2 + y 2 + z 2 + t2 = 7, a2 + b2 + c2 + d2 = 1, ax + by + cz + d = 0.
Chng minh. a) iu cn chng minh tng ng vi chng minh khng tn ti nghim
(x, y, z, t) khc (0, 0, 0, 0) ca phng trnh
x2 + y 2 + z 2 = 7t2 .

(4.1)

Gi s tn ti nghim nguyn khng m khng ng thi bng 0. Chn x, y, z, t l nghim ca


(4.1) c x + y + z + t nh nht.
Nu t l th v tri 7 (mod 8) trong khi x2 + y 2 + z 2 ch c th c s d l 0, 1, 2, 3, 4, 5, 6
(mod 8), mu thun. Vy t chn, suy ra v phi ng d 0 hoc 4 (mod 8). Suy ra x, y, z
phi cng chn. Nhng lc th x2 , y2 , z2 , 2t cng l nghim ca (4.1) mu thun vi cch chn
x, y, z, t. Vy iu gi s l sai, suy ra iu phi chng minh.
b) Gi s h c nghim hu t th t ng nht thc Euler ta s suy ra
7 = (a2 + b2 + c2 + d2 )(x2 + y 2 + z 2 + t2 )
= (ax + by + cz + dt)2 + (ay bx + ct dz)2 + (az bt cx + dy)2 + (at + bz cy dx)2
= (ay bx + ct dz)2 + (az bt cx + dy)2 + (at + bz cy dx)2 .
Suy ra phng trnh X 2 + Y 2 + Z 2 = 7 c nghim hu t, mu thun vi a).

Ti liu tham kho


[1] A. Shenitzer, J. Stillwell, Mathematical Evolutions, MAA, 2002.
[2] J. H. Conway, D. A. Smith, On Quoternions and Octonions, A K Peters, 2003.
[3] S. Savchev, T. Andreescu, Mathematical Miniatures, MAA, 2003, p. 6 7.
[4] Alexander Bogomolny:
http://www.cut-the-knot.org/m/Algebra/BrahmaguptaFibonacci.
shtml
165

Tp ch Epsilon, S 09, 06/2016

166

MT S BI TON TRONG THI VO CC


TRNG CHUYN
(Ban bin tp)

Trong bi vit ny, chng ti gii thiu mt s bi ton trch trong cc thi vo cc
trng Chuyn trn ton Quc va din ra trong cui thng 5 v u thng 6.
Bi 0.1. (Trch thi vo 10 Chuyn Phan Bi Chu-Ngh an)
Gii phng trnh

5 3x + x + 1 = 3x2 4x + 4.
5
Li gii. iu kin: 1 6 x 6 .
3
Bnh phng hai v ca phng trnh ta c
p
2 (5 3x) (x + 1) = 3x2 2x 2.
p

t t = (5 3x) (x + 1) = 3x2 + 2x + 5, t > 0 ta c phng trnh


t2 + 2t 3 = 0 t = 1 (do t > 0).
Suy ra

3x2 + 2x + 5 = 1 3x2 2x 4 = 0 x =

1 13
Kt hp iu kin ta c: x =
l nghim ca phng trnh.
6

13

Bi 0.2. (Trch thi vo 10 Chuyn Ton PTNK)


Tm a > 1 phng trnh
ax2 + (1 2a) x + 1 a = 0
c hai nghim phn bit x1 , x2 tha mn
x22 ax1 = a2 a 1

(1).

Li gii. Vi bi ton lin quan n nghim ca phng trnh bc hai th cng c ta thng
s dng l nh l Vi-et. Vi iu kin x22 ax1 = a2 a 1th ta kt hp vi iu kin
ax22 + (1 2a) x2 + 1 a = 0 (2) x l. Da vo nh l Vi-et, ta c th thay x1 = 2a1
x2
a
bin i (1) v ng thc ch cha x2 v a. C th:
x22 ax1 = a2 a 1


2a 1
2
x2 a
x 2 = a2 a 1
a
ax22 + a2 x2 = a3 + a2 2a (3).
167

Tp ch Epsilon, S 09, 06/2016


Kt hp (2) vi (3) ta c

a2 + 2a 1 x2 = a3 + a2 3a + 1

= (a 1) a2 + 2a 1
x2 = a 1
(do a > 1 nn a2 + 2a 1 > 0 ).
Thay x2 = a 1 vo (2) ta tm c a = 1, a = 3.
Bi 0.3. (Trch thi vo Trng Chuyn L Qy n Khnh Ha)
Gii h phng trnh

2
2
(1)
p y ) = 25
p (x xy)(xy
.
2
2
x xy + xy y = 3(x y) (2)
Li gii. T phng trnh (2) ta c x y > 0 nn x, y > 0. Hn na x = y khng l nghim
ca h, do (2) tng ng vi

x+ y =3 xy

x + y + 2 xy = 9(x y)

4x xy 5y = 0

( x + y)(4 x 5 y) = 0
16
y = x ( do x = y = 0 khng l nghim).
25


25 8
Thay vo (1) ta tm c nghim ca h: (x; y) =
;
.
6 3
Bi 0.4. (Trch thi vo Trng Chuyn L Qy n Vng Tu)
Cho a thc
f (x) = x2 + bx + c. Bit b, c l cc h s dng v f (x) c nghim. Chng minh

f (2) > 9 3 c.
Li gii. T gi thit f (x) c nghim ta c:

= b2 4c > 0 b2 > 4c b > 2 c.

V f (2) = 2b + c + 4 > 4 c + c + 4. Do , chng minh bi ton ta ch cn chng minh


c:

c + 4 c + 4 > 9 3 c.
(1)
chng minh (1) ta c th s dng bt ng thc C si hoc bin i tng ng. Chng hn
ta dng bt ng thc C si nh sau


c + 4 c + 4 = 2 c + c + 1 + (c + 1 + 1) > 2.3 3 c + 3 3 c = 9 3 c.

Bi 0.5. (Trch thi vo 10 Trng Chuyn L T Trng - Cn Th )


Cho a, b, c l di ba cnh ca mt tam gic tha mn 2ab + 3bc + 4ca = 5abc. Tm GTNN
ca biu thc
7
6
5
P =
+
+
.
a+bc b+ca c+ab
168

Tp ch Epsilon, S 09, 06/2016


3 4 2
Li gii. T gi thit ta c + + = 5. Nhn thy (a + b c)+(b + c a) = 2b, (b + c a)+
a b c
(c + a b) = 2c, (c + a b) + (a + b c) = 2a v 7 = 4 + 3, 6 = 4 + 2, 5 = 3 + 2 nn ta
phn tch
 
 


1
1
1
1
1
1
+
+3
+
+2
+
.
P =4
a+bc b+ca
a+bc c+ab
b+ca c+ab
p dng bt ng thc

1 1
4
+ >
x, y > 0, ta c
x y
x+y


16 12
8
3 4 2
P >
+
+
=2
+ +
= 10.
2b 2a 2c
a b c

ng thc xy ra khi a = b = c =

9
9
x=y=z= .
5
5

Vy GTNN ca P bng 10.


Bi 0.6. (Trch thi vo Trng Chuyn L Hng Phong, Nam nh)
Cho x, y, z l cc s thc tha mn (x y)(x z) = 1 v y 6= z. Chng minh:
1
1
1
+
+
> 4.
2
2
(x y)
(y z)
(z x)2
Li gii. Ta c
(x y) + (y z) + (z x) = 0,
nn t a = x y, b = x z ta c y z = a b v ab = 1. Khi
1
1
1
+ 2+
2
a
b
(a b)2


1
1
1
= ab
+ +
a2 b2 a2 + b2 2ab
a b
1
= + +
1 b
b a
+ 2
b a
1
a b
=t+
(vi t = + )
t2
b a
2
(t 3)
=
+ 4 > 4.
t2

y = x 5 1
2
ng thc xy ra chng hn khi

z = x 5 + 1
2
Vy GTNN ca P bng 4.
P =

Bi 0.7. (Trch thi vo Trng Chuyn KHTN Vng 2)


x2 1
y2 1
=
. Chng minh rng:
1) Vi x, y l nhng s nguyn tha mn ng thc
2
3
.
x2 y 2 ..40.
2) Tm tt c cc cp s nguyn (x; y) tha mn ng thc : x4 + 2x2 = y 3 .
169

Tp ch Epsilon, S 09, 06/2016


.
.
Li gii. 1) bi yu cu chng minh x2 y 2 ..40 = 8.5 nn ta cn chng minh x2 y 2 ..5 v
.
x2 y 2 ..8.
.
Ta bit: Nu x, y cng l th x2 y 2 ..8 nn ta i chng minh x, y l.
S dng tnh cht t l thc ta c:
x2 1
y2 1
=
= y 2 x2
2
3
x2 1
y2 1
v
l s nguyn.
2
3
y2 1
x2 1
l s chn do
l s chn do y 2 l s l.
Do x l s l, suy ra
2
3
.
Suy ra x2 y 2 ..8.
.
chng minh x2 y 2 ..5 ta chng x2 v y 2 c cng s d khi chia cho 5.
y2 1
x2 + y 2 2
x2 1
.
=
=
, t y ta c x2 + y 2 2..5 .
T gi thit ta suy ra
2
3
5
Do a2 0, 1, 1 (mod5) nn ta c

nn

.
x2 y 2 1 (mod5) x2 y 2 ..5.
2) Ta c
x4 + 2x2 = y 3 x2 + 1

2


= (y + 1) y 2 y + 1 .

Gi

d = y + 1, y 2 y + 1 = (y + 1, (y + 1)(y 2) + 3) = (y + 1, 3) d|3.
.
Do d|x2 + 1 v khng tn ti x x2 + 1..3 nn ta c d = 1.
T suy ra

y + 1 = m2
.
2
y y + 1 = n2
Mt khc, ta c y + 1 > 0 y > 0.
+) Vi y = 0 ta c x = 0.
+) Xt y > 0, ta c
2
m2 2 = y 2 2y + 1 < y 2 y + 1 = n2 < (y + 1)2 = m4 .
Suy ra n = m2 1 = y y 2 y + 1 = y 2 y = 1 (khng tha).
Vy x = y = 0 l cp nghim cn tm.
Bi 0.8. (Trch thi vo Trng Chuyn L Hng Phong Nam nh) Trn bng ban u ta
ghi s 2 v s 4. Ta thc hin cch vit thm cc s ln bng nh sau: nu trn bng c hai
s, gi s l a, b; vi a 6= b, ta vit thm ln bng s c gi tr l a + b + ab. Hi vi cch thc
hin nh vy, trn bng c th xut hin s 2016 c hay khng? Gii thch.
Li gii. Ta c a + b + ab + 1 = (a + 1) (b + 1), do cc s ghi trn bng ngoi hai s 2 v 4
th cc s khc khi cng thm 1 ta c mt hp s. Tuy nhin 2016 + 1 = 2017 l s nguyn t,
nn 2016 khng th xut hin trn bng.
170

Tp ch Epsilon, S 09, 06/2016


Bi 0.9. (Trch thi vo Trng Chuyn Ton PTNK)
Vi mi s nguyn dng m > 1, k hiu s (m) l c nguyn dng ln nht ca m v khc m.
Cho s t nhin n > 1, t n0 = n v ln lt tnh cc s
n1 = n0 s (n0 ) , n2 = n1 s (n1 ) , ..., ni+1 = ni s (ni ) , ..
Chng minh rng tn ti s nguyn dng k nk = 1 v tnh k khi n = 216 .1417 .
Li gii. Ta thy dy n0 , n1 , ... l dy gim, do n mt lc no gi tr ca dy bng 1.
Nn chng minh th nht ca bi ton ta c th dng phng php phn chng.
Nu khng tn ti k nk = 1 th dy cc s n1 , n2 , ..., ni+1 , ... l dy gim v cha v hn s
nguyn dng khng vt qu n. iu ny v l. Suy ra tn ti k nk = 1.
Ta phn tch n = 216 .1417 = 233 .717 .
Bng cch th mt s trng hp c bit ta rt ra c cc nhn xt sau:
NX 1: Vi n0 = 2t th n1 = t. Suy ra: Nu n0 = 2 th nt = 2t nn n = 1.
NX 2: Nu n0 = 3t, (t, 2) = 1 th n2 = t.
NX 3: Nu n0 = 7t, t khng c c nguyn t nh hn 7 th n4 = t. Suy ra n = 7 th n4 = 1.
T cc nhn xt trn ta c:
Vi n = 233 .717 th n33 = 717 nn n4.17+33 = 1 hay k = 4.17 + 33.
Bi 0.10. (Trch thi vo Trng Chuyn KHTN Vng 2)
Cho tp hp A = {1, 2, 3, ..., n}. Chng minh rng tn ti mt hon v ca A l
B = {a1 , a2 , a3 , ..., an }
sao cho vi mi 1 6 i < j < k 6 n th ai + ak 6= 2aj .
Li gii. Ta s chng minh bi ton bng quy np mt kt qu mnh hn l :
Cho tp hp A = {1, 2, ..., n}. Chng minh rng tn ti mt hon v ca A l B = {a1 , a2 , ..., an }
c xp trn ng trn sao cho trung bnh cng ca hai s bt k khng c t nm gia
hai s .
R rng yu cu 1 6 i < j < k 6 n th ai + ak 6= 2aj th hin tnh cht l trung bnh cng ca
hai s khng nm gia hai s v trong trng hp ny l xt khi cc s t trn mt ng
thng ; bi ton va pht biu r rng l mnh hn.
Ta chng minh rng vi mi n = 2m , m = 0, 1, 2, ... th c th sp xp cc s {1, 2, 3, ..., n} ln
ng trn sao cho iu kin c tho mn v s quy np theo m.
- Vi m = 0, hin nhin kt qu ng.


- Gi s (a1 , a2 , a3 , ..., a2m ) l mt cch sp xp tho iu kin. Xt cc s 1, 2, 3, ..., 2m+1 .
Ta c cch sp xp sau
(b1 , b2 , ..., b2m , ..., b2m+1 ) = (2a1 1, 2a2 1, ..., 2a2m 1, 2a1 , ..., 2a2m )
cng tho iu kin.
Thy vy, xt hai s bi , bj bt k. Nu bi , bj cng tnh chn l th theo gi thit quy np, trung
bnh cng ca bi , bj khng nm gia chng. Cn nu bi , bj khc tnh chn l th trung bnh
cng ca chng khng phi l s nguyn v tt nhin khng thuc dy cho.
Do , trong trng hp n = 2m+1 , ta in c tha mn bi nn theo nguyn l quy np
th c th sp xp vi mi s t nhin n c dng n = 2m .
R rng nu c th in s ng vi s a th vi b < a, ta cng c th xa bt mt s s v quy
tc ca bi vn c tha mn.
Nh th ta c th sp xp cc s vi mi s t nhin n. Ta c pcm.
171

Tp ch Epsilon, S 09, 06/2016


Bi 0.11. (Trch thi vo 10 Chuyn Ton khu vc TP H ni)
Cho 2017 s hu t dng c vit trn ng trn. Chng minh rng tn ti hai s cnh nhau
c vit trn ng trn sao cho khi b hai s th 2015 s cn li khng th chia thnh hai
nhm sao cho tng cc s mi nhm bng nhau.
Li gii. Ta thy nu b x1 , x2 , ..., x2017 tha yu cu bi ton th b kx1 , kx2 , ..., kx2017 cng
tha yu cu bi ton vi k l s hu t dng bt k. Do , ta ch cn chng minh bi ton vi
2017 s nguyn dng v trong c t nht mt s l (v cc s u l chn th ta chia mi
s cho 2).
+) Nu 2017 s u l th ta c th b hai s lin tip bt k v 2015 s cn li khng th chia
thnh hai nhm c tng cc phn t bng nhau (v tng ca 2015 s l s l).
+) Nu trong 2017 c t nht mt s chn th s tn ti hai s lin tip nhau khc tnh chn l v
hai s lin tip nhau cng tnh chn l. Do , nu tng ca 2017 s cho l s chn th ta xa
hai s lin tip nhau m khc tnh chn l, nu tng l s l th ta xa hai s lin tip nhau
cng tnh chn l. Khi tng ca 2015 s cn li l s l, nn khng th chia thnh hai nhm
c tng bng nhau.
Bi 0.12 (THPT chuyn KHTN 2016 vng 1). Cho tam gic ABC nhn ni tip ng trn
(O) vi AB < AC. Phn gic ca BAC ct BC ti D v ct (O) ti E khc A. M l trung
im ca on thng AD. ng thng BM ct (O) ti P khc B . Gi s cc ng thng
EP v AC ct nhau ti N .
1) Chng minh rng t gic AP N M ni tip v N l trung im ca AC.
2) Gi s ng trn (K) ngoi tip tam gic EM N ct ng thng AC ti Q khc N . Chng
minh rng B v Q i xng qua AE.
3) Gi s (K) ct ng thng BM ti R khc M . Chng minh rng RA RC.
P

N
O

Hnh 19.1

Li gii. 1) Do t gic ABEP ni tip nn M P N = BP E = BAE. M AE li l phn


gic ca BAC, do vy BAE = EAC = M AN . Suy ra M P N = M AN . Vy t
gic AP N M ni tip.
172

Tp ch Epsilon, S 09, 06/2016


Do P (O) nn AP B = ACB. Mt khc, AP M = AN M (do t gic AP N M ni
tip). Suy ra ACB = AN M . Do ta c M N k BC. M M li l trung im ca AD
nn theo tnh cht ng trung bnh suy ra N l trung im ca AC.
2) Do EM N Q v AP N M l cc t gic ni tip nn EQC = EM N = AP E = 180
ABE. Suy ra AQE = ABE, kt hp vi QAE = BAE ta c AEB = AEQ.
Ta c 4AEB = 4AEQ(g.c.g). T suy ra B, Q i xng qua AE.
3) Do ERM N v AP N M l cc t gic ni tip nn ERB = M N E = EAP = EBR.
Suy ra tam gic EBR cn ti E, kt hp vi EB = EC (do E l trung im cung BC khng
cha A), ta c ER = EC. Mt khc ta cng c REN = P M N = P AC = N EC.
T suy ra 4REN = 4CEN (c.g.c), do N R = N C = N A. Suy ra ARC = 90 hay
RA RC.
Nhn xt 1. y l bi ton mi v nm chung cho cc th sinh, 1) kh nh nhng 2)
kh hn mt cht v 3) c tnh phn loi. Cu trc bi cht ch vi 3 cu, 2) dng 1) v
3) dng c 1) v 2). Kt qu ny cng nm trong mt bi ton c ngha khi thu gn li
bi nh sau
Bi 0.13 (THPT chuyn KHTN 2016 vng 2). Cho tam gic ABC ni tip ng trn (O).
Phn gic AD ct (O) ti E khc A. N l trung im AC. ng trn ngoi tip tam gic
EM N ct M B ti R khc M . Chng minh rng RA RC.
Bi 0.14 (THPT chuyn KHTN 2016 vng 2). Cho hnh vung ABCD ni tip ng trn
tm (O). P l im thuc cung nh AD ca ng trn (O) v P khc A, D. Cc ng thng
P B, P C ln lc ct AD ti M, N . ng trung trc ca AM ct ng thng AC, P B ln
lt ti E, K. ng trung trc DN ct cc ng thng BD, P C ln lt ti F, L.
1) Chng minh rng ba im K, O, L thng hng.
2) Chng minh rng ng thng P O i qua trung im ca n thng EF .
3) Gi s ng thng EK ct ng thng BD ti S, cc ng thng F L v AC ct nhau ti
T , ng thng ST ct cc ng thng P B, P C ln lt ti U v V . Chng minh rng bn
im K, L, U, V cng thuc mt ng trn.
P
I
A

G M

E
F
O

T
U

C
Q

Hnh 19.2

173

Tp ch Epsilon, S 09, 06/2016


Li gii. 1) Gi G, H ln lt l trung im ca AM v DN . Ta c GK k AB( AM ) v
HL k DC( DN ). Do vy theo tnh cht ng trung bnh suy ra K, L tng ng l trung
im ca M B v N C. T y ta c OK v OL l ng trung bnh ca 4BM D v 4CN A.
Suy ra OK k M D v OL k AN . Do , K, O, L thng hng (tin Euclid).
2) Xt cc tam gic AM E v DN F ln lt cn ti E, F , c EAM = F DN = 45
nn 4AM E v 4DN F l cc tam gic vung cn, do M E OA v N F OD. t
M E ct P O ti I. p dng nh l Thales cho M N k BC v M I k BD( OA), ta c
PM
PN
PI
=
=
. Suy ra IN k OC v IN OD. T , I, N, F thng hng. Ta c
PO
PB
PC
OEIF l hnh ch nht. V vy IO i qua trung im EF . M I P O, nn ta c P O i qua
trung im EF .
3) t P O ct ST ti J. Do IM k OB v IN k OC nn IM P = OBP = OP M v
IN P = OCP = IP N , v vy IP = IM = IN hay I l tm ngoi tip ca 4P M N . Mt
khc, t M E k SB v KM = KB, p dng nh l Thales suy ra KE = KS v 4OES vung
cn ti O. Tng t F L = LT v 4OF T vung cn ti O. T ta c 4OEF = 4OST .
Suy ra OT S = OF E = P OF = 90 P OE = 90 JOT , v vy OJT = 90
1
hay P O ST . T suy ra P V U = 90 IP V = P IN = P M N = P KL (do I
2
l tm ngoi tip 4P M N v M N k KL). Do ta c t gic KLV U ni tip.
Nhn xt 2. y l bi ton mi v nm thi vo cc lp chuyn ton, 1) kh nh nhng,
2) kh hn mt cht v 3) c tnh phn loi. C 2) v 3) u i hi phi dng thm hnh
ph lm bi, mc ch ny lm thi tr nn rt khc bit so vi cc trng khc. Cu
trc bi cht ch vi 3 cu, 2) dng 1) v 3) dng 2). Kt qu ny cng nm trong mt
bi ton c ngha khi thu gn li bi nh sau
Bi 0.15. Cho hnh vung ABCD ni tip trong ng trn (O). P l mt im thuc cung nh
AD ca (O). P B, P C ln lt ct on AD ti M, N . Trung trc ca AM, DN ln lt ct
BD, AC ti S, T . ST ct P C, P B ln lt ti U, V . Chng minh rng ng trn ng knh
U V tip xc (O).
Bi 0.16 (THPT chuyn HSP 2016 vng 1). Cho ba im phn bit A, M, B thng hng v
M nm gia A v B. Trn cng mt na mt phng b l ng thng AB dng hai tam gic
u AM C v BM D. Gi P l giao im ca AD v BC.
1) Chng minh AM PC v BM P D
l cc t gic ni tip.
2) Chng minh rng CP.CB + DP.DA = AB
3) ng ni tm ca hai ng trn ngoi tip cc t gic AM P C v BM P D ct P A v P B
ti E v F . Chng minh CDF E l hnh thang.
Li gii. 1) Ta c M C = M A, M B = M D v CM B = AM D = 120 , do 4M CB =
4M AD(c.g.c). T suy ra M CP = M AP v M BP = M DP . Vy AM P C v
BM P D l cc t gic ni tip.
2) Do CM D = 180 AM C BM D = 60 = M BD nn CM tip xc vi ng
trn ngoi tip 4BM D. Ta c CM P = M BP , suy ra 4CP M 4CM B(g.g), suy ra
CP
CM
=
hay CM 2 = CP CB. Chng minh tng t ta cng c DM 2 = DP DA.
CM CB

Vy CP CB + DP DA = CM + DM = AM + BM = AB.
3) Gi I, J ln lt tm ngoi tip ca cc t gic AM P C v BM P D. Ta c EP M =
F P M = 60 nn E, F i xng qua P M v P EM F l hnh thoi. Do theo nh l Thales
174

Tp ch Epsilon, S 09, 06/2016


D

C
P
J
E
A

F
I
B

Hnh 19.3

AE
AE
AM
PF
AP
hay
. Mt khc ta c P IA = 2P M A = 2P DB =
=
=
=
AP
AB
PB
PF
PB
AP
AE
IA
P JB hay 4P IA 4BJP , suy ra
=
=
, suy ra 4IAE 4JP F (c.g.c).
JP
PB
PF
IE
IA
IC
T ta c
=
=
, kt hp CIE = DJF (do IC k JD( AB)), ta c
JF
JP
JD
4CIE 4DJF (c.g.c), suy ra CEI = DF J. Do vy, CE k DF hay CDF E l hnh
thang.
ta c

Nhn xt 3. Cu hnh ca ra khng mi nhng 3) mi v ph hp vi phn loi. Bi ton


c ba ng theo cu trc mt bi ton thi hnh hc r rng kt hp vi hau u d v 3)
phn loi ph hp cho mt thi chung. Kt qu ny cng nm trong mt bi ton c ngha
khi tng qut nh sau
Bi 0.17. Cho tam gic ABC vi D thuc on BC. E, F thuc on CA, AB sao cho
DE k AB, DF k AC. ng trn (K), (L) ngoi tip tam gic DCF v DBE ct nhau ti G
khc D. KL ct GB, GC ti M, N . Chng minh rng F M k EN .
Nhn xt 4. Cc nm gn y, phn hnh hc trong mt s thi vo cc lp chuyn ton cc
trng chuyn c truyn thng c nhiu bi ton hnh mi c ni dung khc cc thi vo lp
10 thng thng. L mt gio vin theo st i tuyn v c tham gia vo cng vic bi dng
hc sinh gii th ti cng c mt i li bn. Vic cc trng i mi thi theo phong cch st
hn cho cc i tng hc sinh gii khng lm mt i tnh truyn thng ca thi cp 2 vo cp
3, v mc ch chnh ca thi vo trng chuyn l lc ra hc sinh gii ton. Nh kho st qua
im s mt s trng th vn c lun im 10 ton chuyn v ph 8,9 kh cao, iu cho
thy thi vn ph hp. Mt khc nh p n cng b th r rng cc bi ton nm hon
ton trong chng trnh cp 2. Mt s kin cho rng cc bi ton ph hp hn khi gii trn
my tnh vi cc cng c hnh hc mnh c trn mng, liu rng c mt tnh khch quan khi bt
hc sinh phi t v hnh v gii. Tr li cho kin ny kh n gin, bn thn ti l mt ngi
hay ra hnh th i vi ti my tnh hon ton ch l cng c gip con ngi ch chc
chn khng thay con ngi ra . Chng hn khi bn cng hoc nhn hai s ln mt thi
gian th bm my tnh b ti, i vi cc bi ton hnh cng vy, nu thiu thi gian v tay cn
chnh tay th dng my cho mt thi gian. Nhng mt hnh hay l hon ton nm tnh
cht hnh m ngi ra t c v mun kim tra kin thc ca hc sinh. S thc rng
mt hc sinh c t duy hnh tt th cng khng cn v hnh bng thc v compa cng c th
175

Tp ch Epsilon, S 09, 06/2016


ngh ra li gii v vit li. Tt nhin rng vi cc bi ton phc tp nh hin nay th s tng
tng l cha m phi nhn hnh v, nhng iu khng c ngha l my tnh thay th
con ngi, n ch ng vai tr h tr. Vy nn ci vic c my tnh hay khng c my tnh khi
gii hnh cng khng quan trng bng ngi gii c t duy hnh hc sc xo ti u.
Li gii 3 bi ton trn l ca em Trnh Huy V, hc sinh lp 12 A1 Ton, THPT chuyn KHTN.
Phn nhn xt l ca Trn Quang Hng, gio vin THPT chuyn KHTN.

176

Tuyn chn cc thi Olympic nm 2016 dnh


cho hc sinh THCS
(Ban bin tp)

thi 1. (EGMO 2016) Cho n l s nguyn dng l v x1 , x2 , . . . , xn l cc s thc khng


m. Chng minh rng
min (x2i + x2i+1 ) 6 max (2xj xj+1 ),
16i6n

16j6n

trong xn+1 = x1 .
thi 2. (Saudi Arabia TST 2016, Junior) Cho a, b, c l cc s thc dng tha mn iu kin
a + b + c = 3. Chng minh rng
a2

1
1
4
+
> .
2
2
+b +c
3
abc

thi 3. (HSP HN, tuyn sinh 10) Cho a, b, c l cc s thc khng m tha mn iu kin
a + b + c = 1. Chng minh rng ta c bt ng thc

5a + 4 + 5b + 4 + 5c + 4 > 7.
thi 4. (AMC 2016, 10B) Gi s f (x) =

10
P

([kx] k[x]) trong [x] k hiu s nguyn ln

k=2

nht khng vt qu x. Nu x > 0 th f (x) c th nhn c bao nhiu gi tr?


thi 5. (APMOPS 2016) Bit s a1 = 1000, a2 = 1016 v a3 =
gi tr ca ba15 c trong bxc l phn nguyn ca x.

a1 +a2
,
2

a4 =

a2 +a3
,...
2

Tnh

thi 6. (Tournament of the towns 2016) Tt c cc s nguyn t 1 n 1000000 c vit trn


mt bng giy theo mt th t no . Sau bng giy c ct thnh cc mu cha hai ch s
lin tip. Chng minh rng cc mu giy ny lun cha tt c cc s c hai ch s, cho d th
t ban u ca cc s l nh th no.
thi 7. (SASMO 2016, G8) T 72 s 1, 2, 3, . . . , 72 c th chn nhiu nht bao nhiu s sao
cho trong cc s c chn, khng c s no c tch bng 72?
thi 8. (Moscow MO 2016, G8) Tm s nguyn dng nh nht l bi ca 99 m trong cch
vit thp phn ca n ch c cc ch s chn.
thi 9. (PTNK 2016, tuyn sinh lp 10) Lp 9A c 27 hc sinh nam v 18 hc sinh n. Nhn
dp sinh nht bn X (l mt thnh vin ca lp), cc bn trong lp c rt nhiu mn qu tng
X. Ngoi ra mi bn nam ca lp lm 3 tm thip v mi bn n xp 2 hoc 5 con hc tng
bn X. Bit s tm thip v s con hc bng nhau, hi bn X l nam hay n?
thi 10. (Purple Comet 2016) C 16 im c xp vo li 4 4 nh hnh v.
177

Tp ch Epsilon, S 09, 06/2016

Khong cch gia hai im l s bc i t nht theo chiu ngang v chiu dc theo cc ng
li i t im ny n im kia. V d hai im cnh nhau c khong cch 1 cn hai im
hai gc i nhau ca li c khong cch 6. Gi khong cch trung bnh gia hai im khc
nhau ca li l m
vi m, n l cc s nguyn dng nguyn t cng nhau. Hy tm m + n.
n
thi 11. (Moscow MO 2016, G7) Mt gc ca tam gic bng 60 v cnh i din vi gc
ny bng mt phn ba chu vi tam gic. Chng minh tam gic l tam gic u.
thi 12. (Tournament of the towns 2016) Cho tam gic nhn ABC vi C = 60 . Gi H l
trc tm tam gic. ng trn tm H bn knh HC ct cc ng thng CA v CB ti cc
im th hai M v N tng ng. Chng minh rng ng thng AN v BM song song hoc
trng nhau.
thi 13. (Moscow MO 2016, G8) im O l tm ng trn ngoi tip tam gic nhn ABC.
ng thng vung gc vi cnh AC ct on BC v ng thng BC cc im Q v P
tng ng. Chng minh rng cc im B, O v trung im cc on AP v CQ nm trn mt
ng trn.
thi 14. (EGMO 2016) Cho t gic ABCD ni tip. Hai ng cho AC v BD ct nhau
ti X. Gi s C1 , D1 , M ln lt l trung im ca CX, DX v CD. ng thng AD1 v
BC1 ct nhau ti Y, M Y ct AC v BD ti im th hai E, F tng ng. Chng minh rng
XY tip xc ng trn i qua E, F, X.
thi 15. (SASMO 2016, G7) Trong hnh di y, ABCD l hnh vung, bit CDE = 72
v CED = 54 .

178

Tp ch Epsilon, S 09, 06/2016


Tm gc EAD.
thi 16. (APMOPS 2016) Bn ng trn bng nhau cng i qua mt im v tip xc vi
ng trn ln. Bit rng bn knh ca ng trn ln bng 14.

Ly =

22
.
7

Tnh din tch phn t m.

thi 17. (SASMO 2016, G9) Cho ABCD l t giy hnh ch nht vi AB = 18 v AD = 12.
im E l im trn CD sao cho DE : CD = 1 : 2. Hnh ch nht ABCD c gp li sao
cho A trng vi E.

Tnh chiu di on gp.


thi 18. (Saudi Arabia TST 2016, Junior) Cho t gic ABCD ni tip ng trn (O). AC
ct BD ti P. Gi Q, R l hai im trn cung ACD khng cha cc im A, B. Cc ng
thng RA, RC ct ng trn ngoi tip tam gic P QR ti cc im th hai L, K. P K, P L
ct BC, AD ti M, N tng ng. Chng minh rng BC v AD ct nhau ti im nm trn
ng trn ngoi tip tam gic QM N.
thi 19. (Moscow MO 2016, G7) Hy thay vo cc du su ch s phn bit sao cho cc
phn s u ti gin v ng thc ng

+ = .

thi 20. (SASMO 2016, G8) Trong php tnh di y, cc ch ci khc nhau i din cho
cc ch s khc nhau.
S

179

O
3
S

Tp ch Epsilon, S 09, 06/2016


Hy tm s c 5 ch s MATHS.
thi 21. (SASMO 2016, G9) Trong php tnh di y, cc ch ci khc nhau i din cho
cc ch s khc nhau.

S
C
A

T A
O O
S M

Y
L
O

Hy tm s c 5 ch s SASMO.
thi 22. (AMC 2016, 10A) Php ton c cc tnh cht a (b c) = (a b) c v a a = 1
vi mi cc s thc khng m a, b v c ( y biu th php nhn). Nghim ca phng trnh
2016 (6 x) = 100 c vit di dng pq , trong p v q l cc s nguyn dng nguyn t
cng nhau. Hy tnh p + q.
thi 23. (IMC 2016, 2nd round, G8) P (x) l mt a thc bc 8 tha mn iu kin
P (1) = P (1), P (2) = 6 + P (2), P (3) = 24 + P (3),
v P (4) = 60 + P (4). Hy tnh gi tr ca P (5)P (5).
thi 24. (Purple Comet 2016) a thc bc ba p(x) v q(x) tha mn cc iu kin
p(1) = q(2), p(3) = q(4), p(5) = q(6),
v p(7) = q(8) + 13. Hy tm p(9) q(10).
thi 25. (Saudi Arabia TST 2016, Junior) Cho k l s nguyn dng. Chng minh rng tn
ti cc s nguyn x, y khng s no chia ht cho 3 sao cho x2 + 3y 2 = 3k .
thi 26. (Saudi Arabia Training Camp 2016) Cho S l tp hp tt c cc s t nhin biu din
c di dng x2 + 3y 2 vi x, y l cc s nguyn, tc l S = {n N | n = x2 + 3y 2 trong
x, y l cc s nguyn }. Chng minh cc tnh cht sau y ca S :
i) Nu m S, n S th m n S.
ii) Nu N S v 2 | N th

N
4

S. Nu N S, s nguyn t p S v p | N th

N
p

S.

iii) Cho p l s nguyn t dng 3k + 1. Chng minh rng tn ti s nguyn N sao cho N 2 + 3
chia ht cho p. T suy ra p S.
iv) Tm iu kin cn v mt s nguyn dng N > 1 thuc S.
thi 27. (SASMO 2016, G7) C bao nhiu cch sp xp cc ch ci ca t SASMO sao cho
hai ch S xp cnh nhau?
thi 28. (APMOPS 2016) C 6 ch A, P, M, O, P, S. Mt ci my tnh sp xp cc t theo
th t trong t in: AM OP P S, AM OP SP, AM OSP P, . . . , SP P OM A. Hi t P OAM SP
v tr th bao nhiu?
180

Tp ch Epsilon, S 09, 06/2016


thi 29. (Moscow MO 2016, G6) mt thnh ph nh ch c mt tuyn tu in. N chy
vng trn v tu in chy theo vng trn theo c hai pha. Trn vng c 3 bn l Rp xic, Cng
vin v S th. T Cng vin n S th th ng i qua Rp xic di gp 3 ln ng i khng
qua Rp xic. T Rp xic n S th th ng i qua Cng vin ngn bng mt na ng i
qua ng i khng qua Cng vin. Hi ng i no t Cng vin n Rp xic ngn hn
Qua S th hay khng qua S th v ngn hn bao nhiu ln?
thi 30. (Moscow MO 2016, G8) C 2n vin si (n nguyn dng cho trc) ang c xp
thnh 3 ng. Mi ln thc hin cho php ly mt na s vin si t ng si c chn vin si v
chuyn sang ng khc. Chng minh rng, d cc ng si ban u c xp th no, sau mt
s hu hn bc thc hin nh th, ta c th to ra mt ng si c ng n vin si.
thi 31. (Moscow MO 2016, G8) Xung quanh mt bn trn c 10 ngi ang ngi, mi ngi
trong h hoc l hip s, l ngi lun ni tht v k ni di l ngi lun ni di. C hai ngi
trong h tuyn b C hai ngi ngi cnh ti u l k ni di cn tt c nhng ngi cn li
u ni C hai ngi ngi cnh ti u l hip s. Hi c bao nhiu hip s trong s 10 ngi
ny (lit k tt c cc phng n c th v chng minh khng cn phng n no khc).
thi 32. (Gulf MO 2016) Gi s c 4 ngi A, B, C v D nh tennis i vi nhau. H c
th t chc cc trn u nh sau: Trn u A v B u vi C v D, trn tip theo A v C nh
vi B v D, cui cng A v D nh vi B v C. Ci hay ca cch sp xp ny l hai iu kin
sau c tha mn:
a) Hai cy vt bt k chung i vi nhau ng 1 ln.
b) Hai cy vt bt k u hai i khc nhau ng 2 ln.
Hi c th sp xp cc trn u sao cho cc iu kin a) v b) c tha mn trong cc trng
hp sau? Gii thch r cu tr li.
i) C 5 ngi chi.
ii) C 7 ngi chi.
iii) C 9 ngi chi.
thi 33. (IMC 2016, 2nd round, G6) ng A c khong hn 1000 con g. ng A m s g
ca mnh ri bo cho B. ng B li bo cho C. ng C bo cho D v D li bo cho E. Tr tru
l tt c u nhm ln hoc c tnh nhm ln: A m tha 9 con g so vi thc t. B trc khi
bo cho C i ch hai ch s cui cng. C i ch ch s u tin v ch s th ba trc
khi bo cho D. D nhn i s nhn c ri mi bo cho E. K l thay, s m E nhn c li
ng bng s g m A c. Hi A c bao nhiu con g?
thi 34. (IMC 2016, 2nd round, G8) mt khu th c 12 con ng, gm 6 con ng
ngang song song vi nhau v 6 ng dc, cc ng dc u song song vi cc ng ngang.
12 con ng to thnh 36 ng t (ni giao nhau gia cc ng dc v ng ngang). Mt
ngi i b ang khng ng ng t no, v anh ta c nh s i qua tt c cc ng t ri tr
v ni ang ng. Hi anh ta s phi r t nht bao nhiu ln?
181

Tp ch Epsilon, S 09, 06/2016


thi 35. (Purple Comet 2016) C 10 vin gch lt nn c sp thnh mt hng dc, mi vin
c th c sn bng mt trong 4 mu (), vng (V), xanh (X), v trng (T). Tm s cch sn
sao cho mi mt blc 5 vin gch lin tip u cha 4 mu. V d cch t TXTVXTV v
TTXVTVXT th c nhng TXVVXTTV th khng c v 5 vin gch lin nhau XVVXT
khng cha mu .
thi 36. (AMC 2016, 10B) Mt s i bng thi u vng trn 1 lt, hai i bt k u vi
nhau ng 1 trn. Mi i thng 10 trn v thua 10 trn, khng c ha. Hi c bao nhiu b 3
i {A, B, C} m A thng B, B thng A, C thng A?
thi 37. (SASMO 2016, G6) Mt chic xe mt 3 gi 41 pht i t thnh ph A n thnh
ph B v quay v t B v A. Xe i xung dc vi vn tc 6km/h (kilmt/gi), trn ng bng
vi vn tc 5km/h v i ln dc vi vn tc 4km/h. on ng t A n B c 4km ng bng.
Tnh khong cch gia hai thnh ph.
thi 38. (SASMO 2016, G7) Khong cch gia nh ca Tom v nh ca Ben l 36km. Vo lc
11 gi sng, Tom v Ben bt u i n nh ca nhau. C sau 10 pht chy nhanh vi vn tc
5m/s (mt/ giy), Tom li chy chm vi vn tc 0.5m/s trong 4 pht cn Ben th c sau 15 pht
i xe p vi vn tc 5m/s li ngh 3 pht. Hi h gp nhau vo lc my gi?

182

LI GII THI TON QUC T


FORMULA OF UNITY
THE THIRD MILLENNIUM
(Ban bin tp)

Tip theo Epsilon s 6, Ban bin tp xin gii thiu vi bn c li gii ca thi
ca k thi Formula of Unity. Phn 1 gm cc ca khi lp 5, 6, 7, 8. Cc khi
cn li s c gii thiu vo k ti.

1. thi cho khi lp R5


Bi 1. Peter, Basil v Anatoly gp tin tit kim mua mt qu bng gi 9 la. Bit rng, s
tin gp ca mi ngi khng nhiu hn mt na tng s tin gp ca hai ngi cn li. Hi
Peter ng gp bao nhiu tin?
Li gii. Nu s tin ca ba bn khng bng nhau th phi c ngi c c nhiu tin nht. Gi s
ngi l Peter. Khi , s tin ca Peter s nhiu hn trung bnh cng s tin ca hai bn cn
li. Tuy nhin, theo bi th s tin ca Peter li khng nhiu hn na tng s tin ca hai bn
cn li. Do , s tin ca 3 bn gp l bng nhau v bng 3 la.
Bi 2. Pauline vit hai s A v B ln bng. Victoria xa hai s A; B v vit tng C v tch D ca
chng. Sau , Pauline li xo hai s C v D; thay bi tng E v tch F ca chng. Bit rng mt
trong hai s E v F l s l. Hi l s no?
Li gii. Theo bi th:
C =A+B
D = AB
E = A + B + AB
F = (A + B)AB
Ta thy F lun l s chn, v nu trong 2 s A, B c mt s chn th F chn; cn nu A, B cng
l th A + B chn v F cng chn.
Do , nu mt trong hai s E, F l th s ch c th l E (chng hn khi A chn, B l).
Bi 3. Ta ni rng hc sinh A hc tt hn hc sinh B nu im ca A cao hn im ca B trong
phn ln cc bi kim tra. Sau 3 bi kim tra, thy gio nhn xt rng hc sinh A hc tt hn
hc sinh B, hc sinh B hc tt hn hc sinh C v hc sinh C li hc tt hn hc sinh A. Liu iu
ny c th xy ra khng?
183

Tp ch Epsilon, S 09, 06/2016


Li gii. iu ny c th xy ra. Ta a ra mt v d trong bng sau: A B C 1 9 8 10 2 10 9 8 3 8
10 9
Ghi ch: bi ny c l ly tng t IMO Shortlist 2014 nh sau: C mt b bi c v s
qun m mi qun c vit bi 1 s thc; ngoi ra, vi mi s thc x th ch c ng mt qun
c vit s x.
Hai ngi chi la chn ty 2 tp hp ri nhau A, B m mi tp gm 100 qun bi t b bi
trn. Nhim v ca chng ta l chn mt lut chi no cn c trn cc qun bi chn, c
th xc nh ai l ngi thng cuc. c bit, lut li phi tha mn mt s rng buc nh sau:
1. N ch da trn vic so snh cc s c vit trn hai b 100 qun bi (khng quan tm
n gi tr c th ca chng l bao nhiu).
2. Nu lit k cc tp hp theo th t tng dn: A = {a1 , a2 , ..., a100 } v B = {b1 , b2 , ..., b100 }
Ngoi ra, ta c ai > bi vi i = 1, 2, 3, ..., 100 th khi A phi thng B.
3. Nu c 3 ngi chi ln lt chn c ba b 100 qun bi l A, B, C m b A thng b
B, b B thng b C th b A cng phi thng b C.
Hi c tt c bao nhiu cch nh ngha lut chi nh th?
Hai lut chi l khc nhau nu tn ti 2 tp hp A, B m A thng B trong lut ny, nhng li
thua B trong lut kia.
Bi 4. Nu Leon b im km trng, cu ta s dnh c bui ti ni di m. Cn nu ngc
li, cu ta s lun ni tht. Leon c mt c em gi lun c m cho ko nu nh hm c b
khng b im km. Mt bui ti, Leon ni vi m: "Hm nay con b nhiu im km hn em".
Hi c em gi ca Leon c c m cho ko hay khng?
Li gii. Cu ni ca Leon khng th l cu ni tht. Suy ra Leon b im km v em gi Leon
c s im km bng hoc nhiu hn Leon. Suy ra c gi ca Leon khng c m cho ko. Cu
tr li l ph nh.
Bi 5. Mt t lch ma thut ch ng ngy tt c cc ngy chn ca thng v sai ngy vo tt c
cc ngy l. Hi s ln nht cc ngy lin tip nhau c ghi cng ngy trn lch l bao nhiu?
V ngy c th l ngy no trong thng?
Li gii. S ln nht tha mn l 4. l cc ngy 31, 1, 2, 3 v c ghi l ngy 2.
Bi 6. C bao nhiu s c biu din thp phn gm 10 ch s khc nhau v c cha on 0123?
Li gii. Coi 0123 l 1 ch s, cn 4, 5, 6, 7, 8, 9 l 6 ch s khc. V 0123 khng c xp u
nn c 6 cch chn v tr cho 0123. Sau c 6! cch xp 6 ch s cn li vo 6 ch cn li. p
s l 6.6! = 4320.
Bi 7. Hnh vung 8 8 c v trn t giy k vung dc theo cc ng k. Alex ct hnh
vung 8 8 theo cc ng k thnh 7 phn vi chu vi bng nhau. Hy ch ra mt cch ct ca
Alex.
Li gii. Cch ct m t nh hnh bn di:
184

Tp ch Epsilon, S 09, 06/2016

2. thi cho khi lp R6


Bi 1. C 14 ngi ngi quanh mt vng trn. Peter, Victoria, Anatoly v Genghis ang ngi
cnh nhau theo th t v cc bn c cc ng xu mnh gi 1, 2, 5 v 10 rp. Mt ngi bt k c
th a mt ng xu ca mnh cho ngi bn tri hoc bn phi ca mnh nu c ng 3 ngi
ngi gia h. Sau mt lc chuyn nh vy, cc bn Peter, Victoria, Anatoly v Gengis li nhn
c cc ng xu. Hi lc ny, bn no ang cm ng xu no? Hy ch ra tt c cc kh nng
v chng minh khng cn trng hp no khc.
Li gii. Ta thy ngi s 1 s c th ln lt chuyn tin cho ngi th 5, 9, 13, 3, . . . tc l
nhng ngi v tr l s chuyn tin cho nhau. Tng t, nhng ngi v tr chn chuyn tin
cho nhau. Suy ra cc phng n trn l tt c cc phng n tha mn. 1, 2, 5, 10; 1, 10, 5, 2; 5,
2, 1, 10, 5, 10, 1, 2.
Bi 2. Pauline vit hai s A v B ln bng. Victoria xa hai s A; B v vit tng C v tch D ca
chng. Sau , Pauline li xo hai s C v D; thay bi tng E v tch F ca chng. Bit rng mt
trong hai s E v F l s l. Hi l s no?
Li gii. Xem li gii ca phn trc.
Bi 3. Ta ni rng hc sinh A hc tt hn hc sinh B nu im ca A cao hn im ca B trong
phn ln cc bi kim tra. Sau 3 bi kim tra, thy gio nhn xt rng hc sinh A hc tt hn
hc sinh B, hc sinh B hc tt hn hc sinh C v hc sinh C li hc tt hn hc sinh A: Liu iu
ny c th xy ra khng?
Li gii. Xem li gii ca phn trc.
185

Tp ch Epsilon, S 09, 06/2016


Bi 4. Nu Leon b im km trng, cu ta s dnh c bui ti ni di m. Cn nu ngc
li, cu ta s lun ni tht. Leon c mt c em gi lun c m cho ko nu nh hm c b
khng b im km. Mt bui ti, Leon ni vi m: "Hm nay con b nhiu im km hn em".
Hi c em gi ca Leon c c m cho ko hay khng?
Li gii. Xem li gii ca phn trc.
Bi 5. Mt t lch ma thut ch ng ngy tt c cc ngy chn ca thng v sai ngy vo tt c
cc ngy l. Hi s ln nht cc ngy lin tip nhau c ghi cng ngy trn lch l bao nhiu?
V ngy c th l ngy no trong thng?
Li gii. Xem li gii ca phn trc.
Bi 6. C bao nhiu s c biu din thp phn gm 10 ch s khc nhau v c cha on 0123
hoc on 3210?
Li gii. Kt qu l 6.6! + 7! = 9360. Lp lun tng t bi 6 ca lp 5.
Bi 7. Hnh vung 8 8 c v trn t giy k vung dc theo cc ng k. Alex ct hnh
vung 8 8theo cc ng k thnh 7 phn vi chu vi bng nhau. Hy ch ra mt cch ct ca
Alex.
Li gii.
Xem li gii ca phn trc.

3. thi dnh cho khi lp R7


Bi 1. Mt t lch ma thut ch ng ngy tt c cc ngy chn ca thng v sai ngy vo tt c
cc ngy l. Hi s ln nht cc ngy lin tip nhau c ghi cng ngy trn lch l bao nhiu?
V ngy c th l nhng ngy no?
Li gii. Xem li gii ca phn trc.
Bi 2. Hy in vo cc ca bng vung 5 5 cc s nguyn dng phn bit sao cho tng
cc s trn mi hng, mi ct l bng nhau v nh nht c th. Bit rng, cc s 1, 2, 3, 4, 2015
c in trc trn mt ng cho.

186

Tp ch Epsilon, S 09, 06/2016


Li gii. Do cc s l phn bit nn tng 8 s cng hng hoc cng ct vi 2015 ti thiu l
5+6+ +12 = 68. Suy ra tng chung ca cc hng, cc ct phi t nht bng 2015+34 = 2049.
V ta c th sp c nh sau:

Bi 3. Hnh vung 8 8 c v trn t giy k vung dc theo cc ng k. Alex ct hnh


vung 8 8 theo cc ng k thnh 7 phn vi chu vi bng nhau. Hy ch ra mt cch ct ca
Alex.
Li gii. Xem li gii ca phn trc.
Bi 4. C 27 con gin tham gia mt cuc chy ua. Trong mi vng s c ba con gin chy. Mi
con gin chy vi tc c nh, khng i gia cc vng ua, v tc ca cc con gin l i
mt khc nhau. Sau mi vng, ngi ta ghi li th t v ch ca cc con gin tham gia vng
ua . Hi 14 vng ua c xc nh chnh xc theo th t hai con gin chy nhanh nht
khng?
Li gii. Cu tr li l khng nh.
Ta dng 9 vng u loi 9 con chm nht. 3 vng tip theo chn ra con nhanh nht trong 9 con
nhanh nht v loi 3 con chm nht. Vng 13 chn ra con nhanh nht (v ch) trong 3 con nhanh
nht. Lc ny ch cn 3 con c th ng nh l con ng nh vng 13, con ng nh trong cuc
ua vi con v ch vng 3 trn v con ng nh cuc ua vi con v ch 9 vng u.
Dng trn 14 tm ra con th nh t 3 con ny.
Bi 5. Ta ni rng hc sinh A hc tt hn hc sinh B nu im ca A cao hn im ca B trong
phn ln cc bi kim tra. Sau 3 bi kim tra, thy gio nhn xt rng hc sinh A hc tt hn
hc sinh B; hc sinh B hc tt hn hc sinh C v hc sinh C li hc tt hn hc sinh A: Liu iu
ny c th xy ra khng?
Li gii. Xem li gii ca phn trc.
Bi 6. Ta ni mt s nguyn dng l p nu n l tch cc giai tha ca cc s nguyn t
(khng nht thit phi phn bit). Ta gi mt s hu t dng l tt nu n l t s gia hai s
nguyn dng p. Chng minh rng tt c cc s hu t dng u tt.
187

Tp ch Epsilon, S 09, 06/2016


Li gii. Trc ht, ta thy rng nu mt s hu t dng l tt th tch v thng ca chng cng
u tt.
Ngoi ra, mi s nguyn dng n u c th vit thnh

n!
.
(n1)!

T , ta a bi ton v chng minh mi s nguyn dng u tt v ta s thc hin iu ny


bng quy np.
Vi n = 1, ta c 1 =

2!
2!

Vi n = 2, ta c 2 =

2!2!
2!

Vi n = 3, ta c 3 =

3!
2!

l s tt.
cng l s tt.

cng l s tt.

Khi , vi n > 4, nu n l hp s, ta c th vit n thnh tch ca cc s nguyn t nh hn v


theo gi thit quy np, n cng l s tt.
Nu n > 4 l s nguyn t, ta vit n =
suy ra n l s tt.

n!
(n1)!

v n 1 l hp s, khi n cng l s tt nn

Theo nguyn l quy np th nhn xt c chng minh. Bi ton c gii quyt.


Chng hn 5 =

5!
4!

5!
234

5!
23 3

5!
(2!)3 3!
2!

5!
.
(2!)2 3!

Bi 7. Ta gi mt s nguyn dng l p nu dy cc ch s ca n tng thc s, v d 1589 l


s tng cn 447 th khng. Hy tm s nh nht cc s nguyn dng p vi tng l 2015.
Li gii. Ta s cn 3 s l: 1789 + 189 + 37 = 2015. R rng bn thn s 2015 khng p nn
khng th dng 1 s biu din.
Ta s chng minh rng khng th dng 2 s biu din c 2015.
Gi s x + y = 2015 v x < y (hai s ny khng th bng nhau v 2015 l). D thy rng s p
ln nht khng vt qu 2015 l 1789. Suy ra x < y 6 1789.
D thy nu c hai s cng c 4 ch s tr ln th khng tha nn ta c th t x = abc, y = 1def ,
trong 0 < a < b < c, 1 < d < e < f. Ta thy abc + 1def = 2015. Ta thy c + f = 5
hoc c + f = 15, nhng nu c + f = 5 th c, f qu nh, khng tha. Th nn c + f = 15 v
dn n b + e = 9 (v php tnh trc c nh, v kt qu phi tn cng l 0). Khi php tnh
a + d = 9 (tng t trn, php tnh trc c nh v kt qu phi tn cng l 0). Nhng khi , ta
c b + e = a + d, mu thun v a < b, d < e. Do , khng tn ti 2 s p c tng l 2015. p
s l 3. Ghi ch. S ln nht khng vt qu 2015 tha mn tnh cht: l tng ca 2 s p l s
1978. Khi : 1978 = 189 + 1789.

4. thi dnh cho Khi lp R8


Bi 1. Hy in vo cc ca bng vung 5 5 cc s nguyn dng phn bit sao cho tng
cc s trn mi hng, mi ct l bng nhau v nh nht c th. Bit rng, cc s 1, 2, 3, 4, 2015
c in trc trn mt ng cho.
Li gii. Xem li gii ca phn trc.
188

Tp ch Epsilon, S 09, 06/2016


Bi 2. C 27 con gin tham gia mt cuc chy ua. Trong mi vng s c ba con gin chy. Mi
con gin chy vi tc c nh, khng i gia cc vng ua, v tc ca cc con gin l i
mt khc nhau. Sau mi vng, ngi ta ghi li th t v ch ca cc con gin tham gia vng
ua . Hi 14 vng ua c xc nh chnh xc theo th t hai con gin chy nhanh nht
khng?
Li gii. Xem li gii ca phn trc.
Bi 3. Hy tm mt s nguyn dng sao cho tch cc c t nhin ca n l 1090 .
Li gii. Ta tm s di dng 2m 5n . Cc c ca s ny c dng 2a 5b vi 0 6 a 6 m, 0 6
mn(m+1)(n+1)
4
nn ta a v mn(m + 1)(n + 1) = 360. Chn
b 6 n. T tch cc c bng 10
m = 3, n = 5 th ta c mt s tha mn yu cu bi ton l 23 55 = 25000.
Bi 4. John c 12 que g vi di mi que l mt s nguyn dng khng vt qu 56. Chng
minh rng John c 3 que c th to thnh mt tam gic.
Li gii. Gi s ngc li, khng c 3 que to thnh tam gic. Xp th t chiu di cc que
g a1 6 a2 6 ... 6 a12 th t gi thit, ta s suy ra an+2 > an+1 + an . T y ln lt suy ra
a3 > a2 + a1 > 2, a4 > a3 + a2 > 3, a5 > a4 + a3 > 5 C nh th a6 > 8, a7 > 13, a8 >
21, a9 > 34, a10 > 55, a11 > 89. iu ny mu thun, suy s John s lun to c tam gic.
Bi 5. Ta ni mt s nguyn dng l p nu n l tch cc giai tha ca cc s nguyn t
(khng nht thit phi phn bit). Ta gi mt s hu t dng l tt nu n l t s gia hai s
nguyn dng p. Chng minh rng tt c cc s hu t dng u tt.
Li gii. Xem li gii ca phn trc.
Bi 6. Cho tam gic ABC vi B = 30 , C = 105 v D l trung im on thng BC.
Tm gc BAD? Li gii. H CH vung gc vi AB th suy ra CHD l tam gic u v AHC l
tam gic vung cn ti H. T y suy ra tam gic AHD cn ti H v
BAD = HAD = 15 .
Bi 7. Ta ni rng hc sinh A hc tt hn hc sinh B nu im ca A cao hn im ca B trong
phn ln cc bi kim tra. Sau 3 bi kim tra, thy gio nhn xt rng hc sinh A hc tt hn
hc sinh B; hc sinh B hc tt hn hc sinh C v hc sinh C li hc tt hn hc sinh A. Liu iu
ny c th xy ra khng?
Li gii. Xem li gii ca phn trc.

189

Tp ch Epsilon, S 09, 06/2016

190

CC VN C IN V HIN I
Trn Nam Dng - i hc Khoa hc T nhin - HQG TP.HCM

LI GII THIU
Chuyn mc ny dnh cho cc vn c in v hin i c trnh by di
dng cc bi ton xu chui. c th l chui cc bi gii bi ton ng chu,
2
chng minh ng thc Euler k diu 1 + 212 + 312 + = 6 , mt chui bi ton
vn tr ... Cch trnh by xut pht t nhng vn n gin, d hiu, nhng khi
nim mi s c nh ngha lun trong bi c th c tng i c lp. V mi
mt chui bi s nu ra nhng vn nht nh, c th l gii quyt mt bi ton
kinh in hay nu ra nhng gi thuyt mi, nhng vn mi. Li gii v tho lun
v cc bi ton s c ng s N + 3.
Trong s ny, chng ti xin gii thiu vi bn c hai thi chnh thc v hai
ngh dnh cho hc sinh ph thng trong k thi Olympic Ton hc sinh vin v
hc sinh ton quc din ra t ngy 11/4/2016 - 17/4/2016 ti thnh ph Quy Nhn,
tnh Bnh nh.

191

Tp ch Epsilon, S 09, 06/2016

thi chnh thc


Ch : i s
Mc tiu ca bi thi ny l tm hiu mt s trng hp ring ca nh l Markov: Nu P (x) l
mt a thc vi h s thc v c bc khng vt qu n th
0

max |P (x)| 6 n2 max |P (x)|.


|x|61

|x|61

Chng minh ca nh l Markov vt qu chng trnh ton THPT. Ta s tm cch chng minh
nhng trng hp ring khi n 6 3 ca nh l v kho st mt s bi ton xung quanh cc trng
hp .
Trong cc bi ton di y, bin s x ch nhn gi tr thc.

A - Bt ng thc Markov cho a thc bc nht


Bi PT. 1. Gi s a, b l hai s thc sao cho |ax + b| 6 1 khi |x| 6 1. Chng minh rng:
(i) |a| 6 1.
(ii) |bx + a| 6 1 khi |x| 6 1.

B - Bt ng thc Markov cho cc a thc bc hai v bc ba


Bi PT. 2. Gi s a, b, c l ba s thc sao cho cc gi tr ca a thc ax2 + bx + c ti 1, 0, 1
u thuc on [1, 1].
(i) Chng minh rng |2ax + b| 6 4 khi |x| 6 1.
(ii) Chng minh rng |cx2 + bx + a| 6 2 khi |x| 6 1.
Bi PT. 3. Gi s a, b, c, d l bn s thc sao cho cc gi tr , , v ca a thc
ax3 + bx2 + cx + d tng ng ti 1, 12 , 21 , 1 u thuc on [1, 1].
(i) Chng minh rng vi mi s thc A, B ta c |A + B| + |A B| = 2 max{|A|, |B|}.
(ii) Bng cch biu din 3ax2 + 2bx + c theo , , , v x, hy chng minh bt ng thc
|3ax2 + 2bx + c| 6 9 khi |x| 6 1.
(iii) Chng minh rng |dx3 + cx2 + bx + a| 6 4 khi |x| 6 1.

C - Hai bt ng thc khc cho cc tam thc


Bi PT. 4. Cho a, b, c l ba s thc v n l mt s nguyn dng. Gi s rng a thc
f (x) = ax2n + bx + c c cc gi tr ti 1, 0, 1 u thuc on [1, 1]. Chng minh
192

Tp ch Epsilon, S 09, 06/2016


(i) |f (x)| 6

2n1

2n1
4n n2n

khi |x| 6 1.

(ii) Vi mi 1 6 M < ta c |f (x)| 6 2M 2n 1 khi 1 6 |x| 6 M.

Ch : S hc
Th sinh c s dng kt qu ca cc cu trc trong chng minh ca cu sau.
S phn b ca s nguyn t trong tp hp s t nhin, cch xy dng cc s nguyn t l nhng
bi ton c quan tm t rt lu trong S hc. Di y chng ta s tm cch chng minh
trng hp c bit ca mt trong nhng kt qu p nht ca S hc: nh l Dirichlet v s
tn ti v hn s nguyn t trong mt cp s cng m s hng u tin v cng sai nguyn t
cng nhau.

A - Khi nim cp
Bi PT. 5. Cho a, n l cc s nguyn nguyn t cng nhau vi n > 2. Chng minh rng tn ti
mt s nguyn dng c nh nht vi tnh cht ac 1 (mod n).
S nguyn c c gi l cp ca a modulo n v c k hiu l ordn (a).
Bi PT. 6. Chng minh rng vi mi s nguyn dng k, ak 1 (mod n) khi v ch khi
ordn (a) | k.
Bi PT. 7. Chng minh rng ordn (a) | (n), trong k hiu hm s phi ca Euler, nh
ngha bi cng thc: (1) = 1 v vi n > 1,


Y
1
(n) =
1
.
p
p l c nguyn t ca n

(Nhc li rng k hiu x | y ngha l x l mt c ca y).

B - S tn ti s nguyn t trong mt s cp s cng


Bi PT. 8. Chng minh rng tn ti v hn s nguyn t c dng 4k + 3.
Bi PT. 9. (i) Chng minh rng c nguyn t l ca mt s c dng n2 + 1 lun ng d vi 1
modulo 4.
(ii) Chng minh rng tn ti v hn s nguyn t c dng 4k + 1.
Bi PT. 10. (i) Chng minh rng c nguyn t khc 3 ca s t nhin c dng n2 n + 1 phi
ng d vi 1 modulo 6.
(ii) Chng minh rng tn ti v hn s nguyn t c dng 6k + 1.

C - S tn ti s nguyn t trong cp s cng c dng nk + 1


193

Tp ch Epsilon, S 09, 06/2016


Trong cc bi tp sau y, ta c nh mt s nguyn k > 3.
Vi a l mt s nguyn khc 0 v p l mt s nguyn t, ta dng k hiu vp (a) ch s m ng
ca p trong phn tch ca a ra tha s nguyn t, ni cch khc pvp (a) | a nhng pvp (a)+1 6 | a.
Bi PT. 11. Gi s p l mt c nguyn t ca k k 1. K hiu c l cp ca k modulo p. Chng
minh rng vp (k c 1) = vp (k k 1).
Ta nhc li rng mt s nguyn dng c gi l khng c c chnh phng nu trong phn
tch ra tha s nguyn t ca n, mi s nguyn t u xut hin vi s m 6 1. Nh vy, cc s
nguyn dng khng c c chnh phng u tin l 1, 2, 3, 5, 6, 7, 10, 11, 13, 14, 15, 17, . . .
Bi PT. 12. K hiu D l tp tt c cc c nguyn dng d ca k sao cho d < k m kd l mt
s nguyn khng c c chnh phng. K hiu D1 = {d D | s c nguyn t ca kd l l },
D2 = {d D | s c nguyn t ca kd l chn }. t
A=

(k d 1), B =

dD1

(k d 1).

dD2

Chng minh rng vi mi s nguyn t p m p | k k 1 nhng p 6 1 (mod k) th ta c


vp (A) = vp (B) + vp (k k 1).
(Ta qui c A = 1 nu D1 = v tng t B = 1 nu D2 = ).
Bi PT. 13. Chng minh rng k k 1 c mt c nguyn t dng nk + 1.
Bi PT. 14. Chng minh rng tn ti v hn s nguyn t c dng nk + 1.

194

Tp ch Epsilon, S 09, 06/2016

thi ngh
Ch : i s
Trong cc phng php gii ton bt ng thc v cc tr, phng php hm s l mt phng
php hiu qu v c tm p dng rng. C th k n vic s dng tnh n iu ca hm s,
kho st hm s, bt ng thc Jensen, bt ng thc Karamata, phng php tip tuyn ... Thm
ch phng php dn bin, mt phng php c v ngoi thun i s cng mang hi hng ca
phng hm s.
Qua vic gii lot bi ton di y, chng ta s hiu su hn v tm p dng ca cc phng
php hm s, cng nh rn luyn kh nng p dng ng thi nhiu phng php x l nhng
tnh hung phc tp. Mc tiu cui cng ca chng ta l gii quyt bi ton sau:
Vi n l s nguyn dng cho trc v x1 , x2 , . . . , xn l cc s thc khng m tha mn iu
kin x1 + x2 + + xn = 1. Tm gi tr ln nht ca biu thc
P =

3x1 + 1
3x2 + 1
3xn + 1
+ 2
+ + 2
.
2x1 + 1 5x2 2x2 + 1
5xn 2xn + 1

5x21

Trong li gii bi ton ny, chng ta s s dng cc tnh cht c bn sau y ca hm s li v


hm s lm.
Xt hm s f (x) c o hm bc 2 trn on [a, b].
00

Nu f (x) 6 0 trn [a, b] th vi mi x1 , x2 , . . . , xn thuc [a, b] ta c




x1 + x2 + + xn
f (x1 ) + f (x2 ) + + f (xn ) 6 nf
.
n
(Bt ng thc Jensen)
00

Nu f (x) > 0 trn [a, b] th vi x, y, z, t thuc [a, b] m x 6 y 6 z 6 t v x + t = y + z


th f (y) + f (z) 6 f (x) + f (t).
1) Hy gii bi ton khi n = 2.
2) t f (x) =

3x+1
,
5x2x+1

hy tnh f (x). Chng minh rng vi 3 6 n 6 9 th ta c bt ng thc


 

 
1
1
1
0
f (x) 6 f
x
+f
,
n
n
n

vi mi x [0, 1]. T suy ra li gii bi ton trong trng hp 3 6 n 6 9.


3) Xt n = 10, hy tm mt hng s x > 0 sao cho bt ng thc
 

 
1
1
1
0
f (x) 6 f
x
+f
,
10
10
10
ng vi mi x [x , 1] (x khng nht thit phi l s nh nht tha mn iu kin ny). T
gii bi ton trong hai trng hp sau
195

Tp ch Epsilon, S 09, 06/2016


min{x1 , x2 , . . . , x10 } > x .
min{x1 , x2 , . . . , x10 } < x .
00

00

4) Tnh f (x), chng minh rng phng trnh f (x) = 0 c 2 nghim , thuc [0, 1] vi
1
1
< < < < 1.
15
2
5) Hy s dng 4. v cc tnh cht ca hm li, lm phn dn nhp, gii bi ton trong trng
hp n > 10.

Ch : Hnh hc
Hnh hc tam gic lun hp dn v k b. Cc nh ton hc t thi c i cho n tn ngy nay
vn khng ngng pht hin ra nhng tnh cht tuyt p lin quan n tam gic. Nh ton hc
v i Leonard Euler l ngi ng gp nhiu kt qu p v tam gic nh cng thc Euler
IO2 = R2 2Rr, nh l Euler v tam gic Pedal, ng thng Euler, ng trn Euler, ...
Trong bi ton ny, chng ta s cp ti mt kt qu t ni ting hn v cng kh hn, l
nh l Feuerbach v 5 ng trn.
Chng ta u bit rng trong mt tam gic bt k, c 4 ng trn tip xc ng thi vi cc
ng thng cha 3 cnh ca tam gic l ng trn ni tip v 3 ng trn bng tip. Chng
ta cng bit rng trong mt tam gic, chn 3 ng cao, trung im 3 cnh v trung im cc
on ni trc tm H vi cc nh cng nm trn mt ng trn, gi l ng trn 9 im Euler.
Chng ta cng bit rng tm ng trn Euler E l trung im ca on OH vi O l tm ng
trn ngoi tip tam gic ABC v bn knh ng trn Euler bng mt na bn knh ng trn
ngoi tip. Nhng kt qu tuyt p sau th chng ta t bit hn, v nu bit cng t ai bit cch
chng minh.
nh l Feuerbach: ng trn 9 im Euler tip xc vi ng trn ni tip v cc ng trn
bn ting ca tam gic.
Mc tiu ca lot bi ton ny l chng minh nh l trn. C nhiu cch khc nhau chng
minh nh l ny. y chng ta s s dng php nghch o.
0

Nhc li, php nghch o tm A bn knh k l php bin hnh bin im M thnh im M trn
0
tia AM tha mn iu kin AM AM = k 2 . Ta k hiu php nghch o ny l Inv(A, k 2 ).
Php nghch o tm A bn knh k c cc tnh cht c bn sau:
Nu A thuc ng trn C(O; r) tm O bn knh r th php nghch o (tm A bn knh
k) bin C(O; r) thnh ng thng l vung gc vi OA.
Nu l l mt ng thng khng qua A th nh ca l qua php nghch o l mt ng
trn m ng thng l vung gc vi ng thng ni A vi tm ng trn .
Nu A khng nm trn C(O; r) th nh ca C(O; r) qua php nghch o l ng trn
0
0
C (O; r ) vi
k2
0
r =r
.
|AO2 r2 |
196

Tp ch Epsilon, S 09, 06/2016


0

Nu M , N tng ng l nh ca M v N qua php nghch o tm A bn knh k th


0

M N = MN

k2
.
AM AN

Cho A l mt im khng nm trn ng trn C(O; r). Gi P C(A) l phng tch ca


im A i vi ng trn C. Khi php nghch o tm A bn knh k vi k 2 = P C(A)
s bit C(O; r) thnh chnh n.
1) Cho tam gic ABC. Xt Inv l php nghch o tm A bn knh k nh mt nh x t R2 \ {A}
vo R2 \ {A}. Chng minh rng nu C(O; R) l ng trn ngoi tip tam gic ABC th
Inv(C(O; R)) l ng thng l i song song ng thng BC i vi gc BAC (tc l l v
BC to vi phn gic ca gc BAC nhng gc ng v b nhau).
2) Cho tam gic ABC. ng trn ni tip tam gic ABC tm I bn knh r tip xc vi cc
cnh BC, CA, AB ti P, Q, R tng ng. t p = a+b+c
l na chu vi ca tam gic:
2
a) Chng minh rng CP = CQ = pc, BP = BR = pb, AR = AQ = pa.
b) Gi s ng trn bng tip gc A tm Ia bn knh ra tip xc vi cnh BC v cc cnh
CA, AB ni di Pa , Qa , Ra . Chng minh rng BPa = pc v CPa = pb.
c) Gi A3 l chn ng phn gic trong ca gc A. Chng minh rng
BA3 =

ac
ab
, CA3 =
.
b+c
b+c

d) Gi A1 l trung im cnh BC v A2 l chn ng cao h t A xung BC. Chng minh


A1 A2 =

|b2 c2 |
.
2a

3) Cho C(O; r) l ng trn tm O bn knh r v A l mt im khng thuc C. Xt php


nghch o tm A bn knh k vi k 2 = P C(A) l phng tch ca im A i vi ng trn
C(O; r). Khi ng trn C(O; r) bt bin i vi php nghch o Inv(A; P C(A)).
4) Trong tam gic ABC gi A1 , A2 , A3 ln lt l trung im cnh BC, chn ng cao h t
nh A v chn ng phn gic trong gc A, P l tip im ca ng trn ni tip vi BC.
Chng minh rng A1 P = A1 A2 A1 A3 .
5) Xt tam gic ABC vi cc im A1 , A2 , A3 , P, Pa c k hiu nh trn. Xt php nghch
o tm A1 bn knh k vi k 2 = A1 P, c k hiu l Inv.
a) Chng minh rng C(I; r) v C(Ia ; ra ) u bt bin i vi Inv.
b) Gi s C9 l ng trn 9 im Euler v d l nh ca C9 qua php nghch o Inv, d =
Inv(C9 ), chng minh rng d i song song ng thng BC i vi gc BAC.
c) Chng minh rng d i qua A3 .
0

d) Gi B C l tip tuyn chung th hai ca hai ng trn C(I; r) v C(Ia , ra ) vi B thuc


0
0
0
AB, C thuc AC (tip tuyn chung th nht chnh l BC). Chng minh rng B C i
0
0
song song BC i vi gc BAC, suy ra d = B C v t suy ra C9 tip xc vi C(I; r)
v C(Ia ; ra ).
197

You might also like